Download as pdf or txt
Download as pdf or txt
You are on page 1of 238

 

Therefore, the solution of the system is (3, 5).

ANSWER:  
3-1 Solving Systems of Equations (3, 5)

Solve each system of equations by using a table. 2. 


1. 

SOLUTION:  
SOLUTION:   Write each equation in slope-intercept form.
The equations are in slope-intercept form.  
Make a table to find a solution that satisfies both
equations.
 
 
Also:

 
Therefore, the solution of the system is (3, 5).
 
ANSWER:   Make a table to find a solution that satisfies both
(3, 5) equations.
 
2. 

SOLUTION:  
Write each equation in slope-intercept form.  
  Therefore, the solution of the system is (2, 7).

ANSWER:  
(2, 7)
  Solve each system of equations by graphing.
Also: 3. 

SOLUTION:  
Graph the equations y = –3x + 6 and 2y = 10x – 36.
 
 
Make a table to find a solution that satisfies both
equations.
 

   
Therefore, the solution of the system is (2, 7). The lines intersect at the point (3, –3). So, the
solution of the system is (3, –3).
ANSWER:  
(2, 7) ANSWER:  
eSolutions Manual - Powered by Cognero Page 1
(3, –3)
Solve each system of equations by graphing.
3. 
4. 
  The lines intersect at the point (–4, –5). So, the
Therefore, the solution of the system is (2, 7). solution of the system is (–4, –5).

ANSWER:  
3-1 Solving Systems of Equations ANSWER:  
(2, 7) (–4, –5)

Solve each system of equations by graphing. 5. 


3. 

SOLUTION:  
SOLUTION:   Graph the equations y = 0.5x + 4 and 3y = 4x – 3.
Graph the equations y = –3x + 6 and 2y = 10x – 36.  
 

 
  The lines intersect at the point (6, 7). So, the solution
The lines intersect at the point (3, –3). So, the of the system is (6, 7).
solution of the system is (3, –3).
ANSWER:  
ANSWER:   (6, 7)
(3, –3)
6. 
4. 
SOLUTION:  
Graph the equations –3y = 4x + 11 and 2x + 3y = –7.
SOLUTION:  
 
Graph the equations y = –x – 9 and 3y = 5x + 5.
 

 
The lines intersect at the point (–2, –1). So, the
  solution of the system is (–2, –1).
The lines intersect at the point (–4, –5). So, the
solution of the system is (–4, –5). ANSWER:  
(–2, –1)
ANSWER:  
(–4, –5) 7. 
5. 
SOLUTION:  
Graph the equations 4x + 5y = –41 and 3y – 5x = 5.
SOLUTION:  
 
Graph the equations y = 0.5x + 4 and 3y = 4x – 3.
 

eSolutions Manual - Powered by Cognero Page 2


The lines intersect at the point (–2, –1). So, the
solution of the system is (–2, –1).

ANSWER:  
3-1 Solving Systems of Equations
(–2, –1)
 
7.  a. Write equations that represent the cost of printing
digital photos at each lab.
b. Under what conditions is the cost to print digital
SOLUTION:   photos the same at both stores?
Graph the equations 4x + 5y = –41 and 3y – 5x = 5. c. When is it best to use EZ Online Digital Photos
  and when is it best to use the local pharmacy?
 
SOLUTION:  
a. Let x be the charge for a digital photo and y be the
total cost.
 
At EZ Online Digital Photos:
y = 0.15x + 2.70
   
The lines intersect at the point (–4, –5). So, the
At Local Pharmacy:
solution of the system is (–4, –5).
y = 0.25x
ANSWER:    
(–4, –5) b. Equate the total costs and find x.
 
8. 

SOLUTION:  
When x = 27, y = 6.75.
Graph the equations 8x – 4y = 50 and x + 4y = –2.
   
Therefore, the cost will be the same at both stores
when 27 photos are taken.
c.  

 
That is, you should use EZ Online Digital Photos if
  you are printing more than 27 digital photos and the
The lines intersect at the point (6, –2). So, the local pharmacy if you are printing fewer than 27
solution of the system is (6, –2). photos.

ANSWER:   ANSWER:  
(6, –2) a.
b.  for 27 photos
9. CCSS MODELING Refer to the table below. c. You should use EZ Online Digital Photos if you
  are printing more than 27 digital photos and the local
pharmacy if you are printing fewer than 27 photos.

Graph each system of equations and describe it


as consistent and independent,
consistent and dependent, or inconsistent.

  10. 
a. Write equations that represent the cost of printing
eSolutions Manual
digital - Powered
photos by lab.
at each Cognero SOLUTION:   Page 3

b. Under what conditions is the cost to print digital


photos the same at both stores?
a.
b.  for 27 photos
c. You should use EZ Online Digital Photos if you
3-1 Solving Systems
are printing of Equations
more than 27 digital photos and the local inconsistent
pharmacy if you are printing fewer than 27 photos.

Graph each system of equations and describe it


as consistent and independent, 11. 
consistent and dependent, or inconsistent.
SOLUTION:  
10. 

SOLUTION:  

 
Because the equations are equivalent, their graphs
are the same line. The system is consistent and
  dependent as it has an infinite number of solutions.
The lines are parallel. They do not intersect and there  
is no solution. So, the system is inconsistent.
ANSWER:  
ANSWER:  

inconsistent consistent,  dependent
 

11.  12. 

SOLUTION:   SOLUTION:  

   
Because the equations are equivalent, their graphs The lines intersect at one point, so there is one
are the same line. The system is consistent and solution. The system is consistent and independent.
dependent as it has an infinite number of solutions.  
   
ANSWER:   ANSWER:  

eSolutions Manual - Powered by Cognero Page 4


consistent,  dependent
3-1 Solving Systems of Equations consistent,  independent.
   

Solve each system of equations by using


12.  substitution.

SOLUTION:   13. 

SOLUTION:  
Solve the equation  for x.

 
Substitute 3 – 5y for x in the equation
 and solve for y.
   
The lines intersect at one point, so there is one
solution. The system is consistent and independent.
 
 
ANSWER:    
Substitute 1 for y into either original equation and
solve for x.
 

consistent,  independent.  
  The solution is (–2, 1).

Solve each system of equations by using ANSWER:  


substitution. (–2, 1)

13. 
14. 
SOLUTION:  
Solve the equation  for x. SOLUTION:  
Substitute 2x – 10 for y in the equation
   and solve for x.
Substitute 3 – 5y for x in the equation  
 and solve for y.
 

 
Substitute 3 for x into either original equation and
solve for y.
   
Substitute 1 for y into either original equation and
solve for x.
 
eSolutions Manual - Powered by Cognero Page 5
 
 
The solution is (–2, 1).
The solution is (3, –4).
ANSWER:  
3-1 Solving ANSWER:  
(–2, 1) Systems of Equations
(3, –4)

14.  15. 

SOLUTION:   SOLUTION:  
Substitute 2x – 10 for y in the equation Solve the equation  for a.
 and solve for x.  
 

 
Substitute –4b – 4 for a in the equation
 and solve for b.
 
 
Substitute 3 for x into either original equation and
solve for y.
 

 
Substitute –2 for b into either original equation and
solve for a.
   
The solution is (3, –4).

ANSWER:  
(3, –4)

15. 
 
The solution is (4, –2).
SOLUTION:  
Solve the equation  for a. ANSWER:  
  (4, –2)

16. 
 
Substitute –4b – 4 for a in the equation SOLUTION:  
 and solve for b. Solve the equation  for a.
 
 
Substitute 3b – 22 for a in the equation
 and solve for b.
 
 
Substitute –2 for b into either original equation and
solve for a.
 
 
Substitute 6 for b into either original equation and
eSolutions Manual - Powered by Cognero
solve for a. Page 6
 
   
The solution is (4, –2). The solution is (–4, 6).

ANSWER:  
3-1 Solving Systems of Equations ANSWER:  
(4, –2) (–4, 6)

16.  17. 

SOLUTION:   SOLUTION:  
Solve the equation  for a. Solve the equation  for y.
 
 
Substitute 3b – 22 for a in the equation
 and solve for b.
 
 
Substitute 11 – 2x for y in the equation
 and solve for x.
 

 
Substitute 6 for b into either original equation and
solve for a.
   
Substitute 5 for x into either original equation and
solve for y.
 

 
The solution is (–4, 6).

ANSWER:  
(–4, 6)  
The solution is (5, 1).
17. 
ANSWER:  
(5, 1)
SOLUTION:  
Solve the equation  for y.
18. 
 
SOLUTION:  
Solve the equation  for c.
 
 
Substitute 11 – 2x for y in the equation
 and solve for x.
 
 
Substitute  for c in the equation
 and solve for d.
   
Substitute 5 for x into either original equation and
solve for y.
  Manual - Powered by Cognero
eSolutions Page 7
   
The solution is (5, 1). The solution is (–3, 2).

ANSWER:  
3-1 Solving Systems of Equations ANSWER:  
(5, 1) (–3, 2)

Solve each system of equations by using


18.  elimination.

SOLUTION:   19. 
Solve the equation  for c.
  SOLUTION:  
Multiply the equation  by 2.
 

 
 
Substitute  for c in the equation Add the equations to eliminate one variable.
 and solve for d.  
 

 
Substitute 7 for z into either original equation and
  solve for w.
Substitute 2 for d into either original equation and  
solve for c.
 

 
The solution is (–2, 7).
 
The solution is (–3, 2). ANSWER:  
(–2, 7)
ANSWER:  
(–3, 2)
20. 
Solve each system of equations by using
elimination. SOLUTION:  
19.  Add the equations to eliminate y.
 
SOLUTION:  
Multiply the equation  by 2.
 

 
  Substitute 8 for x into either original equation and
Add the equations to eliminate one variable. solve for y.
   
eSolutions Manual - Powered by Cognero Page 8
 
 
The solution is (–2, 7).
The solution is (8, 1).
ANSWER:  
3-1 Solving ANSWER:  
(–2, 7) Systems of Equations
(8, 1)

20.  21. 

SOLUTION:   SOLUTION:  
Add the equations to eliminate y. Multiply the equation  by –2.
   

 
Add the equations to eliminate one variable.
 
 
Substitute 8 for x into either original equation and
solve for y.
 

 
Substitute –4 for a into either original equation and
solve for b.
 
 
The solution is (8, 1).

ANSWER:  
(8, 1)
 
21.  The solution is (–4, –3).

SOLUTION:   ANSWER:  
Multiply the equation  by –2. (–4, –3)
 
22. 

  SOLUTION:  
Add the equations to eliminate one variable. The coefficients of b-variables are 3 and 2 and their
  least common multiple is 6, so multiply each equation
by the value that will make the b-coefficient 6.
 

 
 
Substitute –4 for a into either original equation and
Substitute –1 for a into either original equation and
solve for b.
solve for b.
 
 

eSolutions Manual - Powered by Cognero Page 9


  Because 0 = 92 is not true, this system has no
The solution is (–4, –3). solution.

ANSWER:  
3-1 Solving Systems of Equations ANSWER:  
(–4, –3) No solution

22.  24. 

SOLUTION:   SOLUTION:  
The coefficients of b-variables are 3 and 2 and their The coefficients of y-variables are 4 and 5 and their
least common multiple is 6, so multiply each equation least common multiple is 20, so multiply each
by the value that will make the b-coefficient 6. equation by the value that will make the y-coefficient
  20.
 

 
Substitute –1 for a into either original equation and  
solve for b. Substitute 1 for x into either original equation and
  solve for y.
 

 
The solution is (–1, 1).  
The solution is (1, –6).
ANSWER:  
(–1, 1) ANSWER:  
(1, –6)
23. 
25. MULTIPLE CHOICE What is the solution of the
linear system?
SOLUTION:  
The coefficients of a-variables are 5 and 2 (in        
absolute sense) and their least common multiple is 10,
A (8, –10) 
so multiply each equation by the value that will make
the y-coefficient 10. B (2, –2) 
  C (–10, 14) 
D no solution
SOLUTION:  
Subtract the equations to eliminate one variable.
 
 
Because 0 = 92 is not true, this system has no
solution.

ANSWER:  
No solution

24.   
Substitute –2 for y into either original equation and
solve for x.
SOLUTION:    
The coefficients of y-variables are 4 and 5 and their
eSolutions Manual - Powered by Cognero Page 10
least common multiple is 20, so multiply each
equation by the value that will make the y-coefficient
20.
   
The solution is (1, –6). The solution is (2, –2). So the correct choice is B.

ANSWER:  
3-1 Solving Systems of Equations ANSWER:  
(1, –6) B

25. MULTIPLE CHOICE What is the solution of the Solve each system of equations by using a table.
linear system?
26. 
       
SOLUTION:  
A (8, –10) 
B (2, –2)  y = 5x + 3
C (–10, 14)  y=x–9
 
D no solution
Make a table to find a solution that satisfies both
SOLUTION:   equations.
Subtract the equations to eliminate one variable.  
 

 
Therefore, the solution of the system is (–3, –12).
 
Substitute –2 for y into either original equation and ANSWER:  
solve for x. (–3, –12)
 

27. 

SOLUTION:  
Write each equation in slope-intercept form.
 

 
The solution is (2, –2). So the correct choice is B.

ANSWER:  
B  
Also:
Solve each system of equations by using a table.
26. 

SOLUTION:  
y = 5x + 3  
y=x–9 Make a table to find a solution that satisfies both
  equations.
Make a table to find a solution that satisfies both  
equations.
 

 
Therefore, the solution of the system is (4, –1).
 
eSolutions Manual - Powered by Cognero Page 11
Therefore, the solution of the system is (–3, –12). ANSWER:  
(4,–1)
ANSWER:  
 
 
Therefore, the solution of the system is (–3, –12).
Therefore, the solution of the system is (4, –1).
ANSWER:  
3-1 Solving ANSWER:  
(–3, –12)Systems of Equations
(4,–1)

27.  28. 

SOLUTION:   SOLUTION:  
Write each equation in slope-intercept form. Write each equation in slope-intercept form.
   

Also:

 
Also:
 
Make a table to find a solution that satisfies both
equations.
 
 
Make a table to find a solution that satisfies both
equations.
 

 
Therefore, the solution of the system is (4, 3).

ANSWER:  
  (4,3)
Therefore, the solution of the system is (4, –1).
29. FUNDRAISER To raise money for new uniforms,
ANSWER:   the band boosters sell T-shirts and hats. The cost and
(4,–1) sale price of each item is shown. The boosters spend
a total of $2000 on T-shirts and hats. They sell all of
the merchandise, and make $3375. How many T-
28.  shirts did they sell?
 
SOLUTION:  
Write each equation in slope-intercept form.
 

Also:

SOLUTION:  
Let x be the number of T-shirts.
Let y be the number of hats.
  The system of equations is
Make a table to find a solution that satisfies both  
equations.
 
 
eSolutions Manual - Powered by Cognero Page 12
Solve the equation of cost price for y.
 
   
Therefore, the solution of the system is (4, 3). Thus, they sold 250 T-shirts.

ANSWER:  
3-1 Solving Systems of Equations ANSWER:  
(4,3) 250 T-shirts

29. FUNDRAISER To raise money for new uniforms, Solve each system of equations by graphing.
the band boosters sell T-shirts and hats. The cost and
30. 
sale price of each item is shown. The boosters spend
a total of $2000 on T-shirts and hats. They sell all of
the merchandise, and make $3375. How many T- SOLUTION:  
shirts did they sell? Graph the equations –3x + 2y = –6 and –5x + 10y =
  30.
 

SOLUTION:  
Let x be the number of T-shirts.
Let y be the number of hats.
The system of equations is  
  The lines intersect at the point (6, 6). So, the solution
of the system is (6, 6).

ANSWER:  
 
(6, 6)
Solve the equation of cost price for y.
 
31. 

SOLUTION:  
Substitute  for y in the equation of sale Graph the equations 4x + 3y = –24 and 8x – 2y = –
16.
price and solve for x.  
 

 
  The lines intersect at the point (–3, –4). So, the
Thus, they sold 250 T-shirts. solution of the system is (–3, –4).
ANSWER:   ANSWER:  
250 T-shirts (–3,–4)
Solve each system of equations by graphing.
32. 
30. 
SOLUTION:  
SOLUTION:  
eSolutions Manual - Powered by Cognero Graph the equations 6x – 5y = 17 and 6x + 2y =Page
31. 13
Graph the equations –3x + 2y = –6 and –5x + 10y =  
30.
The lines intersect at the point (–3, –4). So, the are the same line. So, the system has an infinite
solution of the system is (–3, –4). number of solutions.

ANSWER:  
3-1 Solving Systems of Equations ANSWER:  
(–3,–4) Infinite solutions

32.  34. 

SOLUTION:   SOLUTION:  
Graph the equations 6x – 5y = 17 and 6x + 2y = 31. Graph the equations y – 3x = – 29 and 9x – 6y =
  102.
 

 
The lines intersect at the point (4.5, 2). So, the  
solution of the system is (4.5, 2). The lines intersect at the point (8, –5). So, the
ANSWER:   solution of the system is (8, –5).
(4.5, 2) ANSWER:  
(8,–5)
33. 
35. 
SOLUTION:  
Graph the equations –3x – 8y = 12 and 12x + 32y = SOLUTION:  
–48. Graph the equations –10x + 4y = 7 and 2x – 5y = 7.
   

   
Because the equations are equivalent, their graphs The lines intersect at the point (–1.5, –2). So, the
are the same line. So, the system has an infinite solution of the system is (–1.5, –2).
number of solutions.
ANSWER:  
ANSWER:  
(–1.5,–2)
Infinite solutions
36. CCSS MODELING Jerilyn has a $10 coupon and a
34.  15% discount coupon for her favorite store. The
store has a policy that only one coupon may be used
per purchase. When is it best for Jerilyn to use the
SOLUTION:  
$10 coupon, and when is it best for her to use the
Graph the equations y – 3x = – 29 and 9x – 6y = 15% discount coupon?
102.
  Manual - Powered by Cognero
eSolutions SOLUTION:   Page 14
Let $x be the total cost (excluding discounts) of the
items that Jerilyn purchased.
The lines intersect at the point (–1.5, –2). So, the $66.67.
solution of the system is (–1.5, –2).
ANSWER:  
ANSWER:  
3-1 Solving Systems of Equations $10 coupon for a purchase less than $66.67 and 15%
(–1.5,–2) discount coupon for a purchase over $66.67.

36. CCSS MODELING Jerilyn has a $10 coupon and a Graph each system of equations and describe it
15% discount coupon for her favorite store. The as consistent and independent, consistent and
store has a policy that only one coupon may be used dependent, or inconsistent.
per purchase. When is it best for Jerilyn to use the
$10 coupon, and when is it best for her to use the 37. 
15% discount coupon?
SOLUTION:  
SOLUTION:  
Graph the equations y = 3x – 4 and y = 6x – 8.
Let $x be the total cost (excluding discounts) of the
 
items that Jerilyn purchased.
If she uses the $10 coupon, the cost will be $(x –
10).
If she uses the 15% coupon, the cost will be $0.85x.
 
Assume:
 

 
 
  The lines intersect at one point, so there is one
So, when the total cost of the items purchased is less solution. The system is consistent and independent.
than $66.67, it is best to use the $10 coupon. The
15% discount is best when the purchase is more than ANSWER:  
$66.67.

ANSWER:  
$10 coupon for a purchase less than $66.67 and 15%
discount coupon for a purchase over $66.67.

Graph each system of equations and describe it


as consistent and independent, consistent and
dependent, or inconsistent.
37. 
38. 
SOLUTION:  
Graph the equations y = 3x – 4 and y = 6x – 8.
SOLUTION:  
 
Graph the equations y = 2x – 1 and y = 2x + 6.
 

 
The lines intersect at one point, so there is one
solution. The system is consistent and independent.  
The lines are parallel. They do not intersect and there
ANSWER:   is no solution. So, the system is inconsistent.
eSolutions Manual - Powered by Cognero Page 15

ANSWER:  
3-1 Solving Systems of Equations

38.  39. 

SOLUTION:   SOLUTION:  
Graph the equations y = 2x – 1 and y = 2x + 6. Graph the equations 2x + 5y = 10 and –4x – 10y =
  20.
 

 
The lines are parallel. They do not intersect and there  
is no solution. So, the system is inconsistent. The lines are parallel. They do not intersect and there
is no solution. So, the system is inconsistent.
ANSWER:  
ANSWER:  

39. 
40. 
SOLUTION:  
Graph the equations 2x + 5y = 10 and –4x – 10y = SOLUTION:  
20. Graph the equations x – 6y = 12 and 3x + 18y = 14.
   

   
The lines are parallel. They do not intersect and there The lines intersect at one point, so there is one
is no solution. So, the system is inconsistent. solution. The system is consistent and independent.

ANSWER:   ANSWER:  
eSolutions Manual - Powered by Cognero Page 16
3-1 Solving Systems of Equations

40.  41. 

SOLUTION:   SOLUTION:  
Graph the equations x – 6y = 12 and 3x + 18y = 14. Graph the equations –5x – 6y = 13 and 12y + 10x =
  –26.
 

 
The lines intersect at one point, so there is one  
solution. The system is consistent and independent. Because the equations are equivalent, their graphs
are the same line. The system is consistent and
ANSWER:   dependent as it has an infinite number of solutions.

ANSWER:  

41. 
42. 
SOLUTION:  
Graph the equations –5x – 6y = 13 and 12y + 10x =
–26. SOLUTION:  
  Graph the equations 8y – 3x = 15 and –16y + 6x = –
30.
 

 
Because the equations are equivalent, their graphs
are the same line. The system is consistent and  
dependent as it has an infinite number of solutions. Because the equations are equivalent, their graphs
are the same line. The system is consistent and
ANSWER:   dependent as it has an infinite number of solutions.
eSolutions Manual - Powered by Cognero Page 17

ANSWER:  
3-1 Solving Systems of Equations

Solve each system of equations by using


42.  substitution.

SOLUTION:   43. 
Graph the equations 8y – 3x = 15 and –16y + 6x = –
30. SOLUTION:  
  Solve the equation  for x.
 

Substitute 6 –3y for x in the equation


 and solve for y.
 

 
Because the equations are equivalent, their graphs  
are the same line. The system is consistent and Because 0 = 0 is true, the system has infinite
dependent as it has an infinite number of solutions. solutions.
ANSWER:   ANSWER:  
Infinite solutions

44. 

SOLUTION:  
Solve the equation  for x.
 

Solve each system of equations by using


substitution.
Substitute 14 + 4y for x in the equation
43.   and solve for y.
 
SOLUTION:  
Solve the equation  for x.
 

Substitute 6 –3y for x in the equation


 
 and solve for y.
Substitute –4 for y into either original equation and
  solve for x.
 

 
Because 0 = 0 is true, the system has infinite
eSolutions Manual - Powered by Cognero
solutions. Page 18

ANSWER:    
Because 0 = 0 is true, the system has infinite
 
solutions.
The solution is (–2, –4).
ANSWER:  
3-1 Solving Systems of Equations ANSWER:  
Infinite solutions
(–2, –4)

44.  45. 

SOLUTION:   SOLUTION:  
Solve the equation  for x. Solve the equation  for x.
   

Substitute 14 + 4y for x in the equation


 and solve for y.  
 
Substitute  for x in the equation
 and solve for y.
 

 
Substitute –4 for y into either original equation and
solve for x.
 

 
Substitute 4 for y into either original equation and
solve for x.
 

 
The solution is (–2, –4).

ANSWER:  
(–2, –4)  
The solution is (8, 4).
45. 
ANSWER:  
(8, 4)
SOLUTION:  
Solve the equation  for x.
46. 
 
SOLUTION:  
Solve the equation  for b.
 
 
Substitute  for x in the equation
 
 and solve for y. Substitute –16 –3a for b in the equation
   and solve for a.
 
eSolutions Manual - Powered by Cognero Page 19
   
The solution is (8, 4). The solution is (–6, 2).

ANSWER:  
3-1 Solving Systems of Equations ANSWER:  
(8, 4) (–6, 2)

46.  47. 

SOLUTION:   SOLUTION:  
Solve the equation  for b. Solve the equation  for c.
   

   
Substitute –16 –3a for b in the equation Substitute (–4 – d) for c in the equation
 and solve for a.  and solve for d.
   

 
  Substitute –1 for d into either original equation and
Substitute –6 for a into either original equation and solve for c.
solve for b.  
 

 
  The solution is (–3, –1).
The solution is (–6, 2).
ANSWER:  
ANSWER:   (–3, –1)
(–6, 2)
48. 
47. 
SOLUTION:  
SOLUTION:   Solve the equation  for g.
Solve the equation  for c.  
 

   
Substitute (–4 – d) for c in the equation
 and solve for d. Substitute  for g in the equation
 
 and solve for f .
 

  Manual - Powered by Cognero


eSolutions Page 20
Substitute –1 for d into either original equation and  
solve for c.
Because 0 = 96 is not true, this system has no
  Because 0 = 96 is not true, this system has no
The solution is (–3, –1). solution.

ANSWER:  
3-1 Solving Systems of Equations ANSWER:  
(–3, –1) No solution

49. TENNIS At a park, there are 38 people playing


48.  tennis. Some are playing doubles, and some are
playing singles. There are 13 matches in progress. A
SOLUTION:   doubles match requires 4 players, and a singles
Solve the equation  for g. match requires 2 players.
a. Write a system of two equations that represents
 
the number of singles and doubles matches going on.
b. How many matches of each kind are in progress?
SOLUTION:  
  a. Let x be the number of doubles matches.
Let y be the number of singles matches.
Substitute  for g in the equation The system of equations that represents the number
of singles and doubles matches going on is:
 and solve for f .
 
 

 
b. Solve the equation  for x.
 
 
Because 0 = 96 is not true, this system has no Substitute 13 – y for x in the equation
solution.
 and solve for y.
ANSWER:    
No solution

49. TENNIS At a park, there are 38 people playing


tennis. Some are playing doubles, and some are
playing singles. There are 13 matches in progress. A
doubles match requires 4 players, and a singles
match requires 2 players.
 
a. Write a system of two equations that represents
Substitute 7 for y into either original equation and
the number of singles and doubles matches going on.
solve for x.
b. How many matches of each kind are in progress?
 
SOLUTION:  
a. Let x be the number of doubles matches.
Let y be the number of singles matches.  
The system of equations that represents the number There are 6 doubles matches and 7 singles matches
of singles and doubles matches going on is: are in progress.
 
ANSWER:  
a. x + y = 13 and 4x + 2y = 38
  b. 6 doubles matches and 7 singles matches
b. Solve the equation  for x. Solve each system of equations by using
  elimination.
50. 
Substitute 13 – y for x in the equation
 and solve for y.
  SOLUTION:  
eSolutions Manual - Powered by Cognero Page 21
Multiply the equation  by –4.
 
are in progress.  
The solution is (3, 3).
ANSWER:  
ANSWER:  
a. x + y =
3-1 Solving 13 and 4x
Systems of + 2y = 38
Equations (3, 3)
b. 6 doubles matches and 7 singles matches

Solve each system of equations by using


elimination. 51. 

50.  SOLUTION:  
Add the equations to eliminate one variable.
SOLUTION:    
Multiply the equation  by –4.
 

   
Add the equations to eliminate one variable. Substitute 0 for a into either original equation and
  solve for b.
 

   
Substitute 3 for x into either original equation and The solution is (0, 4).
solve for y.
ANSWER:  
 
(0,4)

52. 

SOLUTION:  
 
Subtract the equations to eliminate one variable.
The solution is (3, 3).
 
ANSWER:  
(3, 3)

51. 
 
SOLUTION:   Substitute –5 for j into either original equation and
Add the equations to eliminate one variable. solve for k.
   

 
 
Substitute 0 for a into either original equation and
solve for b. The solution is (–5, –4).
  ANSWER:  
(–5, –4)
eSolutions Manual - Powered by Cognero Page 22

53. 
 
  Because 0 = 192 is not true, the system has no
The solution is (0, 4). solution.

ANSWER:  
3-1 Solving Systems of Equations ANSWER:  
(0,4) No solution

52.  54. 

SOLUTION:   SOLUTION:  
Subtract the equations to eliminate one variable. The coefficients of b-variables are 5 and 2 and their
  least common multiple is 10, so multiply each
equation by the value that will make the b-coefficient
10.
 

 
Substitute –5 for j into either original equation and  
solve for k. Substitute –5 for a into either original equation and
  solve for b.
 

 
The solution is (–5, –4).  
The solution is (–5, –3).
ANSWER:  
(–5, –4) ANSWER:  
(–5, –3)
53. 
55. 
SOLUTION:  
Multiply the equation  by 4. SOLUTION:  
  Multiply the equation  by 2.
 

 
Add the equations to eliminate one variable.  
  Add the equations to eliminate one variable.
 

 
Because 0 = 192 is not true, the system has no
solution.  
Substitute 8 for r into either original equation and
ANSWER:  
solve for t.
No solution  

54. 
eSolutions Manual - Powered by Cognero Page 23

SOLUTION:  
   
The solution is (–5, –3). The solution is (8, –6).

ANSWER:  
3-1 Solving Systems of Equations ANSWER:  
(–5, –3) (8, –6)

55.  56. 

SOLUTION:   SOLUTION:  
Multiply the equation  by 2. The coefficients of f -variables are 5 and 9 and their
  least common multiple is 45, so multiply each
equation by the value that will make the f -coefficient
45.
 
 
Add the equations to eliminate one variable.
 

 
Substitute –2 for d into either original equation and
solve for f .
 
 
Substitute 8 for r into either original equation and
solve for t.
 

 
The solution is (–2, –4).

ANSWER:  
  (–2, –4)
The solution is (8, –6).

ANSWER:   57. 
(8, –6)
SOLUTION:  
Rewrite the equations in the standard form
56. 
 
SOLUTION:  
The coefficients of f -variables are 5 and 9 and their
least common multiple is 45, so multiply each
 
equation by the value that will make the f -coefficient
45. The coefficients of v-variables are 5 and 8 and their
least common multiple is 40, so multiply each
 
equation by the value that will make the v-coefficient
40.
 

 
Substitute –2 for d into either original equation and
solve for f .
   
Substitute 5 for u into either original equation and
solve for v.
eSolutions Manual - Powered by Cognero Page 24
 
   
The solution is (–2, –4). The solution is (5, 4).

ANSWER:  
3-1 Solving Systems of Equations ANSWER:  
(–2, –4) (5, 4)

57.  58. 

SOLUTION:   SOLUTION:  
Rewrite the equations in the standard form Rewrite the equation  in the standard 
form
   

 
  Multiply the equation  by 5.
The coefficients of v-variables are 5 and 8 and their  
least common multiple is 40, so multiply each
equation by the value that will make the v-coefficient
40.
   
Add the equations to eliminate one variable.
 

 
Substitute 5 for u into either original equation and
solve for v.  
  Because 0 = 0 is true, the system has infinite
solutions.

ANSWER:  
Infinite solutions

Use a graphing calculator to solve each system


of equations. Round the coordinates of the
  intersection to the nearest hundredth.
The solution is (5, 4).
59. 
ANSWER:  
(5, 4) SOLUTION:  
Write each equation in the form y = mx + b.
58.   

SOLUTION:  
Rewrite the equation  in the standard 
form
 
 
 
Enter  as Y1 and   as Y2. 
Multiply the equation  by 5.
  Then graph the lines.
KEYSTROKES:  Y =   (    5     −   1   5  
 )     ÷  1   2  ENTER     (  (−) 6  ·  1    
     +    1    1   )    ÷  4   ·   2    ENTER 
Add the equations to eliminate one variable.   ZOOM    6
eSolutions Manual - Powered by Cognero Page 25
   
Find the intersection of the lines.
KEYSTROKES:
Because 0 = 0 is true, the system has infinite hundredth are .
solutions.
ANSWER:  
ANSWER:  
3-1 Solving Systems of Equations (2.07, –0.39)
Infinite solutions

Use a graphing calculator to solve each system


of equations. Round the coordinates of the 60. 
intersection to the nearest hundredth.
SOLUTION:  
59. 
Write each equation in the form y = mx + b.
 
SOLUTION:  
Write each equation in the form y = mx + b.
 

 
Enter  as Y1 and   as 
  Y2. Then graph the lines.
Enter  as Y1 and   as Y2.  KEYSTROKES:  Y =   (    3   ·   8      + 
  1   9   )     ÷  2  ·    9    ENTER     (  6  · 
Then graph the lines.
 6       +    2   3   )   ÷  5   ·   4    ENTER 
KEYSTROKES:  Y =   (    5     −   1   5  
 ZOOM  3  ENTER  ENTER  
 )     ÷  1   2  ENTER     (  (−) 6  ·  1    
 
   +    1    1   )    ÷  4   ·   2    ENTER  Find the intersection of the lines.
  ZOOM    6 KEYSTROKES:   2nd   [CALC] 5  ENTER   
  ENTER   ENTER
Find the intersection of the lines.  
KEYSTROKES:   2nd   [CALC] 5  ENTER   
ENTER   ENTER
 

 
The coordinates of the intersection to the nearest
hundredth are .
 
The coordinates of the intersection to the nearest ANSWER:  
hundredth are . (–26.01, –27.54)

ANSWER:  
61. 
(2.07, –0.39)
SOLUTION:  
60.  Write each equation in the form y = mx + b.
 
SOLUTION:  
Write each equation in the form y = mx + b.
 
eSolutions Manual - Powered by Cognero
  Page 26

Enter  as Y1 and   as 


The coordinates of the intersection to the nearest The coordinates of the intersection to the nearest
hundredth are . hundredth are .

ANSWER:  
3-1 Solving Systems of Equations ANSWER:  
(–26.01, –27.54) (15.03, 10.98)

Solve each system of equations.


61. 
62. 
SOLUTION:  
Write each equation in the form y = mx + b. SOLUTION:  
  Multiply the equation  by 4.
 

   
Add the equations to eliminate one variable.
Enter  as Y1 and   as   
Y2. Then graph the lines.
KEYSTROKES:  Y =   (    (−)  5   ·   8  
   +   1   8   )     ÷  (−)  6  ·  3   ENTER 
    (  4  ·  3       +    3   2   )   ÷  8   ·   8     
 ENTER  ZOOM  3  ENTER  ENTER Because 0 = 0 is true, the system has infinite
  solutions.
Find the intersection of the lines.
ANSWER:  
KEYSTROKES:   2nd   [CALC] 5  ENTER   
ENTER   ENTER Infinite solutions
 
63. 

SOLUTION:  
The coefficients of t-variables are 5 and 3 and their
least common multiple is 15, so multiply each
equation by the value that will make the t-coefficient
15.
   
The coordinates of the intersection to the nearest
hundredth are .

ANSWER:  
(15.03, 10.98)

Solve each system of equations.


62. 

SOLUTION:  
Multiply the equation  by 4.
 

 
Add the equations to eliminate one variable.
  Manual - Powered by Cognero
eSolutions Page 27
Because 0 = 0 is true, the system has infinite
 
solutions.
The solution is (–5, 4).
ANSWER:  
3-1 Solving Systems of Equations ANSWER:  
Infinite solutions
(–5, 4)

63.  64. 

SOLUTION:   SOLUTION:  
The coefficients of t-variables are 5 and 3 and their The coefficients of t-variables are 10 and 4 and their
least common multiple is 15, so multiply each least common multiple is 20, so multiply each
equation by the value that will make the t-coefficient equation by the value that will make the t-coefficient
15. 20.
   

   
Substitute –5 for r into either original equation and Substitute –3 for v into either original equation and
solve for t. solve for t.
   

   
The solution is (–5, 4). The solution is (2.5, –3).
ANSWER:   ANSWER:  
(–5, 4) (2.5, –3)

64.  65. 

SOLUTION:   SOLUTION:  
The coefficients of t-variables are 10 and 4 and their Rewrite the equations in the standard form of the
least common multiple is 20, so multiply each equation
equation by the value that will make the t-coefficient  
20.
 
 
The coefficients of x-variables are 4 and 6 and their
least common multiple is 12, so multiply each
equation by the value that will make the x-coefficient
 
12.
Substitute –3 for v into either original equation and
 
solve for t.
 

 
Because 0 = 0 is true, the system has infinite
solutions.
  ANSWER:  
The solution is (2.5, –3). infinite solutions
eSolutions Manual - Powered by Cognero Page 28

ANSWER:  
(2.5, –3)
  Because 0 = 0 is true, the system has infinite
The solution is (2.5, –3). solutions.

ANSWER:  
3-1 Solving Systems of Equations ANSWER:  
(2.5, –3) infinite solutions

65. 
66. 
SOLUTION:  
Rewrite the equations in the standard form of the
equation SOLUTION:  
 
Solve the equation  for z.

 
The coefficients of x-variables are 4 and 6 and their
least common multiple is 12, so multiply each Substitute  for z in the equation
equation by the value that will make the x-coefficient
12.  and solve for y.
 

 
Because 0 = 0 is true, the system has infinite  
solutions. Because 0 = 0 is true, the system has infinite
ANSWER:   solutions.
infinite solutions ANSWER:  
infinite solutions

66. 
67. 

SOLUTION:  
SOLUTION:  
Solve the equation  for z.
Multiply the equation  by  .
 
Substitute  for z in the equation

 and solve for y.

 
Add the equations to eliminate one variable.
 

 
Because 0 = 0 is true, the system has infinite
solutions.

ANSWER:  
infinite solutions
eSolutions Manual - Powered by Cognero Page 29
 
67.  Substitute 16 for a into either original equation and
Because 0 = 0 is true, the system has infinite (against the current) in 24 minutes. She can row the
solutions. same distance downstream in 13 minutes. Assume
that both the rowing speed and the speed of the
ANSWER:  
3-1 Solving Systems of Equations current are constant.
infinite solutions a. Find the speed at which Allison is rowing and the
speed of the current.
b. If Allison plans to meet her friends 3 miles
upstream one hour from now, will she be on time?
67.  Explain.
SOLUTION:  
SOLUTION:   a. Let x be the rowing speed.
Let y be the speed of the current.
Multiply the equation  by  .
Allison’s speed in upstream:
 
Allison’s speed in downstream:
 
The system of equations that represents this situation
is
 
Add the equations to eliminate one variable.
   
Add the equations to eliminate one variable.
 

 
 
Substitute 3.56 for x into either original equation and
Substitute 16 for a into either original equation and
solve for y.
solve for b.
 
 

 
The speed of the rowing boat is 3.56 mph and the
speed of the current is 1.06 mph.
 
b. No, Allison will be late.
  To row 3 miles upstream she needs 72 minutes.
The solution is (16, –8). So, 72 – 60 = 12 minutes.
She will be 12 minutes late.
ANSWER:  
(16, –8) ANSWER:  
a. 3.56 mph;1.06 mph
68. ROWING Allison can row a boat 1 mile upstream b. No; she will be 12 minutes late.
(against the current) in 24 minutes. She can row the
same distance downstream in 13 minutes. Assume 69. CCSS MODELING The table shows the winning
that both the rowing speed and the speed of the times in seconds for the 100-meter dash at the
current are constant. Olympics between 1964 and 2008.
a. Find the speed at which Allison is rowing and the  
speed of the current.
b. If Allison plans to meet her friends 3 miles
eSolutions Manualone
upstream - Powered by Cognero
hour from now, will she be on time? Page 30
Explain.
SOLUTION:  
b. No; she will be 12 minutes late.
ANSWER:  
69. CCSS MODELING The table shows the winning
times in Systems
seconds for a. Sample answer for men using (0, 10) and (44,
3-1 Solving of the 100-meter dash at the
Equations 9.69):
Olympics between 1964 and 2008.
   y m = –0.00705x + 10; sample answer for women
using (0, 11.4) and (44, 10.78):
yw = –0.01409x + 11.4
b.

 
a. Write equations that represent the winning times
for men and women since 1964.Assume that both
times continue along the same trend.
b. Graph both equations. Estimate when the
women’s performance will catch up to the men’s
performance. Do you think that your prediction is  
reasonable? Explain. Based on these data, the women’s performance will
  catch up to the men’s performance 198 years after
SOLUTION:   1964, or in the year 2162. The next Olympic year
would be 2164; this prediction is not reasonable. It
a. Sample answer for men using (0, 10) and (44,
9.69): is unlikely that women’s times will ever catch up to
 y m = –0.00705x + 10; sample answer for women men’s times because the times cannot continue to
increase and decrease infinitely.
using (0, 11.4) and (44, 10.78):
 
yw = –0.01409x + 11.4
b. 70. JOBS Levi has a job offer in which he will receive
$800 per month plus a commission of 2% of the total
price of cars he sells. At his current job, he receives
$1200 per month plus a commission of 1.5% of his
total sales. How much must he sell per month to
make the new job a better deal?
SOLUTION:  
Let x be the total price of cars Levi sells.
 

 
Based on these data, the women’s performance will
catch up to the men’s performance 198 years after
1964, or in the year 2162. The next Olympic year
would be 2164; this prediction is not reasonable. It  
is unlikely that women’s times will ever catch up to To make the new job a better deal, he has to sell at
men’s times because the times cannot continue to least $80,000 worth of cars.
increase and decrease infinitely.
ANSWER:  
ANSWER:   more than $80,000
a. Sample answer for men using (0, 10) and (44,
9.69): 71. TRAVEL A youth group went on a trip to an
 y m = –0.00705x + 10; sample answer for women amusement park, travelling in two vans. The number
using (0, 11.4) and (44, 10.78): of people in each van and the total cost of
eSolutions Manual - Powered by Cognero Page 31
admission are shown in the table. Find the adult
yw = –0.01409x + 11.4
price and student price of admission.
b.
 
To make the new job a better deal, he has to sell at students is $9.
least $80,000 worth of cars.
ANSWER:  
ANSWER:  
3-1 Solving Systems of Equations adult: $16; student: $9
more than $80,000
GEOMETRY Find the point at which the
71. TRAVEL A youth group went on a trip to an diagonals of the quadrilaterals intersect.
amusement park, travelling in two vans. The number
of people in each van and the total cost of
admission are shown in the table. Find the adult
price and student price of admission.
 

72. 
SOLUTION:  
The quadrilateral has the vertices A(2, 2), B(0, 8), C
SOLUTION:  
(10, 5), and D(8, 2).
Let x be the adult price of admission.  
Let y be the student price of admission.
The system of equations that represents the situation The equation of the lines
is  
 
  

 
Multiply the equation  by –1.  
  Substitute  for y in the equation

 and solve for x.
 
 
Add the equations to eliminate one variable.
 

 
Substitute 9 for y into either original equation and  
solve for x. Substitute 6 for x into either original equation and
  solve for y.
 

 
The price of admission for adults is $16 and for
students is $9.

ANSWER:  
adult: $16; student: $9

GEOMETRY Find the point at which the  


diagonals of the quadrilaterals intersect. The diagonals of the quadrilateral intersect at (6,
eSolutions Manual - Powered by Cognero 3.5). Page 32

ANSWER:  
 
Substitute  for x into either original equation and
3-1 Solving Systems of Equations
  solve for y.
The diagonals of the quadrilateral intersect at (6,  
3.5).

ANSWER:  
(6, 3.5)

 
The diagonals of the quadrilateral intersect at

73. 
ANSWER:  
SOLUTION:  
The quadrilateral has the vertices A(3, 4), B(2, 9), C
(11, 18), and D(6, 3).
  74. ELECTIONS In the election for student council,
The equation of the lines Candidate A received 55% of the total votes, while
  Candidate B received 1541 votes. If Candidate C
received 40% of the votes that Candidate A
received, how many total votes were cast?
SOLUTION:  
Let x be the total number of votes.
  The equation that represents the situation is
 
Substitute  for y in the equation
 
 and solve for x. Solve for x.
   

 
The total number of votes is 6700.

  ANSWER:  
6700 votes
Substitute  for x into either original equation and
75. MULTIPLE REPRESENTATIONS In this
solve for y.
problem, you will explore systems of equations with
 
three linear equations and two variables.

eSolutions Manual - Powered by Cognero


a. TABULAR Make a table of x and y-valuesPage
for
33

each equation.
b. ANALYTICAL Which values from the table
ANSWER:  
6700 votes

3-1 MULTIPLE
75. Solving REPRESENTATIONS
Systems of Equations In this
problem, you will explore systems of equations with
three linear equations and two variables.  
d. If all three lines intersect at the same point, then
the system has a solution. The system has no solution
if the lines intersect at 3 different points, or if two or
three lines are parallel.
a. TABULAR Make a table of x and y-values for
each equation. ANSWER:  
b. ANALYTICAL Which values from the table a.
indicate intersections? Is there a solution that
satisfies all three equations?
c. GRAPHICAL Graph the three equations on a
single coordinate plane.
d. VERBAL What conditions must be met for a
system of three equations with two variables to have
a solution? What conditions result in no solution?
SOLUTION:  
a. The table of x and y values for each equation:
 

 
b. Equations 1 and 2 intersect at (4, 4), equations 2
and 3 intersect at (2, 0), and equations 1 and 3
intersect at (1, 5); there is no solution that satisfies all
three equations.
c.

 
b. Equations 1 and 2 intersect at (4, 4), equations 2
and 3 intersect at (2, 0), and equations 1 and 3
intersect at (1, 5); there is no solution that satisfies all  
three equations. d. If all three lines intersect at the same point, then
c. The graph of three equations on a single the system has a solution. The system has no solution
coordinate plane is if the lines intersect at 3 different points, or if two or
  three lines are parallel.

76. CCSS CRITIQUE Gloria and Syreeta are solving


the system  and  Is
either of them correct? Explain your reasoning.

 
d. If all three lines intersect at the same point, then
the system
eSolutions Manual -has a solution.
Powered The
by Cognero system has no solution Page 34
if the lines intersect at 3 different points, or if two or
three lines are parallel.
  Sample answer: Gloria; Syreeta subtracted 26 from
d. If all three lines intersect at the same point, then 17 instead of 17 from 26 and got 3x = –9 instead of
the system has a solution. The system has no solution 3x =9. She proceeded to get a value of –11 for y.
if the lines
3-1 Solving intersect
Systems ofatEquations
3 different points, or if two or She would have found her error if she substituted the
three lines are parallel. solution into the original equations.

76. CCSS CRITIQUE Gloria and Syreeta are solving 77. CHALLENGE Find values of a and b for which the
the system  and  Is following system has a solution of (b – 1, b – 2).
either of them correct? Explain your reasoning.  

SOLUTION:  
Substitute b – 1 for x and b – 2 for y in the equation
 and solve for b.
 

ANSWER:  

 
78. REASONING If a is consistent and dependent with
SOLUTION:  
b, b is inconsistent with c, and c is consistent and
Sample answer: Gloria is correct. Syreeta subtracted independent with d, then a will sometimes, always, or
26 from 17 instead of 17 from 26 and got 3x = –9 never be consistent and independent with d. Explain
instead of 3x = 9. She proceeded to get a value of – your reasoning.
11 for y. She would have found her error if she
substituted the solution into the original equations. SOLUTION:  
Sample answer: Always; a and b are the same line.
ANSWER:   b is parallel to c, so a is also parallel to c. Since c and
Sample answer: Gloria; Syreeta subtracted 26 from d are consistent and independent, then c is not
17 instead of 17 from 26 and got 3x = –9 instead of parallel to d and, thus, intersects d. Since a and c are
3x =9. She proceeded to get a value of –11 for y. parallel, then a cannot be parallel to d, so, a must
She would have found her error if she substituted the intersect d and must be consistent and independent
solution into the original equations. with d.

77. CHALLENGE Find values of a and b for which the ANSWER:  


following system has a solution of (b – 1, b – 2). Sample answer: Always; a and b are the same line.
  b is parallel to c, so a is also parallel to c. Since c and
d are consistent and independent, then c is not
parallel to d and, thus, intersects d. Since a and c are
parallel, then a cannot be parallel to d, so, a must
SOLUTION:   intersect d and must be consistent and independent
Substitute b – 1 for x and b – 2 for y in the equation with d.
 and solve for b.
79. OPEN ENDED Write a system of equations in
 
which one equation needs to be multiplied by 3 and
the other needs to be multiplied by 4 in order to solve
the system with elimination. Then solve your system.
SOLUTION:  
Sample answer:
eSolutions Manual - Powered by Cognero The system of equations is Page 35
 
d are consistent and independent, then c is not
parallel to d and, thus, intersects d. Since a and c are ANSWER:  
parallel, then a cannot be parallel to d, so, a must Sample answer:
intersectSystems
3-1 Solving d and must be consistent and independent
of Equations 4x + 5y = 21 and 3x - 2y = 10;
with d. (4, 1)

79. OPEN ENDED Write a system of equations in 80. WRITING IN MATH Why is substitution


which one equation needs to be multiplied by 3 and sometimes more helpful than elimination, and vice
the other needs to be multiplied by 4 in order to solve versa?
the system with elimination. Then solve your system.
SOLUTION:  
SOLUTION:  
Sample answer: It is more helpful to use substitution
Sample answer: when one of the variables has a coefficient of 1 or if
The system of equations is
a coefficient can be reduced to 1 without turning
 
other coefficients into fractions. Otherwise,
elimination is more helpful because it will avoid the
use of fractions in solving the system.
 
ANSWER:  
Sample answer: It is more helpful to use substitution
  when one of the variables has a coefficient of 1 or if
a coefficient can be reduced to 1 without turning
other coefficients into fractions. Otherwise,
elimination is more helpful because it will avoid the
  use of fractions in solving the system.

81. SHORT RESPONSE Simplify


  SOLUTION:  

ANSWER:  
 
The solution of the system is (4, 1).
82. ACT/SAT Which of the following best describes the
ANSWER:  
graph of the equations?
Sample answer:  
4x + 5y = 21 and 3x - 2y = 10;
(4, 1)

80. WRITING IN MATH Why is substitution  


sometimes more helpful than elimination, and vice A The lines are parallel.
versa? B The lines are perpendicular.
C The lines have the same x-intercept.
SOLUTION:  
D The lines have the same y-intercept.
Sample answer: It is more helpful to use substitution E The lines are the same.
when one of the variables has a coefficient of 1 or if
a coefficient can be reduced to 1 without turning SOLUTION:  
other coefficients into fractions. Otherwise, Write each equation in slope-intercept form.
elimination is more helpful because it will avoid the  
use of fractions in solving the system.
ANSWER:  
Sample answer: It is more helpful to use substitution  
when one of the variables has a coefficient of 1 or if Also:
a coefficient
eSolutions can bebyreduced
Manual - Powered Cognero to 1 without turning   Page 36
other coefficients into fractions. Otherwise,
elimination is more helpful because it will avoid the
The slopes are equal. So the lines are parallel.
The correct choice is A.
ANSWER:  
3-1 Solving Systems of Equations ANSWER:  
A

82. ACT/SAT Which of the following best describes the 83. GEOMETRY Which set of dimensions corresponds
graph of the equations? to a triangle similar to the one shown at the right?
   
F 1 unit, 2 units, 3 units
G 7 units, 11 units, 12 units
  H 10 units, 23 units, 24 units
A The lines are parallel. J 20 units, 48 units, 52 units
B The lines are perpendicular.  
C The lines have the same x-intercept.
D The lines have the same y-intercept.
E The lines are the same.
SOLUTION:  
Write each equation in slope-intercept form.
 
SOLUTION:  
The dimensions of the given triangle and the
dimensions given in choice J have a common ratio.
So, the correct choice is J.
 
Also: ANSWER:  
  J

84. Move-A-Lot Rentals will rent a moving truck for


$100 plus $0.10 for every mile it is driven. Which
equation can be used to find C, the cost of renting a
moving truck and driving it for m miles?
   
The slopes are equal. So the lines are parallel. A C = 0.1(100 + m)
The correct choice is A. B C = 100 + 0.1m
ANSWER:   C C = 100m + 0.1
A D C = 100(m + 0.1)
SOLUTION:  
83. GEOMETRY Which set of dimensions corresponds
Let m be the number of miles the truck is driven.
to a triangle similar to the one shown at the right?
So:
 
C = 100 + 0.10m
F 1 unit, 2 units, 3 units
The correct choice is B.
G 7 units, 11 units, 12 units
H 10 units, 23 units, 24 units ANSWER:  
J 20 units, 48 units, 52 units B
 
85. CRAFTS Priscilla sells stuffed animals at a local
craft show. She charges $10 for the small ones and
$15 for the large ones. To cover her expenses, she
needs to sell at least $350.
a. Write an inequality for this situation.
b. Graph the inequality.
c. If she sells 10 small and 15 large animals, will she
SOLUTION:   cover her expenses?
The dimensions of the given triangle and the SOLUTION:  
eSolutions Manual - Powered
dimensions given inbychoice
Cognero
J have a common ratio. Page 37
a. Let s represent the small animals and l represent
So, the correct choice is J.
the large ones.
Priscilla needs to sell at least $350.
needs to sell at least $350. So, Priscilla will not cover her expenses if she sells
a. Write an inequality for this situation. 10 small and 15 large animals.
b. Graph the inequality. ANSWER:  
3-1 Solving
c. If she Systems of Equations
sells 10 small and 15 large animals, will she
a.
cover her expenses?
b.
SOLUTION:  
a. Let s represent the small animals and l represent
the large ones.
Priscilla needs to sell at least $350.
So:

 
b. The boundary is the graph of 10s + 15l = 350.
Since the inequality is , the boundary line will be
solid. c. no
Test the point (0, 0).
  Write an equation for each function.

 
Shade the region that does not include (0, 0).
  86. 
SOLUTION:  
2
The graph is a translation of the graph of y = x .
When a constant k is added to or subtracted from a
parent function, the result f (x) ± k is a translation of
the graph up or down.
Here, the graph of the parent function is moved 6
units up. So, 6 should be added to the parent function.
So, the equation of the graph is .
 
ANSWER:  
 c. Test the point (10, 15).
 

 
So, Priscilla will not cover her expenses if she sells
10 small and 15 large animals. 87. 

ANSWER:   SOLUTION:  
a. The graph is a transformation of the graph of the
b. parent function .
When a constant h is added to or subtracted from x
before evaluating a parent function, the result, f (x ±
h), is a translation left or right.
When a constant k is added to or subtracted from a
parent function, the result f (x) ± k is a translation of
the graph up or down.
A reflection flips a figure over a line called the line
eSolutions Manual - Powered by Cognero
of reflection. The reflection –f (x) reflects the graph
Page 38
of f (x) across the x-axis and the reflection f (–x)
reflects the graph of f (x) across the y-axis.
c. no The parent function is moved 3 units right and
reflected across the x-axis.
So, the equation of the graph is .
So, the equation of the graph is .
ANSWER:  
3-1 Solving Systems of Equations ANSWER:  

87.  88. 
SOLUTION:   SOLUTION:  
The graph is a transformation of the graph of the The graph is a transformation of the graph of the
parent function . 2
parent function y = x .
When a constant h is added to or subtracted from x When a constant h is added to or subtracted from x
before evaluating a parent function, the result, f (x ± before evaluating a parent function, the result, f (x ±
h), is a translation left or right. h), is a translation left or right.
When a constant k is added to or subtracted from a When a constant k is added to or subtracted from a
parent function, the result f (x) ± k is a translation of parent function, the result f (x) ± k is a translation of
the graph up or down. the graph up or down.
A reflection flips a figure over a line called the line A reflection flips a figure over a line called line of
of reflection. The reflection –f (x) reflects the graph reflection. The reflection–f (x) reflects the graph of f
of f (x) across the x-axis and the reflection f (–x) (x) across the x-axis and the reflection f (–x) reflects
reflects the graph of f (x) across the y-axis. the graph of f (x) across the y-axis.
The parent function is moved 3 units right and Here, the graph of the parent function is moved 4
reflected across the x-axis. units right and reflected across the x-axis.
So, the equation of the graph is . So, the equation of the graph is .

ANSWER:   ANSWER:  

Solve each equation. Check your solution.


89. 2p = 14
SOLUTION:  

88. 
 
SOLUTION:   Substitute p = 7 in the original equation and check.
The graph is a transformation of the graph of the  
2
parent function y = x .
When a constant h is added to or subtracted from x
before evaluating a parent function, the result, f (x ±
h), is a translation left or right.  
When a constant k is added to or subtracted from a So, the solution is p = 7.
parent function, the result f (x) ± k is a translation of ANSWER:  
the graph up or down.
A reflection flips a figure over a line called line of 7
reflection. The reflection–f (x) reflects the graph of f
(x) across the x-axis and the reflection f (–x) reflects 90. –14 + n =  –6
the graph of f (x) across the y-axis.
Here, the graph of the parent function is moved 4 SOLUTION:  
units right and reflected across the x-axis.
So, the equation of the graph is .
eSolutions Manual - Powered by Cognero   Page 39

ANSWER:   Substitute n = 8 in the original equation and check.


 
 
 
So, the solution is p = 7.
So, the solution is a = 3.2.
ANSWER:  
3-1 Solving Systems of Equations ANSWER:  
7
3.2

90. –14 + n =  –6 92. x + 9x – 6x + 4x = 20


SOLUTION:   SOLUTION:  

 
Substitute n = 8 in the original equation and check.
 
 
Substitute x = 2.5 in the original equation and check.
 
 
So, the solution is n = 8.

ANSWER:  
8
 
91. 7a – 3a + 2a – a = 16 So, the solution is x = 2.5.
SOLUTION:   ANSWER:  
2.5

93. 27 = –9(y + 5) + 6(y + 8)


SOLUTION:  

 
Substitute a = 3.2 in the original equation and check.
 

 
Substitute y = –8 in the original equation and check.
 
 
So, the solution is a = 3.2.

ANSWER:  
3.2  
So, the solution is y = –8.
92. x + 9x – 6x + 4x = 20
ANSWER:  
SOLUTION:  
–8

94. –7(p + 7) + 3(p – 4) = –17


SOLUTION:  

 
Substitute x = 2.5 in the original equation and check.
 
eSolutions Manual - Powered by Cognero Page 40

 
 
So, the solution is y = –8.
ANSWER:  
ANSWER:  
3-1 Solving Systems of Equations yes
–8

94. –7(p + 7) + 3(p – 4) = –17 96. 


SOLUTION:   SOLUTION:  
Substitute 1 for x and 1 for y in the inequality.
 

   
Substitute p = –11 in the original equation and check. The point (1, 1) satisfies the inequality
 
ANSWER:  
yes

97. 

SOLUTION:  
Substitute 0 for x and 0 for y in the inequality.
   
So, the solution is p = –11.

ANSWER:  
–11
The point (0, 0) does not satisfy the inequality
Determine whether the given point satisfies
each inequality.
95.  ANSWER:  
no
SOLUTION:  
Substitute 2 for x and –2 for y in the inequality.
 

 
The point (2, –2) satisfies the inequality

ANSWER:  
yes

96. 

SOLUTION:  
Substitute 1 for x and 1 for y in the inequality.
 

eSolutions Manual - Powered by Cognero Page 41

 
3-2 Solving Systems of Inequalities by Graphing

Solve each system of inequalities by graphing.


3. 
1. 
SOLUTION:  
SOLUTION:   The graph of the system of inequalities is
The graph of the system of inequalities is

ANSWER:   ANSWER:  

4. CCSS REASONING The most Kala can spend on


2.  hot dogs and buns for her cookout is $35. A package
of 10 hot dogs costs $3.50. A package of buns costs
SOLUTION:   $2.50 and contains 8 buns. She needs to buy at least
40 hot dogs and 40 buns.
The graph of the system of inequalities is
a. Graph the region that shows how many packages
of each item she can purchase.
b. Give an example of three different purchases she
can make.
SOLUTION:  
a. Let h be the number of package of hotdogs.
Let b be the number of package of buns.
The inequalities that represents the situation are
ANSWER:    

 
The graph of the region that shows the number of
packages of each item Kala can purchase is
 

3. 

SOLUTION:  
eSolutions Manual - Powered by Cognero Page 1
The graph of the system of inequalities is
5. 

 
The graph of the region that showsbytheGraphing
number of SOLUTION:  
3-2 Solving Systems of Inequalities
packages of each item Kala can purchase is The graph of the system of inequalities is
   

.
   
b. Sample answer: 4 packages of hotdogs, 5 The coordinate (–4, 8) can be determined from the
packages of buns; 5 packages of hotdogs, 6 graph.
packages of buns; 6 packages of hotdogs, 5 Solve the system of equations
packages of buns. Substitute 8 for y in the equation  and solve
for x.
ANSWER:  
 
a.

 
So, the coordinate is (3.5, 8).
Solve the system of equations

Substitute 2x + 1 for y in the equation


 and solve for x.

b. Sample answer: 4 packages of hotdogs, 5


packages of buns; 5 packages of hotdogs, 6
packages of buns; 6 packages of hotdogs, 5
packages of buns

Find the coordinates of the vertices of the


triangle formed by each system of inequalities. Substitute 0.5 for x in the equation  and 
find y.
5. 
 
SOLUTION:  
So, the coordinate is (0.5, 2).
The graph of the system of inequalities is
The vertices of the triangle are at (3.5, 8), (–4, 8),
  and (0.5, 2).

ANSWER:  

eSolutions Manual - Powered by Cognero . Page 2


 
The coordinate (–4, 8) can be determined from the
So, the coordinate is (0.5, 2).
The vertices of the triangle are at (3.5, 8), (–4, 8),  
and (0.5,Systems
3-2 Solving 2). of Inequalities by Graphing
 
ANSWER:   So, the coordinate is (3, 5).
The vertices of the triangle are at (–4, 4), (2, –8) and
(3, 5).

ANSWER:  

(3.5, 8), (–4, 8), (0.5, 2)

6.  (2, –8), (3, 5), (–4, 4)

Solve each system of inequalities by graphing.


SOLUTION:  
The graph of the system of inequalities is 7. 
 
SOLUTION:  
Graph the system of inequalities in a coordinate
plane.

.
  .
The coordinate (–4, 4) and (2, –8) can be determined
from the graph. ANSWER:  
Solve the system of equations

 
Substitute 13x – 34 for y in the equation
 and solve for x.

8. 
Substitute 3 for x in the equation  and 
find y.
SOLUTION:  
Graph the system of inequalities in a coordinate
  plane.

 
So, the coordinate is (3, 5).
The vertices of the triangle are at (–4, 4), (2, –8) and
eSolutions Manual - Powered by Cognero Page 3
(3, 5).

ANSWER:  
3-2 Solving Systems of Inequalities by Graphing

8.  10. 

SOLUTION:   SOLUTION:  
Graph the system of inequalities in a coordinate Graph the system of inequalities in a coordinate
plane. plane.

. .

ANSWER:   ANSWER:  

9.  11. 

SOLUTION:   SOLUTION:  
Graph the system of inequalities in a coordinate Graph the system of inequalities in a coordinate
plane. plane.

. .

ANSWER:   ANSWER:  

12. 
10. 
eSolutions Manual - Powered by Cognero Page 4
SOLUTION:  
SOLUTION:  
Graph the system of inequalities in a coordinate
Graph the system of inequalities in a coordinate
3-2 Solving Systems of Inequalities by Graphing

12. 
14. 

SOLUTION:  
Graph the system of inequalities in a coordinate SOLUTION:  
plane. Graph the system of inequalities in a coordinate
plane.

.
.
ANSWER:  
ANSWER:  

13. 
15. 
SOLUTION:  
The graph of the system of inequalities is
SOLUTION:  
Graph the system of inequalities in a coordinate
plane.

ANSWER:  

ANSWER:  

14. 
eSolutions Manual - Powered by Cognero 16. RECORDING Jane’s band wants to spend no more Page 5

SOLUTION:   than $575 recording their first CD. The studio


Graph the system of inequalities in a coordinate charges at least $35 an hour to record. Graph a
3-2 Solving Systems of Inequalities by Graphing

16. RECORDING Jane’s band wants to spend no more 17. SUMMER TRIP Rondell has to save at least $925
than $575 recording their first CD. The studio to go to Rome with his Latin class in 8 weeks. He
charges at least $35 an hour to record. Graph a earns $9 an hour working at the Pizza Palace and
system of inequalities to represent this situation. $12 an hour working at a car wash. By law, he
cannot work more than 25 hours per week. Graph
SOLUTION:   two inequalities that Rondell can use to determine the
Let y be the cost to record the CD. number of hours he needs to work at each job if he
Jane’s band wants to spend no more than $575. wants to make the trip.
So:
SOLUTION:  
Let x represent the time taken to record CD in hours. Let x be the number of hours that Rondell works at
The studio charges at least $35 an hour. the Pizza Palace and y be the number of hours he
So: works at the car wash.
. So:
Graph the system of inequalities.
 
 
Also:

Graph the system of inequalities and find the solution.


 

ANSWER:  

ANSWER:  

Find the coordinates of the vertices of the


triangle formed by each system of inequalities.

18. 

SOLUTION:  
Graph the system of inequalities.

17. SUMMER TRIP Rondell has to save at least $925


to go to Rome with his Latin class in 8 weeks. He
eSolutions Manual - Powered by Cognero Page 6
earns $9 an hour working at the Pizza Palace and
$12 an hour working at a car wash. By law, he
cannot work more than 25 hours per week. Graph
3-2 Solving Systems of Inequalities by Graphing (0, 2), (4, 0), (0, 0)

Find the coordinates of the vertices of the


triangle formed by each system of inequalities. 19. 

18. 
SOLUTION:  
Graph the system of inequalities.
SOLUTION:  
Graph the system of inequalities.

The coordinates of the vertices are (2, –1),


(5, 8), and (–7, 8).
The coordinates of the vertices are (0, 2),
(4, 0), and (0, 0). ANSWER:  

ANSWER:  

(2, –1), (5, 8), (–7, 8)

(0, 2), (4, 0), (0, 0)


20. 

19.  SOLUTION:  
Graph the system of inequalities.
SOLUTION:  
Graph the system of inequalities.

The coordinates of the vertices are (1, 9),


(4, 0), and (4, 9).
The coordinates of the vertices are (2, –1), ANSWER:  
(5, 8), and (–7, 8).

ANSWER:  

eSolutions Manual - Powered by Cognero Page 7

(1, 9), (4, 0), (4, 9)


3-2 Solving Systems of Inequalities by Graphing
(2, –1), (5, 8), (–7, 8) (1, 9), (4, 0), (4, 9)

20.  21. 

SOLUTION:   SOLUTION:  
Graph the system of inequalities. Graph the system of inequalities.

The coordinates of the vertices are (1, 9), The coordinate of the vertices are (–3, 1.5),
(4, 0), and (4, 9). (0, –6), and (5, 7.5).

ANSWER:   ANSWER:  

(1, 9), (4, 0), (4, 9) (–3, 1.5), (5, 7.5), (0, –6)

21.  22. 

 
SOLUTION:  
 
Graph the system of inequalities.
SOLUTION:  
Graph the system of inequalities.

The coordinate of the vertices are (–3, 1.5),


(0, –6), and (5, 7.5).
The coordinates of the vertices are (–6, –5),
ANSWER:   (–2, 4.5), and (7.5, –2).

ANSWER:  

eSolutions Manual - Powered by Cognero Page 8

(–3, 1.5), (5, 7.5), (0, –6)


3-2 Solving Systems of Inequalities by Graphing
(–3, 1.5), (5, 7.5), (0, –6) (–6, –5), (–2, 4.5), (7.5, –2)

22.  23. 

 
SOLUTION:  
 
Graph the system of inequalities.
SOLUTION:  
Graph the system of inequalities.

The coordinates of the vertices are (–2, 3),


(6, –4), and (8, 4).
The coordinates of the vertices are (–6, –5),
ANSWER:  
(–2, 4.5), and (7.5, –2).

ANSWER:  

(8, 4), (6, –4), (–2, 3)

(–6, –5), (–2, 4.5), (7.5, –2) 24. BAKING Rebecca wants to bake cookies and
cupcakes for a bake sale. She can bake 15 cookies
at a time and 12 cupcakes at a time. She needs to
make at least 120 baked goods, but no more than
23. 
360, and she wants to have at least three times as
many cookies as cupcakes. What combination of
batches of each could Rebecca make?
SOLUTION:  
Graph the system of inequalities. SOLUTION:  
Let x be the number of cookies and y be the number
of cupcakes.
So:

And:

Graph the inequalities.


The coordinates of the vertices are (–2, 3),  
(6, –4), and (8, 4).

ANSWER:  

eSolutions Manual - Powered by Cognero Page 9


make 72 cupcakes takes  batches.

ANSWER:  
3-2 Solving Systems of Inequalities by Graphing Sample answer: 15 batches of cookies and 6 batches
(8, 4), (6, –4), (–2, 3) of cupcakes.

24. BAKING Rebecca wants to bake cookies and 25. CELL PHONES Dale has a maximum of 800
cupcakes for a bake sale. She can bake 15 cookies minutes on his cell phone plan that he can use each
at a time and 12 cupcakes at a time. She needs to month. Daytime minutes cost $0.15, and nighttime
make at least 120 baked goods, but no more than minutes cost $0.10. Dale plans to use at least twice
360, and she wants to have at least three times as as many daytime minutes as nighttime minutes. If
many cookies as cupcakes. What combination of Dale uses at least 200 nighttime minutes and does not
batches of each could Rebecca make? go over his limit, what is his maximum bill? his
minimum bill?
SOLUTION:  
Let x be the number of cookies and y be the number SOLUTION:  
of cupcakes. Let x be the number of daytime minutes and y be the
So: number of nighttime minutes.

And:

Graph the inequalities. The optimize function is .


 
Graph the inequalities in the same coordinate plane.
 

 
The dark shaded region is the region where all the
inequalities are true.
Pick a point in this region.  Since we need to have   
whole numbers of batches, choose an x-coordinate The vertices of the shaded region are
which is a multiple of 15 and a y-coordinate which is .
a multiple of 12.
Sample answer: (225, 72).  
To make 225 cookies takes  batches, and to

make 72 cupcakes takes  batches.

ANSWER:  
 
Sample answer: 15 batches of cookies and 6 batches The maximum bill is $110 and the minimum bill is
of cupcakes. $80.
25. CELL PHONES Dale has a maximum of 800 ANSWER:  
minutes on his cell phone plan that he can use each maximum = $110, minimum = $80
month. Daytime minutes cost $0.15, and nighttime
minutes cost $0.10. Dale plans to use at least twice 26. TREES Trees are divided into four categories
as many daytime minutes as nighttime minutes. If according to height and trunk circumference. In one
Dale uses at least 200 nighttime minutes and does not forest, the trees are categorized by the heights and
go over
eSolutions his-limit,
Manual what
Powered is his maximum bill? his
by Cognero
circumferences described in the table. Page 10
minimum bill?  
SOLUTION:  
The maximum bill is $110 and the minimum bill is and 0.5 gallon of water per day. Jessica’s equipment
$80. weighs 10 pounds, and Marc’s equipment weighs 20
pounds.
ANSWER:  
3-2 Solving Systems of Inequalities by Graphing A gallon of water weighs approximately 8 pounds.
maximum = $110, minimum = $80 Each of them carries their own supplies, and Jessica
is capable of carrying 35 pounds while Marc can
26. TREES Trees are divided into four categories carry 50 pounds.
according to height and trunk circumference. In one a. Graph the inequalities that represent how much
forest, the trees are categorized by the heights and they can carry.
circumferences described in the table. b. How many days can they camp, assuming that
  they bring all their supplies in at once?
c. Who will run out of supplies first?
SOLUTION:  
  a. A gallon of water weighs approximately 8 pounds,
a. Write and graph the system of inequalities that so 0.5 gallon of water weighs about 4 pounds. Let x
represents the range of heights h and circumferences be the number of days and y be the number of
c for a co-dominant tree. pounds.
b. Determine the crown class of a basswood that is The system of inequalities that represent the situation
48 feet tall. Find the expected trunk circumference is:
 
SOLUTION:  
a. The height of co-dominant tree ranges from 56 to
72.
So:

The trunk circumference of the co-dominant tree  


ranges from 48 to 60. Graph the system of inequalities in the same
Therefore: coordinate plane.
.  
b. 48 lies between 40 and 55. So, the class is
intermediate and the expected trunk circumference is
34 – 48 in.

ANSWER:  
a.

 
b & c. Assume Jessica only uses the supplies she
has carried, and Marc only uses the supplies he has
b. intermediate; 34–48 in. carried. Then Marc will run out of supplies at the
point where y = 9x + 20 intersects the line y = 50.  
27. CCSS REASONING On a camping trip, Jessica
needs at least 3 pounds of food and 0.5 gallon of This point is , so Marc will run out of
water per day. Marc needs at least 5 pounds of food
and 0.5 gallon of water per day. Jessica’s equipment supplies after  days.  Jessica will run out of 
weighs 10 pounds, and Marc’s equipment weighs 20
pounds. supplies at the point where y = 7x + 10 intersects the
A gallon of water weighs approximately 8 pounds. line y = 35. This point is , so Jessica will
Each of them carries their own supplies, and Jessica run out of supplies after 3.57 days.  Therefore, Marc 
is capable of carrying 35 pounds while Marc can will run out of supplies first; Jessica can last about a
carry 50 pounds. quarter of a day longer.
a. Graph
eSolutions Manualthe
- Powered by Cognero
inequalities that represent how much Page 11
they can carry. ANSWER:  
b. How many days can they camp, assuming that a.
supplies after  days.  Jessica will run out of 
supplies at the point where y = 7x + 10 intersects the
line y = 35.
3-2 Solving This point
Systems is
of Inequalities , soGraphing
by Jessica will
run out of supplies after 3.57 days.  Therefore, Marc 
will run out of supplies first; Jessica can last about a
quarter of a day longer.
29. 
ANSWER:  
a.
SOLUTION:  
Graph the system of inequalities.

ANSWER:  

b. days
c. Marc; Jessica could last about a quarter of a day
longer than Marc.

Solve each system of inequalities by graphing.

28. 
30. 
SOLUTION:  
Graph the system of inequalities. SOLUTION:  
Graph the system of inequalities.

ANSWER:  
ANSWER:  

29. 
31. 
SOLUTION:  
eSolutions Manual - Powered by Cognero Page 12
Graph the system of inequalities. SOLUTION:  
Graph the system of inequalities.
3-2 Solving Systems of Inequalities by Graphing

31.  33. 

SOLUTION:   SOLUTION:  
Graph the system of inequalities. Graph the system of inequalities.

ANSWER:  
ANSWER:  

32. 

34. 
SOLUTION:  
Graph the system of inequalities. SOLUTION:  
Graph the system of inequalities.

ANSWER:  

ANSWER:  

33. 
eSolutions Manual - Powered by Cognero Page 13

SOLUTION:   35. 
3-2 Solving Systems of Inequalities by Graphing

34.  35. 

SOLUTION:   SOLUTION:  
Graph the system of inequalities. Graph the system of inequalities.

ANSWER:   ANSWER:  

35. 
36. 

SOLUTION:  
SOLUTION:  
Graph the system of inequalities.
Graph the system of inequalities.

ANSWER:   Since the inequalities have no shaded region in


common, the system has no solution.

ANSWER:  

eSolutions Manual - Powered by Cognero Page 14

36. 
3-2 Solving Systems of Inequalities by Graphing

37. MUSIC Steve is trying to decide what to put on his


36.  MP3 player. Audio books are 3 hours long and songs
are 2.5 minutes long. Steve wants no more than 4
SOLUTION:   audio books on his MP3 player, but at least ten songs
and one audio book. Each book costs $15.00 and
Graph the system of inequalities.
each song costs $0.95. Steve has $63 to spend on
books and music. Graph the inequalities to show
possible combinations of books and songs that
Steve can have.
SOLUTION:  
Let x represent the number of audio books and y
represent the number of songs that Steve puts on his
MP3 player.
Steve wants at least one audio book but no more
Since the inequalities have no shaded region in than 10, so 1 ≤ x ≤ 4.
common, the system has no solution. He also wants at least 10 songs, so y  ≥ 10.
Since he only has $63 to spend, 15x + 0.95y  ≤ 63.
ANSWER:  
 
Graph the following system of inequalities and the
shaded region will represent the possible
combinations of audio books and songs Steve can
purchase.
1 ≤ x ≤ 4,  y  ≥ 10, and 15x + 0.95y  ≤ 63
 

37. MUSIC Steve is trying to decide what to put on his


MP3 player. Audio books are 3 hours long and songs
are 2.5 minutes long. Steve wants no more than 4
audio books on his MP3 player, but at least ten songs
and one audio book. Each book costs $15.00 and
each song costs $0.95. Steve has $63 to spend on
books and music. Graph the inequalities to show
possible combinations of books and songs that
Steve can have.
ANSWER:  
SOLUTION:  
Let x represent the number of audio books and y
represent the number of songs that Steve puts on his
MP3 player.
Steve wants at least one audio book but no more
than 10, so 1 ≤ x ≤ 4.
He also wants at least 10 songs, so y  ≥ 10.
Since he only has $63 to spend, 15x + 0.95y  ≤ 63.
 
Graph the following system of inequalities and the
shaded region will represent the possible
combinations of audio books and songs Steve can
purchase.
1 ≤ x ≤ 4,  y
eSolutions  ≥ 10, and 15x
Manual - Powered by Cognero + 0.95y  ≤ 63 38. JOBS Louie has two jobs and can work no more Page 15

  than 25 total hours per week. He wants to earn at


least $150 per week. Graph the inequalities to show
3-2 Solving Systems of Inequalities by Graphing

38. JOBS Louie has two jobs and can work no more 39. TIME MANAGEMENT Ramir uses his spare
than 25 total hours per week. He wants to earn at time to write a novel and to exercise. He has
least $150 per week. Graph the inequalities to show budgeted 35 hours per week. He wants to exercise
possible combinations of hours worked at each job at least 7 hours a week but no more than 15. He also
that will help him reach his goal. hopes to write between 20 and 25 hours per week.
  Write and graph a system of inequalities that
represents this situation.
SOLUTION:  
Let w = the number of hours writing, and let e = the
number of hours exercising.
SOLUTION:  
So:
Let x represent the number of hours that Louie
works as a Busboy and y represent the number of
hours he works as a clerk.
So:
 and 
 
  Graph the inequalities.
Graph the system of inequalities.  
 

ANSWER:  
ANSWER:   Let w = the number of hours writing, and let e = the
number of hours exercising.

39. TIME MANAGEMENT Ramir uses his spare


time to write a novel and to exercise. He has
budgeted 35 hours per week. He wants to exercise
at least 7 hours a week but no more than 15. He also
hopes to write between 20 and 25 hours per week. Find the coordinates of the vertices of the
Write and graph a system of inequalities that figure formed by each system of inequalities.
represents this situation.
eSolutions Manual - Powered by Cognero Page 16
SOLUTION:   40. 
Let w = the number of hours writing, and let e = the
ANSWER:  

3-2 Solving Systems of Inequalities by Graphing

Find the coordinates of the vertices of the


figure formed by each system of inequalities.
41. 

40. 
SOLUTION:  
Graph the inequalities.
SOLUTION:    
Graph the inequalities.
 

 
The coordinates of the vertices are (–6, –2),
 
The coordinates of the vertices are (0, 2), , , and (0.8, –8.8).

, , and (2.8, –6.4)  


ANSWER:  
ANSWER:  

42. 
41. 

SOLUTION:  
SOLUTION:   Graph the inequalities.
 
Graph the inequalities.
 

 
The coordinates of the vertices are (–4, 6),
 
eSolutions Manual - Powered by Cognero Page 17
The coordinates of the vertices are (–6, –2), (–3, 8), (4.8, –7.6), and .
, , and (0.8, –8.8).  
   

ANSWER:   ANSWER:  
3-2 Solving Systems of Inequalities by Graphing

43. FINANCIAL LITERACY Mr. Hoffman is


investing $10,000 in two funds. One fund will pay 6%
42.  interest, and a riskier second fund will pay 10%
interest. What is the least amount Mr. Hoffman can
invest in the risky fund and still earn at least $740
after one year?
SOLUTION:  
Graph the inequalities. SOLUTION:  
  Let x be the least amount Mr. Hoffman invests in the
risky fund.
The remaining amount, 10,000 – x would be invested
in the first fund.
So:

 
So, a minimum of $3500 should be invested in the
risky fund to earn at least $740.
 
The coordinates of the vertices are (–4, 6), ANSWER:  
$3500
(–3, 8), (4.8, –7.6), and .
44. DODGEBALL A high school is selecting a
  dodgeball team to play in a fund-raising exhibition
against their rival. There can be between 10 and 15
ANSWER:   players on the team and there must be more girls
than boys on the team.
a. Write and graph a system of inequalities to
represent the situation.
43. FINANCIAL LITERACY Mr. Hoffman is b. List all of the possible combinations of boys and
investing $10,000 in two funds. One fund will pay 6% girls for the team.
interest, and a riskier second fund will pay 10% c. Explain why there is not an infinite number of
interest. What is the least amount Mr. Hoffman can possibilities.
invest in the risky fund and still earn at least $740
after one year? SOLUTION:  
a. Let g be the number of girls and b be the number
SOLUTION:   of boys in the team.
Let x be the least amount Mr. Hoffman invests in the
risky fund. Graph the inequalities in the same coordinate plane.
The remaining amount, 10,000 – x would be invested  
in the first fund.
So:

 
So, a minimum of $3500 should be invested in the
risky fund to earn at least $740.

ANSWER:  
$3500
eSolutions Manual - Powered by Cognero
  Page 18
b. (6, 4), (6, 5), (7, 3), (7, 4), (7, 5), (7, 6), (8, 2), (8,
44. DODGEBALL A high school is selecting a 3), (8, 4), (8, 5), (8, 6), (8, 7), (9, 1), (9, 2), (9, 3),
dodgeball team to play in a fund-raising exhibition (9, 4), (9, 5), (9, 6), (10, 0), (10, 1), (10, 2), (10, 3),
3-2 Solving Systems of Inequalities by Graphing
SOLUTION:  
Graph the inequalities in the same coordinate plane.
 
b. (6, 4), (6, 5), (7, 3), (7, 4), (7, 5), (7, 6), (8, 2), (8,
3), (8, 4), (8, 5), (8, 6), (8, 7), (9, 1), (9, 2), (9, 3),
(9, 4), (9, 5), (9, 6), (10, 0), (10, 1), (10, 2), (10, 3),
(10, 4), (10, 5), (11, 0), (11, 1), (11, 2), (11, 3), (11,
4), (12, 0),  (12, 1), (12, 2), (12, 3), (13, 0), (13, 1),
(13, 2), (14, 0), (14, 1), (15, 0)
c. Sample answer: You cannot have a fraction of a
person.

ANSWER:  
a.

The area defined by the inequalities is the region


quadrilateral ABCD.
The coordinates A(2, 6), B(5, 0), C(-3, -4), and D(-6,
8) can be determined from the graph. To find the
area, divide the region into ,  and 
trapezoid AECF by drawing lines  and  .

b. (6, 4), (6, 5), (7, 3), (7, 4), (7, 5), (7, 6), (8, 2), (8,
3), (8, 4), (8, 5), (8, 6), (8, 7), (9, 1), (9, 2), (9, 3),
(9, 4), (9, 5), (9, 6), (10, 0), (10, 1), (10, 2), (10, 3),
(10, 4), (10, 5), (11, 0), (11, 1), (11, 2), (11, 3), (11,
4), (12, 0),  (12, 1), (12, 2), (12, 3), (13, 0), (13, 1),
(13, 2), (14, 0), (14, 1), (15, 0)
c. Sample answer: You cannot have a fraction of a
person.

45. CHALLENGE Find the area of the region defined


by the following inequalities.

To find the area of each shape use the appropriate


area formula.

SOLUTION:  
Graph the inequalities in the same coordinate plane. Find the height by using the length of where K is
(2, 0).  This height is 3 units long. The corresponding 
base is  or 7.5.
 

eSolutions Manual - Powered by Cognero Page 19

The area defined by the inequalities is the region  


Therefore, the area of the enclosed figure is 75
square units.

3-2 Solving Systems of Inequalities by Graphing ANSWER:  


  75 square units

46. OPEN ENDED Write a system of two inequalities


in which the solution:
  a. lies only in the third quadrant.
Find the height by using the length of where G is b. does not exist.
(-3, 8).  This height is 3 units long. The corresponding c. lies only on a line.
d. lies on exactly one point.
base is  or 11.25.
  SOLUTION:  
a. Sample answer:
b. Sample answer:
y > 2, y < –2
c. Sample answer:

 
d. Sample answer:
Trapezoid AECF

The solution is (0, 0).


Let ,  and  . ANSWER:  
a. Sample answer:
b. Sample answer:
c. Sample answer:
d. Sample answer:  solution at (0, 0)
 
47. CHALLENGE Write a system of inequalities to
Add the areas of the three figures together to find
represent the solution shown. How many points with
the area of the enclosed figure.
integer coordinates are solutions of the system?
 
Therefore, the area of the enclosed figure is 75
square units.

ANSWER:  
75 square units

46. OPEN ENDED Write a system of two inequalities


in which the solution:
a. lies only in the third quadrant.
b. does not exist. SOLUTION:  
c. lies only on a line. Sample answer:
d. lies on exactly one point. First determine the related equations. To do this,
analyze the graph to find the slope and one point for
SOLUTION:   each line. 
a. Sample answer: The uppermost boundary has a slope of or –0.5.
b. Sample answer: This line has a y-intercept of 4 so the equation is y =
y > 2, y < –2 –0.5x + 4. Since the line is solid and the shading is
c. Sample answer: below the line, the inequality is .
The boundary to the left of the origin has a slope of
d. Sample answer: or -3. The y-intercept is -6 so the equation is y =
–3x – 6. Since the line is solid and the shading is
The solution is (0, 0). above the line, the inequality is .
eSolutions Manual - Powered by Cognero The boundary to the right of the origin has a slope of20
Page
ANSWER:   or 2. This line has a y-intercept of -6 so the
a. Sample answer: equation is y = 2x – 6. Since the line is solid and the
below the line, the inequality is . ANSWER:  
The boundary to the left of the origin has a slope of
or -3. The y-intercept is -6 so the equation is y = Sample answer: ; 47
3-2 Solving Systems of Inequalities by Graphing
–3x – 6. Since the line is solid and the shading is
above the line, the inequality is .
The boundary to the right of the origin has a slope of 48. CCSS ARGUMENTS Determine whether the
or 2. This line has a y-intercept of -6 so the following statement is true or false . If false, give a
equation is y = 2x – 6. Since the line is solid and the counterexample.
shading is above the line, the inequality is A system of two linear inequalities has either no
points or infinitely many points in its solution.
.
  SOLUTION:  
To determine how many points with integer True
coordinates are solutions to the system, analyze the
ANSWER:  
graph. Redraw the graph with each axis scaled by
1. Count every integer ordered pair on the true
boundary lines as well as all of the integer points 49. WRITING IN MATH Write a how-to manual for
within the area.   determining where to shade when graphing a system
  of inequalities.
SOLUTION:  
Sample answer: Shade each inequality in its standard
way, by shading above the line if y > and shading
below the line if y < (or you can use test points).
Once you determine where to shade for each
inequality, the area where every inequality needs to
be shaded is the actual solution. This is only the
shaded area.

ANSWER:  
Sample answer: Shade each inequality in its standard
  way, by shading above the line if y > and shading
First count all of the ordered pairs that lie on the below the line if y < (or you can use test points).
boundary lines. There are 12 points. Next count the Once you determine where to shade for each
ordered pairs on the axes to find that there are 12. inequality, the area where every inequality needs to
There are 7 points in the first quadrant, 10 in be shaded is the actual solution. This is only the
Quadrant II, 2 in Quadrant III, and 4 in Quadrant IV. shaded area.
There are a total of 47 points with integer
coordinates that are solutions to the system. 50. WRITING IN MATH Explain how you would test
  to see whether (–4, 6) is a solution of a system of
  inequalities.

ANSWER:   SOLUTION:  
Sample answer: Determine whether the point falls in
the shaded area of the graphs and/or determine
Sample answer: ; 47 whether the values satisfy each inequality.

ANSWER:  
48. CCSS ARGUMENTS Determine whether the Sample answer: Determine whether the point falls in
following statement is true or false . If false, give a the shaded area of the graphs and/or determine
counterexample. whether the values satisfy each inequality.
A system of two linear inequalities has either no
points or infinitely many points in its solution. 51. To be a member of the marching band, a student
must have a grade-point average of at least 2.0 and
SOLUTION:   must have attended at least five after-school
True practices. Choose the system of inequalities that best
represents this situation.
ANSWER:    
true Manual - Powered by Cognero
eSolutions Page 21
A  
49. WRITING IN MATH Write a how-to manual for
determining where to shade when graphing a system
Each output is two more than three times the input.
ANSWER:   The correct choice is G.
Sample answer: Determine whether the point falls in
the shaded
3-2 Solving area ofofthe
Systems graphs and/or
Inequalities by determine
Graphing ANSWER:  
whether the values satisfy each inequality. G

51. To be a member of the marching band, a student 53. SHORT RESPONSE If 3x = 2y and 5y = 6z, what
must have a grade-point average of at least 2.0 and is the value of x in terms of z?
must have attended at least five after-school
practices. Choose the system of inequalities that best SOLUTION:  
represents this situation.
 
A  
Substitute  in the equation 5y = 6z.
B  

SOLUTION:   ANSWER:  
The system of inequalities is:

The correct choice is A. 54. GEOMETRY Look at the graph below. Which of


these statements describes the relationship between
ANSWER:   the two lines?
A  
52. ACT/SAT The table at the right shows a relationship
between x and y. Which equation represents this
relationship?
 

 
A They intersect at (6, 2).
B They intersect at (0, 2).
  C They intersect at (3.5, 0).
  D They intersect at (2, 6).
F SOLUTION:  
G The lines intersect at the point (2, 6).
H The correct choice is D.
J  ANSWER:  
K
D
SOLUTION:  
55. GEOMETRY Find the coordinates of the vertices
Each output is two more than three times the input.
of the parallelogram whose sides are contained in the
The correct choice is G.
lines with equations
ANSWER:  
G SOLUTION:  
53. SHORT RESPONSE If 3x = 2y and 5y = 6z, what Graph the equations y = 3, y = 2x, and .
eSolutions Manual - Powered by Cognero   Page 22
is the value of x in terms of z?
SOLUTION:  
The lines intersect at the point (2, 6). The coordinates of the vertices are (1.5, 3), (3.5, 7),
The correct choice is D. (8, 3), and (10, 7).

ANSWER:   ANSWER:  
3-2 Solving Systems of Inequalities by Graphing
D

55. GEOMETRY Find the coordinates of the vertices Graph each inequality.


of the parallelogram whose sides are contained in the 56. 
lines with equations
SOLUTION:  
The boundary of the graph is the graph of x + y = 6.
SOLUTION:   Since the inequality symbol is ≥, the boundary is solid.
Graph the equations y = 3, y = 2x, and . Test the inequality with the point (0, 0).
   

 
The region that does not contain (0, 0) is shaded.
 

 
The coordinates of the vertices are (1.5, 3), (3.5, 7),
(8, 3), and (10, 7).

ANSWER:  

Graph each inequality.


56. 
ANSWER:  
SOLUTION:  
The boundary of the graph is the graph of x + y = 6.
Since the inequality symbol is ≥, the boundary is solid.
Test the inequality with the point (0, 0).
 

 
The region that does not contain (0, 0) is shaded.
  57. 
SOLUTION:  
The boundary of the graph is the graph of 4x – 3y =
10. Since the inequality symbol is <, the boundary is
dashed.
Test the inequality with the point (0, 0).
 

eSolutions Manual - Powered by Cognero


  Page 23
The region that contains (0, 0) is shaded.
 
3-2 Solving Systems of Inequalities by Graphing

57.  58. 
SOLUTION:   SOLUTION:  
The boundary of the graph is the graph of 4x – 3y = The boundary of the graph is the graph of 5x + 7y =
10. Since the inequality symbol is <, the boundary is –20. Since the inequality symbol is ≥, the boundary is 
dashed. solid.
Test the inequality with the point (0, 0). Test the inequality with the point (0, 0).
   

   
The region that contains (0, 0) is shaded. The region that contains (0, 0) is shaded.
   

  ANSWER:  

ANSWER:  

Graph each function. Identify the domain and


range.

58.  59. 

SOLUTION:  
SOLUTION:  
The boundary of the graph is the graph of 5x + 7y =
–20. Since the inequality symbol is ≥, the boundary is 
solid.
Test the inequality with the point (0, 0).
 

 
eSolutions Manual - Powered by Cognero Page 24
The region that contains (0, 0) is shaded.
  D = {all real numbers},
D = {all real numbers},
3-2 Solving Systems of Inequalities by Graphing

Graph each function. Identify the domain and


range. 60. 

59.  SOLUTION:  

SOLUTION:  

D = {all real numbers},


R = {all real numbers},
D = {all real numbers}, ANSWER:  

ANSWER:  

D = {all real numbers},,


R = {all real numbers},

D = {all real numbers}, 61. 

SOLUTION:  

60. 

SOLUTION:  

ANSWER:  
D = {all real numbers},
R = {all real numbers},
eSolutions Manual - Powered by Cognero Page 25
ANSWER:  
D = {all Systems
3-2 Solving real numbers},,
of Inequalities by Graphing
R = {all real numbers},

62. BOOK CLUB For each meeting of the Putnam


61.  High School book club, $25 is taken from the
activities account to buy snacks and materials. After
SOLUTION:   their sixth meeting, there will be $350 left in the
activities account.
a. If no money is put back into the account, what
equation can be used to show how much money is
left in the activities account after having x number of
meetings?
b. How much money was originally in the account?
c. After how many meetings will there be no money
left in the activities account?
SOLUTION:  
a. The amount spent for 6 meetings is $150.
The amount remaining in the account after 6
meetings is $350. Therefore, the initial amount in the
account is $500.
ANSWER:   Let y be the amount remaining in the account after x
meetings.
So:
y = 500 – 25x.
b. $500.
c.  Replace y with 0 and find x.

After 20 meetings, there will be no money left in the


account.

ANSWER:  
a. y = 500 – 25x
62. BOOK CLUB For each meeting of the Putnam b. $500
High School book club, $25 is taken from the c. 20
activities account to buy snacks and materials. After
their sixth meeting, there will be $350 left in the Find each value if f(x) = 2x + 5 and g(x) = 3x –
activities account. 4.
a. If no money is put back into the account, what 63. 
equation can be used to show how much money is
left in the activities account after having x number of SOLUTION:  
meetings?
b. How much money was originally in the account?  
c. After how many meetings will there be no money Replace x with – 3.
left in the activities account?  
SOLUTION:  
a. The amount spent for 6 meetings is $150.
The amount remaining in the account after 6
meetings is $350. Therefore, the initial amount in the ANSWER:  
account is $500. –1
Let y be the amount remaining in the account after x
meetings.
64. 
So:
eSolutions Manual - Powered by Cognero Page 26
y = 500 – 25x. SOLUTION:  
b. $500.
ANSWER:  
3-2 Solving Systems of Inequalities by Graphing ANSWER:  
–1
–5.5

64.  67. 

SOLUTION:   SOLUTION:  

   
Replace x with –2. Replace x with –0.25.
   

ANSWER:   ANSWER:  
–10 4.5

65.  68. 

SOLUTION:   SOLUTION:  

   
Replace x with –1. Replace x with –0.75.
   

ANSWER:   ANSWER:  
3 –6.25

66. 

SOLUTION:  

 
Replace x with –0.5.
 

ANSWER:  
–5.5

67. 

SOLUTION:  

 
Replace x with –0.25.
 

eSolutions Manual - Powered by Cognero Page 27

ANSWER:  
3-3 Optimization with Linear Programming

Graph each system of inequalities. Name the


coordinates of the vertices of the feasible
region. Find the maximum and minimum values 2. 
of the given function for this region.

1.  SOLUTION:  

SOLUTION:  

 
The lines, y = –3x + 6 and –y = x intersect at (1, 3).
The lines, –y = x and y = 3 intersect at (–3, 3).
The lines, y = –3x + 6 and y = 3 intersect at (3, –3).
  Therefore, the vertices of the feasible region are (1,
The lines y = 5 and y = –x intersect at (–5, 5). 3), (3, –3) and (–3, 3).
The lines x = 4 and y = –x intersect at (4, –4). Substitute the points (1, 3), (3, –3) and (–3, 3) in the
The lines y = 5 and x = 4 intersect at (4, 5). function f (x, y) = 8x + 4y.
Therefore, the vertices of the feasible region are (4,  
5), (4, –4) and (–5, 5).
Substitute the points (4, 5) (4, –4) and (–5, 5) in the
function f (x, y) = 5x – 2y.
 

 
Therefore, the maximum value is 20 and the
minimum value is –12.

  ANSWER:  
Therefore, the maximum value is 28 and the
minimum value is –35.

ANSWER:  

(1, 3), (3, –3), (–3, 3); max = 20, min = –12

3. 

eSolutions Manual - Powered by Cognero SOLUTION:   Page 1

2. 
3-3 Optimization with Linear Programming
(1, 3), (3, –3), (–3, 3); max = 20, min = –12 (2, –4), (4, –4); max does not exist, min = –52

3.  4. 

SOLUTION:   SOLUTION:  

 
The lines, y = –3x + 2 and y = –4 intersect at (2, –4).  
The lines, 9x + 3y = 24 and y = –4 intersect at (4, – The lines 3y = 4x + 26, y = 6 and y = –2x + 2, are
4). intersects at (–2, 6).
Therefore, the vertices of the feasible region are (2, The lines 3y = 4x + 26 and y = –2, are intersects at
–4) and (4, –4). (2, –2).
Substitute the points (2, –4) and (4, –4) in the The lines y = 6 and y = –2x + 2, are intersects at (–8,
function f (x, y) = 2x + 14y. –2).
  Substitute the points (–2, 6), (2, –2) and (–8, –2) in
the function f (x, y) = –3x – 6y.
 

 
So, the minimum value is –52.
Consider another point on the feasible region, (0, 3),
yields a value of 14, which is greater than –48.  
Therefore, there is no maximum value. Therefore, the maximum value is  36 and the 
minimum value is –30.
ANSWER:  
ANSWER:  

(2, –4), (4, –4); max does not exist, min = –52

(2, –2), (–8, –2), (–2, 6); max = 36 min = –30


4. 

5. 
eSolutions Manual - Powered by Cognero Page 2
SOLUTION:  

SOLUTION:  
So, the minimum value is –26.
Consider another point on the feasible region, (–12,
7), yields a value of 207, which is greater than 183.
3-3 Optimization with Linear Programming Therefore, there is no maximum value.
(2, –2), (–8, –2), (–2, 6); max = 36 min = –30
ANSWER:  

5. 

SOLUTION:  

(4, 2), (–1, –3), (–6, 7); max does not exist; min = –
26

6. 

 
The lines 4y = 4x – 8 and y = –3 intersect at (–1, –
SOLUTION:  
3).
The lines 3x + 6y = 12 and 4y = 4x – 8 intersect at
.
The lines 3x + 6y = 24 and y = 7, are intersects at
(–10, 7).
Therefore, the vertices of the feasible region are
(–1, –3), and (–10, 7).
 
Substitute the points (–1, –3), (4, 2) and (–6, 7) in the
The lines y = 2x + 6 and y = –2x –18 intersect at (–6,
function f (x, y) = –12x + 9y.
–6).
  The lines y = 2x + 18 and y = –2x –18 intersect at (–
2.5, –13).
Therefore, the vertices of the feasible region are (–6,
–6) and (–2.5, –13).
Substitute the points (–6, –6) and (–2.5, –13) in the
function f (x, y) = 5x –4y.
 
 
So, the minimum value is –26.
Consider another point on the feasible region, (–12,
7), yields a value of 207, which is greater than 183.
 
Therefore, there is no maximum value.
So, the maximum value is 39.5.
ANSWER:   Consider another point on the feasible region, (8, 10),
yields a value of –10, which is less than –6.
Therefore, there is no minimum value.

ANSWER:  

eSolutions Manual - Powered by Cognero Page 3

(4, 2), (–1, –3), (–6, 7); max does not exist; min = –
7. CCSS PRECISION The total number of workers’ 
hours per day available for production in a
(4, 2), (–1, –3),with
3-3 Optimization (–6,Linear does not exist; min = –
Programming
7); max skateboard factory is 85 hours. There are 40 hours
26 available for finishing decks and quality control each
day. The table shows the number of hours needed in
each department for two different types of
skateboards.
6.   

SOLUTION:    
a. Write a system of inequalities to represent the
situation.
b. Draw the graph showing the feasible region.
c. List the coordinates of the vertices of the feasible
region.
d. If the profit on a pro board is $50 and the profit on
a specialty board is $65, write a function for the total
profit on the skateboards.
  e . Determine the number of each type of skateboard
The lines y = 2x + 6 and y = –2x –18 intersect at (–6, that needs to be made to have a maximum profit.
–6). What is the maximum profit?
The lines y = 2x + 18 and y = –2x –18 intersect at (– SOLUTION:  
2.5, –13).
a. Let g and c represent number of pro and specialty
Therefore, the vertices of the feasible region are (–6,
–6) and (–2.5, –13). board.
Substitute the points (–6, –6) and (–2.5, –13) in the Constraints:
function f (x, y) = 5x –4y.  
 

b.
 
So, the maximum value is 39.5.
Consider another point on the feasible region, (8, 10),
yields a value of –10, which is less than –6.
Therefore, there is no minimum value.

ANSWER:  

 
c. The vertices of the feasible region are (0, 0),
(0,20) and (80, 0).
d. Optimizing function:
 
(–6,–6), (–2.5,–13), no min, max = 39.5

7. CCSS PRECISION The total number of workers’   


hours per day available for production in a e . Substitute the points (0, 0), (0,20) and (80, 0) in the
skateboard factory is 85 hours. There are 40 hours function f (x, y) = 65c + 50g.
available for finishing decks and quality control each  
day. The table shows the number of hours needed in
each department for two different types of
eSolutions Manual - Powered by Cognero Page 4
skateboards.
 
$5200
  Graph each system of inequalities. Name the
e . Substitute the
3-3 Optimization points
with (0, 0),
Linear (0,20) and (80, 0) in the
Programming coordinates of the vertices of the feasible
function f (x, y) = 65c + 50g. region. Find the maximum and minimum values
  of the given function for this region.

8. 

  SOLUTION:  
The maximum value is 5200 at (80, 0).
To maximize the profit 80 specialty boards and 0 pro
boards to be made.
The maximum profit is $5200.

ANSWER:  

a.
 
The lines y = 4 and 2y = –x + 4 intersect at (–4, 4).
b.
The lines y = 4 and 4y – 6x = – 32 intersect at (8, 4).
The lines y = 1 and 2y = –x + 4 intersect at (2, 1).
The lines y = 1 and 4y – 6x = – 32 intersect at (6, 1).
Therefore, the vertices of the feasible region are (8,
4), (–4, 4), (6, 1) and (2, 1).
Substitute the points (8, 4), (–4, 4), (6, 1) and (2, 1) in
the function f (x, y) = –6x + 3y.
 

c. (0, 0), (0, 20), (80, 0)


d.
e . 80 specialty boards, 0 pro boards;
$5200
 
Graph each system of inequalities. Name the Therefore, the maximum value is 36 and the
coordinates of the vertices of the feasible minimum value is –36.
region. Find the maximum and minimum values
of the given function for this region. ANSWER:  

8. 

SOLUTION:  

(8, 4), (6, 1), (2, 1), (–4, 4); max = 36, min = –36

9. 
eSolutions Manual - Powered by Cognero Page 5

 
The lines y = 4 and 2y = –x + 4 intersect at (–4, 4). SOLUTION:  
3-3 Optimization with
(8, 4), (6, 1), (2, 1),Linear
(–4, 4);Programming
max = 36, min = –36 (2, –10), (–3, 0), (–3, 6.5), (2, 9); max = 82, min = –
89

9.  10. 

SOLUTION:   SOLUTION:  

 
The lines x = 2 and y = –2x – 6 intersect at (2, –10).  
The lines x = 2 and 4y = 2x + 32 intersect at (2, 9). The lines x = –2, y = 5 and y = 8 intersect at (–2, 5),
The lines x = –3 and y = –2x – 6 intersect at (–3, 0). (–2, 8).
The lines x = –3 and 4y = 2x + 32 intersect at (–3, The lines x = 4 and 2x + 3y = 26 intersect at (4, 6).
6.5). The lines x = 4 and y = 5 intersect at (4, 5).
Therefore, the vertices of the feasible region are (2, The lines y = 8 and 2x + 3y = 26 intersect at (1, 8).
–10), (2, 9), (–3, 0) and (–3, 6.5). Therefore, the vertices of the feasible region are (1,
Substitute the points (2, –10), (2, 9), (–3, 0) and (–3, 8), (4, 6), (4, 5), (–2, 5) and (–2, 8).
6.5) in the function f (x, y) = –4x – 9y. Substitute the points (1, 8), (4, 6), (4, 5), (–2, 5) and
  (–2, 8) in the function f (x, y) = 8x – 10y.
 

 
Therefore, the maximum value is 82 and the
minimum value is –89.  
ANSWER:   Therefore, the maximum value is –18 and the
minimum value is –96.

ANSWER:  

(2, –10), (–3, 0), (–3, 6.5), (2, 9); max = 82, min = –
89

(1, 8), (4, 6), (4, 5), (–2, 5), (–2, 8); max = –18, min
eSolutions Manual - Powered by Cognero
10.  = –96 Page 6
3-3 Optimization with Linear Programming (1, 8), (4, 6), (4, 5), (–2, 5), (–2, 8); max = –18, min
= –96

(1, 8), (4, 6), (4, 5), (–2, 5), (–2, 8); max = –18, min
= –96
11. 

11. 
SOLUTION:  

SOLUTION:  

 
The lines y = –8 and y = x intersect at (–8, –8).
The lines y = –8 and y = –3x + 10 intersect at (6, –
  8).
The lines y = –8 and y = x intersect at (–8, –8). The lines y = –2 and y = x intersect at (–2, –2).
The lines y = –8 and y = –3x + 10 intersect at (6, – The lines y = –2 and y = –3x + 10 intersect at (4, –
8). 2).
The lines y = –2 and y = x intersect at (–2, –2). Therefore, the vertices of the feasible region are (–8,
The lines y = –2 and y = –3x + 10 intersect at (4, – –8), (6, –8) (–2, –2) and (4, –2).
2). Substitute the points (–8, –8), (6, –8) (–2, –2) and (4,
Therefore, the vertices of the feasible region are (–8, –2) in the function f (x, y) = 5x + 14y.
–8), (6, –8) (–2, –2) and (4, –2).  
Substitute the points (–8, –8), (6, –8) (–2, –2) and (4,
–2) in the function f (x, y) = 5x + 14y.
 

 
Therefore, the maximum value is –8 and the
minimum value is –152.
 
Therefore, the maximum value is –8 and the ANSWER:  
minimum value is –152.

ANSWER:  

(6, –8), (4, –2), (–2, –2), (–8, –8);max = –8, min = –
152

(6, –8), (4, –2), (–2, –2), (–8, –8);max = –8, min = –
eSolutions Manual - Powered by Cognero Page 7
152
12. 
(6, –8), (4, –2),with
3-3 Optimization –2), (–8,
(–2,Linear –8);max = –8, min = –
Programming
152 (2, 0), (6, –1), (6, 3), (2, 5); max = 57, min = 12

12.  13. 

SOLUTION:   SOLUTION:  

   
The lines x = 2 and 2x + 4y = 24 intersect at (2, 5). The lines y = 3 and 2y + x = 8 intersect at (2, 3).
The lines x = 2 and x + 4y = 2 intersect at (2, 0). The lines y = 3 and y – 2x = 23 intersect at (–10, 3).
The lines x = 6 and 2x + 4y = 24 intersect at (6, 3). The lines y = 7 and 2y + x = 8 intersect at (–6, 7).
The lines x = 6 and x + 4y = 2 intersect at (6, –1). The lines y = 7 and y – 2x = 23 intersect at (–8, 7).
Therefore, the vertices of the feasible region are (2, Therefore, the vertices of the feasible region are (2,
5), (2, 0), (6, 3) and (6, –1). 3), (–10, 3), (–6, 7) and (–8, 7).
Substitute the points (2, 5), (2, 0), (6, 3) and (6, –1) in Substitute the points (2, 3), (–10, 3), (–6, 7) and (–8,
the function f (x, y) = 6x + 7y. 7) in the function f (x, y) = –3x + 5y.
   

   
Therefore, the maximum value is 57 and the Therefore, the maximum value is 59 and the
minimum value is 12. minimum value is 9.

ANSWER:   ANSWER:  

(2, 0), (6, –1), (6, 3), (2, 5); max = 57, min = 12 (–10, 3), (2, 3), (–6, 7), (–8, 7); max = 59, min = 9

Graph each system of inequalities. Name the


coordinates of the vertices of the feasible
eSolutions Manual - Powered by Cognero Page 8
13.  region. Find the maximum and minimum values
of the given function for this region.
 
Therefore, the maximum value is –140 and the
3-3 Optimization
(–10, 3), (2, 3),with
(–6,Linear
7), (–8,Programming
7); max = 59, min = 9 minimum value is –252.

Graph each system of inequalities. Name the ANSWER:  


coordinates of the vertices of the feasible
region. Find the maximum and minimum values
of the given function for this region.

14. 

SOLUTION:  
(–9, –9), (–4, –9), (–5, –5), (–9, –5); max = –140,
min = –252

15. 

SOLUTION:  
 
The lines y = –5 and x = –9 intersect at (–9, –5).
The lines y = –5 and 3y + 12x = –75 intersect at (–5,
–5).
The lines y = –9 and x = –9 intersect at (–9, –9).
The lines y = –9 and 3y + 12x = –75 intersect at (–5,
–9).
Therefore, the vertices of the feasible region are (–9,
–5), (–9, –9), (–5, –5) and (–5, –9).
Substitute the points (–9, –5), (–9, –9), (–5, –5) and
(–5, –9) in the function f (x, y) = 20x + 8y.
 
 
The lines x = –8 and 3x + 6y = 36 intersect at (–8,
10).
The lines x = –8 and 2y + 12 = 3x intersect at (–8, –
18).
The lines 3x + 6y = 36 and 2y + 12 = 3x intersect at
(6, 3).
Therefore, the vertices of the feasible region are (–8,
10), (–8, –18) and (6, 3).
 
Substitute the points (–8, 10), (–8, –18) and (6, 3) in
Therefore, the maximum value is –140 and the
the function f (x, y) = 10x – 6y.
minimum value is –252.
 
ANSWER:  

 
Therefore, the maximum value is 42 and the
minimum value is –140.

ANSWER:  
eSolutions Manual - Powered by Cognero Page 9

(–9, –9), (–4, –9), (–5, –5), (–9, –5); max = –140,
min = –252
(–9, –9), (–4, –9), (–5, –5), (–9, –5); max = –140,
3-3 Optimization
min = –252 with Linear Programming
(6, 3), (–8, 10), (–8, –18); max = 42, min = –140

15.  16. 

SOLUTION:   SOLUTION:  

   
The lines x = –8 and 3x + 6y = 36 intersect at (–8, The lines y = x – 2 and 8y +5x = 49 intersect at (5,
10). 3).
The lines x = –8 and 2y + 12 = 3x intersect at (–8, – The lines y = –(x – 2) and y = 8 intersect at (–6, 8).
18). The lines y = 8 and 8y +5x = 49 intersect at (–3, 8).
The lines 3x + 6y = 36 and 2y + 12 = 3x intersect at
The absolute value function  has a cusp at 
(6, 3).
Therefore, the vertices of the feasible region are (–8, (2, 0).
10), (–8, –18) and (6, 3). Therefore, the vertices of the feasible region are (5,
Substitute the points (–8, 10), (–8, –18) and (6, 3) in 3), (–6, 8), (–3, 8) and (2, 0).
the function f (x, y) = 10x – 6y. Substitute the points (5, 3), (–6, 8), (–3, 8) and (2, 0)
  in the function f (x, y) = –5x – 15y.
 

 
Therefore, the maximum value is 42 and the
minimum value is –140.  
Therefore, the maximum value is –10 and the
ANSWER:  
minimum value is –105.

ANSWER:  

(6, 3), (–8, 10), (–8, –18); max = 42, min = –140

(2, 0), (5, 3), (–3, 8), (–6, 8), max = –10, min = –105
eSolutions Manual - Powered by Cognero Page 10
16. 
3-3 Optimization with Linear Programming
(–6, 1), (6, –7), (–6, 5); max = 48, min = 0

(2, 0), (5, 3), (–3, 8), (–6, 8), max = –10, min = –105

18. 

17. 

SOLUTION:  

SOLUTION:  

 
  The lines y = –5 and y = –4x + 15 intersect at (5, –
5).
The lines x = –6 and y + x = –1 intersect at (–6, 5).
The lines y = –17 and y = –4x + 15 intersect at (8, –
The lines x = –6 and 2x +3y  = –9 intersect at (–6,
17).
1).
Therefore, the vertices of the feasible region are (5,
The lines 2y + x = –8 and y + x = –1 intersect at (6,
–5) and (8, –17).
–7).
Substitute the points (5, –5) and (8, –17) in the
Therefore, the vertices of the feasible region are (–6,
function f (x, y) = 8x – 3y.
5),
(–6, 1) and (6, –7).  
Substitute the points (–6, 5), (–6, –1) and (6, –7) in
the function f (x, y) = –10x – 12y.
 

 
  So, the minimum value is –49 and the maximum
Therefore, the maximum value is 48 and the value is 115.
minimum value is 0.  
ANSWER:   ANSWER:  

(–6, 1), (6, –7), (–6, 5); max = 48, min = 0


(5, –5), (8, –17), (-12,-17), (-8,-5) : max =115, min =
-49
eSolutions Manual - Powered by Cognero Page 11

18. 
3-3 Optimization with Linear Programming
(5, –5), (8, –17), (-12,-17), (-8,-5) : max =115, min =
-49 (–8, 44), (16, 32), (–8, –26), (16, 22); max = 672,
min = –486

19. 
20. 

SOLUTION:  

SOLUTION:  

 
The lines x = –8 and 2y + x = 80 intersect at (–8,
44).  
The lines x = –8 and y = 2x – 10 intersect at (–8, –
The lines y = x + 4 and y = –x + 10 intersect at (3,
26).
7).
The lines x = 16 and 2y + x = 80 intersect at (16, 32).
The lines y = x – 4 and y = –x + 10 intersect at (7,
The lines x = 16 and y = 2x – 10 intersect at (16, 22).
3).
Therefore, the vertices of the feasible region are (–8,
The lines y = x + 4 and y = –x – 10 intersect at (–7,
44), (–8, –26), (16, 32) and (16, 22).
–3).
Substitute the points (–8, 44), (–8, –26), (16, 32) and
The lines y = x – 4 and y = –x – 10 intersect at (–3,
(16, 22) in the function f (x, y) = 12x + 15y.
–7).
  Therefore, the vertices of the feasible region are (3,
7), (7, 3), (–7, –3) and (–3, –7).
Substitute the points (3, 7), (7, 3), (–7, –3) and (–3, –
7) in the function f (x, y) = –10x + 9y.
 

 
Therefore, the maximum value is 672 and the
minimum value is –486.

ANSWER:  
 
Therefore, the maximum value is 43 and the
minimum value is –43.

ANSWER:  

(–8, 44), (16, 32), (–8, –26), (16, 22); max = 672,
eSolutions = –486- Powered by Cognero
min Manual Page 12
 
Therefore, the maximum value is 43 and the
3-3 Optimization is –43.
minimum valuewith Linear Programming
 
ANSWER:   Therefore, the maximum value is 60 and the
minimum value is –112.

ANSWER:  

(3, 7), (7, 3), (–3, –7), (–7, –3); max = 43,min = –43

(5, –1), (1, 6), (–2, –8), (–4, –8), (–4, 6), max = 60,
min = –112
21. 

22. 
SOLUTION:  

SOLUTION:  

 
The lines x = –4 and y = –8 intersect at (–4, –8).
The lines x = –4 and y = 6 intersect at (–4, 6).
The lines y = x – 6 and y = –8 intersect at (2, –8).  
The lines y = x – 6 and 4y +7x = 31 intersect at (5, – The lines y = 6 and x + 3y = 14 intersect at (–4, 6).
1). The lines x = 2 and x + 3y = 14 intersect at (2, 4).
The lines 4y +7x = 31 and y = 6 intersect at (1, 6). The lines x = 2 and y = x +1 –2 intersect at (2, 1).
Therefore, the vertices of the feasible region are (–4, The lines y = 0 and y = x +1 –2 intersect at (1, 0).
–8), (–4, 6), (2, –8), (5, –1) and (1, 6). The lines y = 0 and y = –(x +1) –2 intersect at (–3,
Substitute the points (–4, –8), (–4, 6), (2, –8), (5, –1) 0).
and (1, 6) in the function f (x, y) = 12x + 8y. The lines x = –6 and y = –(x +1) –2 intersect at (–6,
  3).
The lines x = –6 and y = 6 intersect at (–6, 6).
Therefore, the vertices of the feasible region are (–4,
6), (2, 4), (2, 1), (1, 0), (–3, 0), (-6, 3) and (–6, 6).
Substitute the points (–4, 6), (2, 4), (2, 1), (1, 0), (–3,
0), (-6, 3) and (–6, 6) in the function f (x, y) = 5x +
4y.
 

 
Therefore, the maximum value is 60 and the
minimum value is –112.
eSolutions Manual - Powered by Cognero Page 13
ANSWER:  
Therefore, the vertices of the feasible region are (–4,
6), (2, 4), (2, 1), (1, 0), (–3, 0), (-6, 3) and (–6, 6).
Substitute the points (–4, 6), (2, 4), (2, 1), (1, 0), (–3,
0), (-6, 3) and with
3-3 Optimization (–6, 6) in the Programming
Linear function f (x, y) = 5x + (–4, 6), (2, 4), (2, 1), (1, 0), (–3, 0), (–6, 3), (–6, 6);
4y. max = 26, min = –18
 
23. COOKING Jenny’s Bakery makes two types of
birthday cakes: yellow cake, which sells for $25, and
strawberry cake, which sells for $35. Both cakes are
the same size, but the decorating and assembly time
required for the yellow cake is 2 hours, while the
time is 3 hours for the strawberry cake. There are
450 hours of labor available for production. How
many of each type of cake should be made to
maximize revenue?
SOLUTION:  
  Let x and y be the number of strawberry and yellow
Therefore, the maximum value is 26 and the cakes.
minimum value is –18. Optimizing function:
 
ANSWER:  

 
Constraints:

 
The vertices of the feasible region are (0, 0), (0, 225)
and (150, 0).
(–4, 6), (2, 4), (2, 1), (1, 0), (–3, 0), (–6, 3), (–6, 6);
max = 26, min = –18  

23. COOKING Jenny’s Bakery makes two types of


birthday cakes: yellow cake, which sells for $25, and
strawberry cake, which sells for $35. Both cakes are
the same size, but the decorating and assembly time
required for the yellow cake is 2 hours, while the  
time is 3 hours for the strawberry cake. There are The maximum value is 5625. Therefore, 225 yellow
450 hours of labor available for production. How cakes and 0 strawberry cakes will maximize the
many of each type of cake should be made to revenue.
maximize revenue?
SOLUTION:   ANSWER:  
Let x and y be the number of strawberry and yellow 225 yellow cakes, 0 strawberry cakes
cakes.
24. BUSINESS The manager of a travel agency is
Optimizing function:
printing brochures and fliers to advertise special
 
discounts on vacation spots during the summer
months. Each brochure costs $0.08 to print, and each
  flier costs $0.04 to print. A brochure requires 3
Constraints: pages, and a flier requires 2 pages. The manager
does not want to use more than 600 pages, and she
needs at least 50 brochures and 150 fliers. How
many of each should she print to minimize the cost?

  SOLUTION:  
The vertices of the feasible region are (0, 0), (0, 225) Let x and y be the number of brochures and fliers.
and (150, 0). Optimizing function:
eSolutions Manual - Powered by Cognero
    Page 14

 
cakes and 0 strawberry cakes will maximize the b. Draw a graph showing the feasible region and list
revenue. the coordinates of the vertices of the feasible region.
c. If the profit is $26 per shed and $30 per play
ANSWER:   with Linear Programming
3-3 Optimization house, how many of each should he paint?
225 yellow cakes, 0 strawberry cakes d. What is the maximum profit?
24. BUSINESS The manager of a travel agency is SOLUTION:  
printing brochures and fliers to advertise special a. Let a and b be the number of sheds and houses.
discounts on vacation spots during the summer
months. Each brochure costs $0.08 to print, and each
b.
flier costs $0.04 to print. A brochure requires 3
pages, and a flier requires 2 pages. The manager
does not want to use more than 600 pages, and she
needs at least 50 brochures and 150 fliers. How
many of each should she print to minimize the cost?
SOLUTION:  
Let x and y be the number of brochures and fliers.
Optimizing function:
 

 
Constraints:  
c. Substitute (25, 20), (0,0), (0, 40) and (45, 0) in the
function f (a, b) = 26a + 30b.
 

 
The vertices of the feasible region are (50, 150),
(100, 150) and (50, 225).
Substitute the points (50, 150), (100, 150) and (50,
225) in the function f (x, y) = 0.08x+ 0.04y.
 
 
The maximum value 1250 at (25, 20). Therefore, he
should paint 25 sheds and 20 houses.
 
d. The maximum value is 1250.
  Therefore, the maximum profit is $1250.
The minimum value 10 at (50, 150).
To minimize the cost, she would print 50 brochures ANSWER:  
and 150 fliers. a.
ANSWER:   b.
50 brochures, 150 fliers

25. CCSS PRECISION Sean has 20 days to paint play


houses and sheds. The sheds can be painted at a rate
of 2.5 per day, and the play houses can be painted at
a rate of 2 per day. He has 45 structures that need to
be painted.
a. Write a system of inequalities to represent the
possible ways Sean can paint the structures.
b. Draw a graph showing the feasible region and list
the coordinates of the vertices of the feasible region.
c. If the profit is $26 per shed and $30 per play c. 25 sheds, 20 play houses
house, how many of each should he paint? d. $1250
d. What is the maximum profit?
eSolutions Manual - Powered by Cognero
26. MOVIES Employees at a local movie theater workPage 15
SOLUTION:   8-hour shifts from noon to 8 P.M. or from 4 P.M. to
a. Let a and b be the number of sheds and houses. midnight. The table below shows the number of
employees needed and their corresponding pay. Find
night-shift workers.  The minimal cost is $776.

ANSWER:  
c. 25 sheds, 20with
3-3 Optimization playLinear
houses Programming 8 day-shift and 6 night-shift workers; $776
d. $1250
27. BUSINESS Each car on a freight train can hold
26. MOVIES Employees at a local movie theater work 4200 pounds of cargo and has a capacity of 480
8-hour shifts from noon to 8 P.M. or from 4 P.M. to cubic feet. The freight service handles two types of
midnight. The table below shows the number of packages: small, which weigh 25 pounds and are 3
employees needed and their corresponding pay. Find cubic feet each, and large, which are 50 pounds and
the numbers of day-shift workers and night-shift are 5 cubic feet each. The freight service charges $5
workers that should be scheduled to minimize the for each small package and $8 for each large
cost. What is the minimal cost? package.
  a. Find the number of each type of package that
should be placed on a train car to maximize revenue.
b. What is the maximum revenue per train car?
c. In this situation, is maximizing the revenue
necessarily the best thing for the company to do?
  Explain.
  SOLUTION:  
  a. Let x be the number of small packages.
SOLUTION:   Let y be the number of large packages.
 
Let x be the number of day-shift workers and let y
Optimizing function:
be the number of night-shirt workers.
Optimizing function:
   
Constraints:

 
Constraints:
 
The vertices of the feasible region are (0,0), (0, 84),
(160, 0) and (120, 24).
  Substitute the points (0,0), (0, 84), (160, 0) and (120,
The vertices of the feasible region are (5, 9) and (8, 24) in the function f (x, y) = 5x+ 8y.
6).  
Substitute the coordinates in the optimizing function.
 

 
160 small packages and 0 large packages.
 
  b. The maximum value is 800. That is the maximum
So, the theater should schedule 8 day-shift and 6 revenue per train car is $800.
night-shift workers.  The minimal cost is $776. c. No. If revenue is maximized, the company will not
deliver any large packages, and customers with large
ANSWER:  
packages to ship will probably choose another
8 day-shift and 6 night-shift workers; $776 carrier.
27. BUSINESS Each car on a freight train can hold ANSWER:  
4200 pounds of cargo and has a capacity of 480 a. 160 small packages, 0 large packages
eSolutions
cubicManual
feet. -The
Powered by Cognero
freight service handles two types of Page 16
b. $800
packages: small, which weigh 25 pounds and are 3
c. No; if revenue is maximized, the company will not
cubic feet each, and large, which are 50 pounds and
deliver any large packages, and customers with large
c. No. If revenue is maximized, the company will not If the plant maximized processing, the profit is
deliver any large packages, and customers with large $23,250.
packages to ship will probably choose another
carrier.
3-3 Optimization with Linear Programming ANSWER:  
$23,250
ANSWER:  
a. 160 small packages, 0 large packages 29. OPEN ENDED Create a set of inequalities that
b. $800 2
forms a bounded region with an area of 20 units and
c. No; if revenue is maximized, the company will not lies only in the fourth quadrant.
deliver any large packages, and customers with large
packages to ship will probably choose another SOLUTION:  
carrier. Sample answer:

28. RECYCLING A recycling plant processes used ANSWER:  


plastic into food or drink containers. The plant Sample answer:
processes up to 1200 tons of plastic per week. At
least 300 tons must be processed for food containers, 30. CHALLENGE Find the area of the bounded region
while at least 450 tons must be processed for drink formed by the following constraints:
containers. The profit is $17.50 per ton for
processing food containers and $20 per ton for
processing drink containers. What is the profit if the SOLUTION:  
plant maximizes processing?
Vertices of the feasible region are (0, 0), (3, 0), (1.5,
SOLUTION:   1.5) and (1.5, –1.5).
Let x be the number of tons of food containers This vertices form a square.
processed. The side of the feasible region is 2.12132 unit.
Let y be the number of tons of drink containers Therefore, the area of the feasible region is
2 2
processed. (2.12132) or 4.5 unit .
 
Optimizing function: ANSWER:  
 

  31. CCSS ARGUMENTS Identify the system of


Constraints: inequalities that is not the same as the other three.
Explain your reasoning.
a.

 
The vertices of the feasible region are (300, 450),
(300, 900) and (750, 450).
Substitute the points (300, 450), (300, 900) and (750,
450) in the function f (x, y) = 17.5x + 20y.
 
b.

 
Therefore, the maximum value is 23250.
If the plant maximized processing, the profit is
$23,250. c.
ANSWER:  
$23,250

29. OPEN ENDED Create a set of inequalities that


2
forms
eSolutions a bounded
Manual region
- Powered with an
by Cognero area of 20 units and Page 17
lies only in the fourth quadrant.
SOLUTION:   d.
2 2 the other three are bounded.
(2.12132) or 4.5 unit .
ANSWER:  
ANSWER:  
3-3 Optimization with Linear Programming b; The feasible region of Graph b is unbounded while
the other three are bounded.

31. CCSS ARGUMENTS Identify the system of 32. REASONING Determine whether the following
inequalities that is not the same as the other three. statement is sometimes, always, or never true.
Explain your reasoning. Explain your reasoning.
a. An unbounded region will not have both a
maximum and minimum value.
SOLUTION:  
Sample answer: Always; if a point on the unbounded
region forms a minimum, then a maximum cannot
also be formed because of the unbounded region.
There will always be a value in the solution that will
produce a higher value than any projected maximum.
b.
ANSWER:  
Sample answer: Always; if a point on the unbounded
region forms a minimum, then a maximum cannot
also be formed because of the unbounded region.
There will always be a value in the solution that will
produce a higher value than any projected maximum.

33. WRITING IN MATH Upon determining a


c. bounded feasible region, Ayumi noticed that vertices
A(–3, 4) and B(5, 2) yielded the same maximum
value for f (x, y) = 16y + 4x. Kelvin confirmed that
her constraints were graphed correctly and her
vertices were correct. Then he said that those two
points were not the only maximum values in the
feasible region. Explain how this could have
happened.
d. SOLUTION:  
Sample answer: Even though the region is bounded,
multiple maximums occur at A and B and all of the
points on the boundary of the feasible region
containing both A and B. This happened because that
boundary of the region has the same slope as the
function.

ANSWER:  
SOLUTION:   Sample answer: Even though the region is bounded,
b; The feasible region of Graph b is unbounded while multiple maximums occur at A and B and all of the
the other three are bounded. points on the boundary of the feasible region
containing both A and B. This happened because that
ANSWER:   boundary of the region has the same slope as the
b; The feasible region of Graph b is unbounded while function.
the other three are bounded.
34. Kelsey worked 350 hours during the summer and
32. REASONING Determine whether the following earned $2978.50. She earned $6.85 per hour when
statement is sometimes, always, or never true. she worked at a video store and $11 per hour as an
Explain your reasoning. architectural intern. Let x represent the number of
An unbounded region will not have both a hours she worked at the video store and y represent
maximum and minimum value. the number of hours that she interned. Which system
of equations represents this situation?
SOLUTION:  
eSolutions Manual - Powered by Cognero
Sample answer: Always; if a point on the unbounded A Page 18

region forms a minimum, then a maximum cannot


also be formed because of the unbounded region. B
multiple maximums occur at A and B and all of the Therefore, the total cost of the dinner including tip is
points on the boundary of the feasible region $60 + $10.20 = $70.20.
containing both A and B. This happened because that
boundary of the
3-3 Optimization region
with has Programming
Linear the same slope as the ANSWER:  
function. $70.20

34. Kelsey worked 350 hours during the summer and 36. ACT/SAT For a game she is playing, Liz must draw
earned $2978.50. She earned $6.85 per hour when a card from a deck of 26 cards, one with each letter
she worked at a video store and $11 per hour as an of the alphabet on it, and roll a die. What is the
architectural intern. Let x represent the number of probability that Liz will draw a letter in her name and
hours she worked at the video store and y represent roll an odd number?
the number of hours that she interned. Which system  
of equations represents this situation? F
A G
H
B
J
C K

SOLUTION:  
D
Liz has 3 letters in her name.
The probability of getting one of those three letters is
SOLUTION:  
Total number of hours worked is 350. .
That is, x + y = 350. On a die, there are 3 odd numbers.
Total amount she earned is $2978.50.
That is 6.85x + 11y = 2978.50 The probability of getting an odd number is .
Therefore, option B is the correct answer.
Therefore, the probability that Liz will draw a letter in
ANSWER:  
her name and roll an odd number is .
B
Option H is the correct answer.
35. SHORT RESPONSE A family of four went out to
dinner. Their bill, including tax, was $60. They left a ANSWER:  
17% tip on the total cost of their bill. What is the total H
cost of the dinner including tip?
37. GEOMETRY Which of the following best describes
SOLUTION:   the graphs of y = 3x – 5 and 4y = 12x + 16?
The tip is 17% of 60.  
A The lines have the same y-intercept.
So, the tip amount is $10.20. B The lines have the same x-intercept.
Therefore, the total cost of the dinner including tip is C The lines are perpendicular.
$60 + $10.20 = $70.20.
D The lines are parallel.
ANSWER:   SOLUTION:  
$70.20 The slope of the line y = 3x – 5 is 3.
The slope of the line 4y = 12x + 16 is 3.
36. ACT/SAT For a game she is playing, Liz must draw Since the slopes are equal, the lines are parallel.
a card from a deck of 26 cards, one with each letter Option D is the correct answer.
of the alphabet on it, and roll a die. What is the
probability that Liz will draw a letter in her name and ANSWER:  
roll an odd number? D
 
F Solve each system of inequalities by graphing.
G 38. 
H
SOLUTION:  
J Manual - Powered by Cognero
eSolutions Page 19

K
Since the slopes are equal, the lines are parallel.
Option D is the correct answer.

ANSWER:   with Linear Programming


3-3 Optimization
D

Solve each system of inequalities by graphing.


38.  40. 

SOLUTION:   SOLUTION:  

ANSWER:   no solution

ANSWER:  

no solution
39. 
41. BUSINESS Last year the chess team paid $7 per
SOLUTION:   hat and $15 per shirt for a total purchase of $330.
This year they spent $360 to buy the same number of
shirts and hats because the hats now cost $8 and the
shirts cost $16. Write and solve a system of two
equations that represents the number of hats and
shirts bought each year.
SOLUTION:  
Let x represent number hats and y represent number
of shirts.
 
The equations are
ANSWER:  
.
 
Solve.
 

eSolutions Manual - Powered by Cognero Page 20

40. 
 
 
There are 15 hats and 15 shirts.

3-3 Optimization with Linear Programming ANSWER:  


no solution hats: 15, shirts: 15

41. BUSINESS Last year the chess team paid $7 per Write an equation in slope-intercept form for
hat and $15 per shirt for a total purchase of $330. the line that satisfies each set of conditions.
This year they spent $360 to buy the same number of 42. passes through (5, 1) and (8, –4)
shirts and hats because the hats now cost $8 and the
SOLUTION:  
shirts cost $16. Write and solve a system of two
equations that represents the number of hats and Slope of the line passing through the points is (5, 1)
shirts bought each year. and (8, –4) is .
SOLUTION:  
Let x represent number hats and y represent number Substitute for m in the slope-intercept form.
of shirts.
 
The equations are
Substitute 5 for x and 1 for y and solve for b.
.
 
Solve.
 

 
Therefore, the equation of the line passing through
the points (5, 1) and (8, –4) is .

  ANSWER:  
Substitute 15 for y in the first equation and solve for
x.
 
43. passes through (–3, 5) and (3, 2)
SOLUTION:  
Slope of the line passing through the points is (–3, 5)

  and (3, 2) is .
There are 15 hats and 15 shirts.

ANSWER:   Substitute for m in the slope-intercept form.


hats: 15, shirts: 15

Write an equation in slope-intercept form for Substitute 3 for x and 2 for y and solve for b.
the line that satisfies each set of conditions.
42. passes through (5, 1) and (8, –4)
SOLUTION:  
Slope of the line passing through the points is (5, 1)
and (8, –4) is .
Therefore, the equation of the line passing through
Substitute for m in the slope-intercept form.
the points (–3, 5) and (3, 2) is .
eSolutions Manual - Powered by Cognero Page 21
Substitute 5 for x and 1 for y and solve for b. ANSWER:  
ANSWER:  
ANSWER:  
3-3 Optimization with Linear Programming

43. passes through (–3, 5) and (3, 2) Find the x-intercept and the y-intercept of the
graph of each equation. Then graph the
SOLUTION:   equation.
Slope of the line passing through the points is (–3, 5) 44. 
and (3, 2) is . SOLUTION:  
To find the x-intercept, substitute 0 for y and solve
Substitute for m in the slope-intercept form. for x.
 

Substitute 3 for x and 2 for y and solve for b.

 
To find the y-intercept, substitute 0 for x and solve
for y.
 

Therefore, the equation of the line passing through


the points (–3, 5) and (3, 2) is .
 
Therefore, the x- and the y- intercepts are 3 and 5
ANSWER:   respectively.
 

Find the x-intercept and the y-intercept of the


graph of each equation. Then graph the
equation.
44. 
SOLUTION:  
To find the x-intercept, substitute 0 for y and solve
for x.
  ANSWER:  
3; 5

 
To find the y-intercept, substitute 0 for x and solve
for y.
 

  45. 
Therefore, the x- and the y- intercepts are 3 and 5
respectively. SOLUTION:  
  To find the x-intercept, substitute 0 for y and solve
eSolutions Manual - Powered by Cognero
for x. Page 22
 
3-3 Optimization with Linear Programming

45.  46. 
SOLUTION:   SOLUTION:  
To find the x-intercept, substitute 0 for y and solve To find the x-intercept, substitute 0 for y and solve
for x. for x.
   

 
To find the y-intercept, substitute 0 for x and solve  
for y. To find the y-intercept, substitute 0 for x and solve
  for y.
 

 
Therefore, the x- and the y- intercepts are 6 and –2
respectively.  
  Therefore, the x- and the y- intercepts are
 respectively.
 

ANSWER:  
6; –2

ANSWER:  

46. 
SOLUTION:  
To find the x-intercept, substitute 0 for y and solve
for x.
  47. 
SOLUTION:  
eSolutions Manual - Powered by Cognero To find the x-intercept, substitute 0 for y and solve
Page 23
for x.
 
3-3 Optimization with Linear Programming

47.  48. 
SOLUTION:   SOLUTION:  
To find the x-intercept, substitute 0 for y and solve To find the x-intercept, substitute 0 for y and solve
for x. for x.
  0=x
To find the y-intercept, substitute 0 for x and solve
for y.
y =0
Therefore, the x- and the y- intercepts are 0 and 0
  respectively.
To find the y-intercept, substitute 0 for x and solve  
for y.
 

 
Therefore, the x- and the y- intercepts are 5 and 2
respectively.
 
ANSWER:  
0; 0

ANSWER:  
5; 2
49. 
SOLUTION:  
To find the x-intercept, substitute 0 for y and solve
for x.
 

48. 
 
SOLUTION:   To find the y-intercept, substitute 0 for x and solve
To find the x-intercept, substitute 0 for y and solve for y.
for x.  
0=x
To find the y-intercept, substitute 0 for x and solve
for y.
eSolutions Manual - Powered by Cognero Page 24
y =0 Therefore, the x- and the y- intercepts are  and –2
Therefore, the x- and the y- intercepts are 0 and 0
respectively. respectively.
3-3 Optimization with Linear Programming

49.  Evaluate each expression if x = –1, y = 3, and z


= 7.
SOLUTION:   50. 
To find the x-intercept, substitute 0 for y and solve
for x. SOLUTION:  
  Substitute –1, 3 and 7 for x, y and z and simplify.
 

ANSWER:  
  9
To find the y-intercept, substitute 0 for x and solve
for y.
  51. 
SOLUTION:  
Substitute –1, 3 and 7 for x, y and z and simplify.
 
Therefore, the x- and the y- intercepts are  and –2
respectively.
 

ANSWER:  
9

52. 
SOLUTION:  
Substitute –1, 3 and 7 for x, y and z and simplify.
 

ANSWER:  

ANSWER:  
–8

53. 
SOLUTION:  
Substitute –1, 3 and 7 for x, y and z and simplify.
 

Evaluate each expression if x = –1, y = 3, and z


= 7.
50.  ANSWER:  
SOLUTION:   –5
Substitute –1, 3 and 7 for x, y and z and simplify.
54. 
 
eSolutions Manual - Powered by Cognero Page 25
SOLUTION:  
Substitute –1, 3 and 7 for x, y and z and simplify.
ANSWER:   with Linear Programming
3-3 Optimization
–5

54. 
SOLUTION:  
Substitute –1, 3 and 7 for x, y and z and simplify.
 

ANSWER:  
20

55. 
SOLUTION:  
Substitute –1, 3 and 7 for x, y and z and simplify.
 

ANSWER:  
15

eSolutions Manual - Powered by Cognero Page 26


3-4 Systems of Equations in Three Variables

Solve each system of equations.


Solve each system of equations.
1. 
1. 

SOLUTION:  
SOLUTION:  

 
  Eliminate one variable.
Eliminate one variable. Multiply the first equation by 3 and the second
Multiply the first equation by 3 and the second equation by 4 then add.
equation by 4 then add.  
 

 
  Solve the third and fourth equations.
Solve the third and fourth equations.  
 

 
  Substitute 5 for c in the third equation and solve for
Substitute 5 for c in the third equation and solve for a.
a.  
 

 
  Substitute –2 for a and 5 for c in the second
Substitute –2 for a and 5 for c in the second equation, and solve for b.
equation, and solve for b.  
 

 
  Therefore, the solution is (–2, –3, 5).
Therefore, the solution is (–2, –3, 5).
ANSWER:  
ANSWER:   (–2, –3, 5)
(–2, –3, 5)
eSolutions Manual - Powered by Cognero Page 1

2. 
2. 
 
Therefore, the solution is (–2, –3, 5).

ANSWER:  
3-4 Systems of Equations in Three Variables
(–2, –3, 5)  
Eliminate one variable.
Multiply the second equation by –2 and add with the
2.  third equation.
 

SOLUTION:  

 
Multiply the first equation by 2 and add with the
  second equation.
Multiply the third equation by 2 and with the second  
equation.
 

 
Solve the fifth and fourth equations.
   
Substitute 1 for z in the first equation and solve for y.
 

 
 
Substitute 3 for y in the fourth equation and solve for
Substitute –6 for y and 1 for z in the third equation, x.
and solve for x.
 
 

 
 
Substitute –4 for x and 3 for y in the second equation,
Therefore, the solution is (4, –6, 1). and solve for z.
ANSWER:    
(4, –6, 1)

3. 
 
SOLUTION:   Therefore, the solution is (–4, 3, 6).

ANSWER:  
(–4, 3, 6)

  4. 
Eliminate one variable.
Multiply the second equation by –2 and add with the
eSolutions Manual - Powered by Cognero Page 2
third equation. SOLUTION:  
 
(–4, 3, 6)

3-4 Systems of Equations in Three Variables  


4. 
Therefore, the solution is (–2, 2, –5).

ANSWER:  
SOLUTION:  
(–2, 2, –5)

5. 
 
Eliminate one variable. SOLUTION:  
Multiply the first equation by 2 and add with the
second equation.
 

 
Eliminate one variable.
Multiply the first equation by 2, multiply the second
equation by 3 and add.
   
Multiply the second equation by –3, multiply the third
equation by 2 and add.
 

 
Multiply the second equation by 2 and add with the
third equation.
 
 
To solve the fourth and fifth equations, add both
equations.
 
 
Multiply the first equation by –4, multiply the third
  equation by 3, and add.
Substitute –2 for r in the fifth equation and solve for  
t.
 

 
 
Since the equations 4, 5 and 6 are same, the system
Substitute –2 for r and –5 for t in the first equation,
of equations has an infinite number of solutions.
and solve for s.
  ANSWER:  
Infinite solutions

6. 
 
Therefore, the solution is (–2, 2, –5). SOLUTION:  
ANSWER:  
eSolutions Manual - Powered by Cognero Page 3
(–2, 2, –5)
6. 
3-4 Systems of Equations in Three Variables  
SOLUTION:   Therefore, the solution is (3, –4, 8).

ANSWER:  
(3, –4, 8)

7. DOWNLOADING Heather downloaded some


  television shows. A sitcom uses 0.3 gigabyte of
Eliminate one variable. memory; a drama, 0.6 gigabyte; and a talk show, 0.6
Multiply the second equation by 3 and add with the gigabyte. She downloaded 7 programs totaling 3.6
first equation. gigabytes. There were twice as many episodes of the
  drama as the sitcom.
a. Write a system of equations for the number of
episodes of each type of show.
b. How many episodes of each show did she
download?
SOLUTION:  
  a. Let s, t and d be the number of sitcoms, talk
Multiply the second equation by 8 and add with the shows and dramas respectively.
third equation. The system of equations is:
   

 
b. Solve .
   
Solve the fourth and fifth equations.
 

 
Substitute 2s for d in the first and third equation.
 
 
Substitute 3 for a in the fourth equation and solve for
c.
 

 
Solve the fourth and fifth equations.
 
 
Substitute 3 for a and 8 for c in the second equation,
and solve for b.
 
 
Substitute 2 for s in the second equation and solve for
d.
 
 
Therefore, the solution is (3, –4, 8).
eSolutions Manual - Powered by Cognero   Page 4
ANSWER:   Substitute 2 for s and 4 for d in the first equation and
(3, –4, 8) solve for t.
Substitute 2 for s in the second equation and solve for
d.
 
3-4 Systems of Equations in Three Variables
 
  Substitute –4 for z in the fourth equation and solve
Substitute 2 for s and 4 for d in the first equation and for x.
solve for t.  
 

 
Therefore, she downloaded 2 episodes of sitcom, 4
episodes of drama and 1 episode of talk show.  
Substitute –8 and –4 for x and z in the first equation
ANSWER:   and solve for y.
a.  
b. 2 sitcoms, 4 dramas, 1 talk show

Solve each system of equations.

8. 
 
Therefore, the solution is (–8, 4, –4).
SOLUTION:  
ANSWER:  
(–8, 4, –4)

  9. 
Eliminate one variable.
Multiply the first equation by –4 and add with the
second equation. SOLUTION:  
 

 
 
Eliminate one variable.
Multiply the first equation by 3 and add with the third
Multiply the third equation by 4 and add the first
equation.
equation with that.
 
 

 
 
Solve the fourth and fifth equations.
Multiply the third equation by –3 and add the second
  equation with that.
 

 
 
Substitute –4 for z in the fourth equation and solve
eSolutions Manual - Powered by Cognero Page 5
Solve the fourth and fifth equations.
for x.
   
 

3-4 Systems of Equations in Three Variables


 
  Multiply the first equation by 3 and add with the third
Solve the fourth and fifth equations. equation.
   

 
  Solve the fourth and fifth equations.
Substitute –2 for b in the fifth equation and solve for  
c.
 

   
Substitute –2 for b and –4 for c in the third equation Substitute –3 for y in the fifth equation and solve for
and, solve for a. z.
   

   
Therefore, the solution is (–3, –2, –4). Substitute –3 and 2 for y and z in the first equation
and solve for x.
ANSWER:    
(–3, –2, –4)

10. 
 
SOLUTION:   Therefore, the solution is (8, –3, 2).

ANSWER:  
(8, –3, 2)

  11. 
Eliminate one variable.
Multiply the first equation by –2 and add with the
second equation. SOLUTION:  
 

 
  Eliminate one variable.
Multiply the first equation by 3 and add with the third Multiply the first equation by –4 and add with the
equation. second equation.
eSolutions Manual - Powered by Cognero Page 6
   
 
Therefore, the solution is (–2, –1, 4).
 
3-4 Systems of Equations in Three Variables ANSWER:  
Eliminate one variable. (–2, –1, 4)
Multiply the first equation by –4 and add with the
second equation.
  12. 

SOLUTION:  

 
Multiply the third equation by 2, and add with the
second equation.
   
Eliminate one variable.
Multiply the first equation by 2 and add with the
second equation.
 
 
Solve the fourth and fifth equations
 

 
. Multiply the second equation by 2 and the third
equation by 4 then add.
 
 
Substitute –2 for r in the fifth equation and solve for
s.
 
 
Solve the fourth and the fifth equation.
 

   
Substitute –2 and –1 for r and s in the first equation This is a false statement. Therefore, there is no
and solve for t. solution.
 
ANSWER:  
No solution

  13. 
Therefore, the solution is (–2, –1, 4).

ANSWER:   SOLUTION:  
(–2, –1, 4)

12. 
 
Eliminate one variable.
SOLUTION:  
Add the first and the third equations.
eSolutions Manual - Powered by Cognero
  Page 7

 
This is a false statement. Therefore, there is no
solution.

ANSWER:  
3-4 Systems of Equations in Three Variables
No solution  
Eliminate one variable.
Multiply the second equation by –3 and add with the
13.  first equation.
 

SOLUTION:  

 
Multiply the second and third equation by 5 and –2
respectively and add.
   
Eliminate one variable.
Add the first and the third equations.
 
 

   
Multiply the first equation by 3 and add with the Solve the fourth and fifth equations.
second equation.  
 

   
Multiply the third equation by –3 and add with the Substitute 3 for y in the fourth equation and solve for
second equation. x.
   

 
  Substitute –1 and 3 for x and y in the first equation
Since the equations 4, 5 and 6 are same, the system and solve for z.
has an infinite number of solutions.  
ANSWER:  
Infinite solutions

14. 
 
Therefore, the solution is (–1, 3, 7).
SOLUTION:   ANSWER:  
(–1, 3, 7)

15. 
 
Eliminate one variable.
Multiply
eSolutions the
Manual secondby
- Powered equation
Cognero by –3 and add with the SOLUTION:   Page 8
first equation.
 
(–1, 3, 7)

15. Systems of Equations in Three Variables


3-4
 
Therefore, the solution is (–4, –1, 6).
SOLUTION:  
ANSWER:  
(–4, –1, 6)

  16. 
Eliminate one variable.
Multiply the first and second equation by 3 and 5 SOLUTION:  
respectively then add.
 

 
Eliminate one variable.
 
Multiply the first and second equation by 5 and 2
Multiply the second equation by 2 and add with the respectively then add.
third equation.
 
 

 
  Multiply the first equation by 4 and add with the third
Solve the fourth and fifth equations. equation.
   

 
  Solve the fourth and fifth equations.
Substitute –4 for x in the fifth equation and solve for  
z.
 

   
Substitute –4 and 6 for x and z in the first equation Substitute –8 for a in the fourth equation and solve
and solve for y. for b.
   

 
Substitute –8 and –7 for a and b in the third equation
and solve for c.
   
Therefore, the solution is (–4, –1, 6).
eSolutions Manual - Powered by Cognero Page 9
ANSWER:  
(–4, –1, 6)
 
Therefore, the solution is (–8, –7, –5).
 
Substituteof–8Equations
3-4 Systems and –7 forina Three
and b inVariables
the third equation ANSWER:  
and solve for c. (–8, –7, –5)
 

17. 

SOLUTION:  
 
Therefore, the solution is (–8, –7, –5).

ANSWER:  
(–8, –7, –5)
 
Eliminate one variable.
Multiply the first equation by 4 and add with the
17. 
second equation.
 
SOLUTION:  

 
  Multiply the first equation by 7 and add with the third
equation.
Eliminate one variable.
Multiply the first equation by 4 and add with the  
second equation.
 

   
Multiply the first equation by 7 and add with the third Solve the fourth and the fifth equation.
equation.  
 

 
This is a false statement. Therefore, there is no
solution.

ANSWER:  
No solution
 
Solve the fourth and the fifth equation.
 
18. 

  SOLUTION:  
This is a false statement. Therefore, there is no
solution.

ANSWER:  
No solution
 
eSolutions Manual - Powered by Cognero Page 10
Eliminate one variable.
Subtract the first equation from the third equation.
18. 
 
This is a false statement. Therefore, there is no  
solution. Therefore, the solution is (6, 3, –4)

ANSWER:  
3-4 Systems of Equations in Three Variables ANSWER:  
No solution (6, 3, –4)

18.  19. 

SOLUTION:   SOLUTION:  

   
Eliminate one variable. Eliminate one variable.
Subtract the first equation from the third equation. Multiply the first equation by –3 and add with the
  second equation.
 

 
Multiply the fourth equation by 4 and add with the  
second equation. Multiply the second equation by –4 and add with the
  third equation.
 

   
Substitute 3 for y in the fourth equation and solve for Multiply the second equation by –4 and the third
z. equation by 3 then add.
   

 
Substitute 3 for y in the first equation and solve for x.  
  Since the equations 4, 5 and 6 are same, the system
has an infinite number of solutions.

ANSWER:  
Infinite solutions
 
Therefore, the solution is (6, 3, –4) 20. CCSS SENSE-MAKING A friend e-mails you the
results of a recent high school swim meet. The e-
ANSWER:  
mail states that 24 individuals placed, earning a
(6, 3, –4) combined total of 53 points. First place earned 3
points, second place earned 2 points, and third place
earned 1 point. There were as many first-place
19.  finishers as second- and third-place finishers
combined.
a. Write a system of three equations that represents
eSolutions Manual - Powered by Cognero
SOLUTION:   Page 11
how many people finished in each place.
b. How many swimmers finished in first place, in
second place, and in third place?
mail states that 24 individuals placed, earning a  
combined total of 53 points. First place earned 3 7 simmers placed third, 5 simmers placed second,
points, second place earned 2 points, and third place and 12 simmers placed first.
earned 1 of
3-4 Systems point. There were
Equations as many
in Three first-place
Variables  
finishers as second- and third-place finishers c. The statement is false because when you solve for
combined. second place, you get a negative as an answer and
a. Write a system of three equations that represents you cannot have a negative person.
how many people finished in each place.
b. How many swimmers finished in first place, in ANSWER:  
second place, and in third place? a. x + y + z = 24, 3x + 2y + z = 53, x = y + z.
c. Suppose the e-mail had said that the athletes b. 7 swimmers placed third, 5 swimmers placed
scored a combined total of 47 points. Explain why second, and 12 swimmers placed first.
this statement is false and the solution is c. The statement is false because when you solve for
unreasonable. second place, you get a negative as an answer and
you cannot have a negative person.
SOLUTION:  
a. Let x, y, and z be the number of swimmers 21. AMUSEMENT PARKS Nick goes to the
finished in first place, in second place and in third amusement park to ride roller coasters, bumper cars,
place. and water slides. The wait for the roller coasters is 1
hour, the wait for the bumper cars is 20 minutes long,
and the wait for the water slides is only 15 minutes
long. Nick rode 10 total rides during his visit.
Because he enjoys roller coasters the most, the
  number of times he rode the roller coasters was the
b. Name the equations. sum of the times he rode the other two rides. If Nick
waited in line for a total of 6 hours and 20 minutes,
how many of each ride did he go on?
SOLUTION:  
  Let x, y and z be the number of raids in roller
Substitute x for y + z in the first equation and solve coaster, bumper car and water slide respectively.
for x. Nick rode 10 rides during his visit.
   

 
The number of times that Nick rode the roller coaster
is the sum of the times he rode the other two rides.
 
So:
Substitute 12 for x and 5 for y + z in the second
equation and solve for y.
   
He waited in line for a total of 6 hours 20 minutes.
 

 
Substitute x and y values in the first equation and
solve for z.  
  Substitute x for y + z in the first equation and solve
for x.
 
 
7 simmers placed third, 5 simmers placed second,
and 12 simmers placed first.
   
c. The statement is false because when you solve for Substitute 5 for x in the second and the third equation
second place, you get a negative as an answer and and simplify.
you cannot have a negative person.  
eSolutions Manual - Powered by Cognero Page 12
ANSWER:  
a. x + y + z = 24, 3x + 2y + z = 53, x = y + z.
pad replacement.
b. If Ramón wants his brake pads replaced and his 
  radiator flushed, how much should he plan to spend?
Substituteof5Equations
3-4 Systems for x in the in
second andVariables
Three the third equation
and simplify. SOLUTION:  
  a. Let x, y and z be the price for oil change, brake
pad replacement and radiator flush.

 
  Substitute 39.99 for x + y in the third equation and
Multiply the fifth equation by –3 and add with the solve for z.
fourth equation.  
 

 
Substitute 10 for z in the first equation and solve for
x.
 
 
Substitute 1 for y in the fifth equation and solve for z.
   
Substitute 19.99 for x in the second equation and
solve for y.
   
Nick rode the roller coaster, bumper cars and water
slides 5, 1 and 4 times respectively.

ANSWER:    
roller coasters: 5; bumper cars: 1; water slides: 4 Therefore, the cost for oil change is $19.99, the cost
for brake pad replacement is $20 and the cost for
22. BUSINESS Ramón usually gets one of the routine radiator flush is $10.
maintenance options at Annie’s Garage. Today  
however, he needs a different combination of work b. The cost for brake pad replacement and radiator
than what is listed. flush are $20 and $10. Therefore, he should spend
  $30.

ANSWER:  
a. oil change: $19.99; brake pad replacement: $20;
radiator flush: $10
b. $30

23. FINANCIAL LITERACY Kate invested $100,000


in three different accounts. If she invested $30,000
more in account A than account C and is expected to
earn $6300 in interest, how much did she invest in
  each account?
a. Assume that the price of an option is the same  
price as purchasing each item separately. Find the
prices for an oil change, a radiator flush, and a brake
pad replacement.
b. If Ramón wants his brake pads replaced and his 
radiator flushed, how much should he plan to spend?
SOLUTION:   SOLUTION:  
a. Let x, y and z be the price for oil change, brake Let a, b and c be the amount invested in the Account
pad Manual
eSolutions replacement andbyradiator
- Powered Cognero flush. A, B and C respectively. Page 13
 
 

3-4 Systems of Equations in Three Variables


SOLUTION:  
Let a, b and c be the amount invested in the Account  
A, B and C respectively. Therefore, she invested $55,000, $20,000 and $25,000
  in the account A, B and C respectively.

ANSWER:  
  A: $55,000; B: $20,000; C: $25,000
Kate invested $30,000 more in account A than
account C. 24. CCSS REASONING Write a system of equations
  to represent the three rows of figures below. Use the
Therefore, system to find the number of red triangles that will
  balance one green circle.
Substitute c + 30000 for a in the first equation and  
simplify.
 

 
Total interest amount is $6300. That is,
  SOLUTION:  
  . t + c = s, p + t = c, 2s = 3p
where t represents triangle, c represents circle, s
  represents square, and p represents pentagon; 5 red
Substitute c + 30000 for a and simplify. triangles
 
ANSWER:  
t + c = s, p + t = c, 2s = 3p
where t represents triangle, c represents circle, s
  represents square, and p represents pentagon; 5 red
Solve the third and fourth equations. triangles
 
25. CHALLENGE The general form of an equation for
a parabola is  where (x, y) is a point
on the parabola. If three points on a parabola are (2,
–10), (–5, –101), and (6, –90), determine the values
of a, b, and c and write the general form of the
  equation.
Substitute 25000 for c in the second equation and SOLUTION:  
solve for a. Substitute the points (2, –10), (–5, –101), and (6, –
 
90) in the equation .
 
 
Substitute 25000 for c in the third equation and solve
for b.
 

 
Therefore, she invested $55,000, $20,000 and $25,000  
eSolutions Manual - Powered by Cognero Solve the equations 1, 2 and 3. Page 14
in the account A, B and C respectively.
 
ANSWER:  
Therefore, the equation of the parabola is y = –3x +
4x – 6.

ANSWER:  
3-4 Systems of Equations in Three Variables

 
Solve the equations 1, 2 and 3. 26. PROOF Consider the following system and prove
  that if b = c = –a, then ty = a.
 
rx + ty + vz = a
rx – ty + vz = b
  rx + ty – vz = c
Solve the fourth and fifth equations.
  SOLUTION:  
First add a and b to eliminate ty.
 

 
Substitute –3 for a in the fourth equation and solve
 
for b.
Use this equation to prove that a = ty.
 
 

 
 
 
Substitute –3 and 4 for a and b in the first equation. ANSWER:  
 

 
The value of a, b and c are –3, 4 and –6
respectively.
2
Therefore, the equation of the parabola is y = –3x +
4x – 6. 27. OPEN ENDED Write a system of three linear
equations that has a solution of (–5,–2, 6). Show that
ANSWER:   the ordered triple satisfies all three equations.
SOLUTION:  
Sample answer:
26. PROOF Consider the following system and prove
that if b = c = –a, then ty = a.
 
rx + ty + vz = a
rx – ty + vz = b
rx + ty – vz = c
SOLUTION:  
First add a and b to eliminate ty.
 

eSolutions Manual - Powered by Cognero Page 15

 
3-4 Systems of Equations in Three Variables

28. REASONING Use the diagram below of the


solution of systems of equations to consider a system
on inequalities in three variables. Describe the
solution of such a system.

ANSWER:  
Sample answer:
SOLUTION:  
Sample answer:  The solution of an inequality in 3 
variables would be the region of space on one side or
the other of a plane, which the plane included if the
inequality is  or  . The solution of a system of
inequalities in 3 variables would be the intersection of
the regions of space that are solution to the individual
inequalities in the system.

ANSWER:  
Sample answer:  The solution of an inequality in 3 
variables would be the region of space on one side or
the other of a plane, which the plane included if the
inequality is  or  . The solution of a system of
inequalities in 3 variables would be the intersection of
the regions of space that are solution to the individual
inequalities in the system.

29. WRITING IN MATH Use your knowledge of


solving a system of three linear equations with three
variables to explain how to solve a system of four
equations with four variables.
SOLUTION:  
28. REASONING Use the diagram below of the
solution of systems of equations to consider a system Sample answer: First, combine two of the original
on inequalities in three variables. Describe the equations using elimination to form a new equation
solution of such a system. with three variables. Next, combine a different pair
of the original equations using elimination to eliminate
the same variable and form a second equation with
three variables. Do the same thing with a third pair of
the original equations. You now have a system of
three equations with three variables. Follow the same
procedure you learned in this section. Once you find
the three variables, you need to use them to find the
eSolutions Manual - Powered by Cognero
eliminated variable. Page 16

ANSWER:  
Sample answer: First, combine two of the original
the other of a plane, which the plane included if the
inequality is  or  . The solution of a system of  
inequalities in 3 variables would be the intersection of Eliminate one variable.
the regions
3-4 Systems of of space thatin
Equations areThree
solution to the individual
Variables Multiply the first equation by 2 and with the second
inequalities in the system. equation.
 
29. WRITING IN MATH Use your knowledge of
solving a system of three linear equations with three
variables to explain how to solve a system of four
equations with four variables.
SOLUTION:    
Multiply the second equation by 2 and add with the
Sample answer: First, combine two of the original
third equation.
equations using elimination to form a new equation
with three variables. Next, combine a different pair  
of the original equations using elimination to eliminate
the same variable and form a second equation with
three variables. Do the same thing with a third pair of
the original equations. You now have a system of
three equations with three variables. Follow the same  
procedure you learned in this section. Once you find Solve the fourth and fifth equations.
the three variables, you need to use them to find the  
eliminated variable.

ANSWER:  
Sample answer: First, combine two of the original
equations using elimination to form a new equation
with three variables. Next, combine a different pair
 
of the original equations using elimination to eliminate
the same variable and form a second equation with Substitute 2 for x in the fourth equation and solve for
three variables. Do the same thing with a third pair of y.
the original equations. You now have a system of  
three equations with three variables. Follow the same
procedure you learned in this section. Once you find
the three variables, you need to use them to find the
eliminated variable.
 
30. What is the solution of the system of equations Substitute 2 and 5 for x and y in the first equation and
shown below? solve for z.
 

 
A (0, 3, 3) The solution is (2, 5, 3).
B (2, 5, 3) Option B is the correct answer.
C no solution
D infinitely many solutions ANSWER:  
B
SOLUTION:  
31. ACT/SAT The graph shows which system of
equations?

 
Eliminate one variable.
Multiply the first equation by 2 and with the second
equation.
 
eSolutions Manual - Powered by Cognero A                   D  Page 17
First system of equations satisfies the point (3, –2).
Therefore, option A is the correct answer.

3-4 Systems of Equations in Three Variables ANSWER:  


A

A                   D  32. EXTENDED RESPONSE Use the graph to find


the solution of the systems of equations. Describe
one way to check the solution.

B                   E 
SOLUTION:  
The lines intersect at (2, 8). So the solution is (2, 8).
Sample answer: You can substitute (2, 8) into each
of the equations and make sure the equations are
true.

ANSWER:  
(2, 8); Sample answer: You can substitute (2, 8) into
C each of the equations and make sure the equations
are true.

33. Which of the following represents a correct


procedure for solving each equation?
SOLUTION:  
The lines intersect at (3, –2). Substitute the point in
each system of equations.
  F

 
First system of equations satisfies the point (3, –2).  
Therefore, option A is the correct answer.

ANSWER:   J
A

32. EXTENDED RESPONSE Use the graph to find  


the solution of the systems of equations. Describe
one way to check the solution. SOLUTION:  
eSolutions Manual - Powered by Cognero Page 18
Substitute for x in the equation .
 
J  
Therefore, option J is the correct answer.

3-4 Systems
  of Equations in Three Variables ANSWER:  
J
SOLUTION:  
A feasible region has vertices at (–3, 2), (1, 3),
Substitute for x in the equation . (6, 1), and (2, –2). Find the maximum and
  minimum values of each function.
34. 

SOLUTION:  
Substitute the points (–3, 2), (1, 3), (6, 1), and (2, –
2) in the function f (x, y) = 2x – y.
 
 
Substitute 20 for x in the equation .
 

 
The maximum value is 11 and the minimum value is
  –8.
Substitute 6 for x in the equation .
ANSWER:  
 
11; –8

35. 

SOLUTION:  
  Substitute the points (–3, 2), (1, 3), (6, 1), and (2, –
Substitute 2 for x in the equation . 2) in the function f (x, y) = x + 5y.
 
 

 
Therefore, option J is the correct answer.  
The maximum value is 16 and the minimum value is
ANSWER:   –8.
J
ANSWER:  
A feasible region has vertices at (–3, 2), (1, 3), 16; –8
(6, 1), and (2, –2). Find the maximum and
minimum values of each function. 36. 
34. 
SOLUTION:  
SOLUTION:   Substitute the points (–3, 2), (1, 3), (6, 1), and (2, –
Substitute the points (–3, 2), (1, 3), (6, 1), and (2, – 2) in the function f (x, y) = y – 4x.
2) in the function f (x, y) = 2x – y.  
 

eSolutions Manual - Powered by Cognero Page 19


The maximum value is 16 and the minimum value is The maximum value is 9 and the minimum value is –
–8. 8.

ANSWER:  
3-4 Systems of Equations in Three Variables ANSWER:  
16; –8 9; –8

36.  38. SKI CLUB The ski club’s budget for the year is
$4250. They are able to find skis for $75 per pair and
SOLUTION:   boots for $40 per pair. They know they should buy
Substitute the points (–3, 2), (1, 3), (6, 1), and (2, – more boots than skis because the skis are adjustable
2) in the function f (x, y) = y – 4x. to several sizes of boots.
  a. Give an example of three different purchases that
the ski club can make.
b. Suppose the ski club wants to spend all of its
budget. What combination of skis and boots should
they buy? Explain.
SOLUTION:  
a. Sample answer: 40 boots, 35 skis; 45 boots, 32
  skis; 50 boots, 30 skis
The maximum value is 14 and the minimum value is b. 50 boots and 30 skis cost exactly $4250.
–23.
ANSWER:  
ANSWER:   a. Sample answer: 40 boots, 35 skis; 45 boots, 32
14; –23 skis; 50 boots, 30 skis
b. 50 boots and 30 skis cost exactly $4250.
37. 
Solve each system of equations.
SOLUTION:  
Substitute the points (–3, 2), (1, 3), (6, 1), and (2, – 39. 
2) in the function f (x, y) = –x + 3y.
  SOLUTION:  

 
Substitute y + 5 for x in the second equation and
solve for y.
 
 
The maximum value is 9 and the minimum value is –
8.

ANSWER:  
9; –8
 
38. SKI CLUB The ski club’s budget for the year is Substitute 1 for y in the first equation and solve for x.
$4250. They are able to find skis for $75 per pair and  
boots for $40 per pair. They know they should buy
more boots than skis because the skis are adjustable
to several sizes of boots.  
a. Give an example of three different purchases that Therefore, the solution is (6, 1).
the ski club can make.
b. Suppose the ski club wants to spend all of its ANSWER:  
budget. What combination of skis and boots should (6, 1)
they buy? Explain.
SOLUTION:   40. 
a. Sample answer: 40 boots, 35 skis; 45 boots, 32
eSolutions Manual - Powered by Cognero Page 20
skis; 50 boots, 30 skis
SOLUTION:  
b. 50 boots and 30 skis cost exactly $4250.
 
 
Therefore, the solution is (6, 1).
Therefore, the solution is (3, 4).
ANSWER:  
3-4 Systems ANSWER:  
(6, 1) of Equations in Three Variables
(3, 4)

40.  41. 

SOLUTION:   SOLUTION:  

   
Add both the equations. Multiply the first and second equation by –4 and 5
  respectively then add.
 

 
Substitute 3 for x in the second equation and solve
for y.
 
 
Substitute –5 for y in the first equation and solve for
x.
 

 
Therefore, the solution is (3, 4).

ANSWER:  
(3, 4)  
Therefore, the solution is (8, –5).

ANSWER:  
(8, –5)

42. 

SOLUTION:  

 
Substitute x – 7 for y in the second equation and
solve for x.
 

 
Substitute 9 for x in the first equation and solve for y.
 

eSolutions Manual - Powered by Cognero Page 21


 
Therefore, the solution is (9, 2).
 
Therefore, the solution is (8, –5).

ANSWER:  
3-4 Systems of Equations in Three Variables
(8, –5)

42. 

SOLUTION:  

 
Substitute x – 7 for y in the second equation and
solve for x.
 

 
Substitute 9 for x in the first equation and solve for y.
 

 
Therefore, the solution is (9, 2).

ANSWER:  
(9, 2)

eSolutions Manual - Powered by Cognero Page 22


However, this value is irrelevant since it is the sum of
5 different types of data.
d. Sample answer: The sums are 48, 45, 53, 91, and
3-5 Operations with Matrices 131. These values are irrelevant since they are the
sums of 2 different types of data.

1. CCSS MODELING Use the table that shows the Perform the indicated operations. If the matrix
city and highway gas mileage of five different types does not exist, write impossible.
of vehicles. 2. 
 
SOLUTION:  
Add corresponding elements.

Simplify.
 
a. Organize the gas mileages in a matrix.
b. Which type of vehicle has the best gas mileage? ANSWER:  
c. Add the elements of each row and interpret the
results.
d. Add the elements of each column and interpret the
results. 3. 
SOLUTION:  
SOLUTION:  
a. Write the city gas mileage in the first row and the
The dimensions of the matrixes are not equal. So,
highway gas mileage in the second row.
impossible to add the matrices.
  ANSWER:  
b. APV gives more gas mileage than others. impossible
c. Sample answer: City: The sum is 168. However,
this value is irrelevant since it is the sum of 5 4. 
different types of data. Highway: The sum is 200.
However, this value is irrelevant since it is the sum of
5 different types of data. SOLUTION:  
d. Sample answer: The sums are 48, 45, 53, 91, and Subtract corresponding elements.
131. These values are irrelevant since they are the
sums of 2 different types of data.

ANSWER:    
Simplify.
a.

b. APV
c. Sample answer: City: The sum is 168. However, ANSWER:  
this value is irrelevant since it is the sum of 5
different types of data. Highway: The sum is 200.
However, this value is irrelevant since it is the sum of
5 different types of data.
d. Sample answer: The sums are 48, 45, 53, 91, and
131. These values are irrelevant since they are the 5. 
sums of 2 different types of data.
SOLUTION:  
Perform the indicated operations. If the matrix Subtract corresponding elements.
does not exist, write impossible.
2. 

SOLUTION:    
Add corresponding elements. Simplify.
eSolutions Manual - Powered by Cognero Page 1
Simplify.
ANSWER:  

3-5 Operations with Matrices

5. 
7. 

SOLUTION:  
Subtract corresponding elements. SOLUTION:  
Distribute the scalar.

 
Simplify.
 
Multiply.

ANSWER:  

ANSWER:  
Perform the indicated operations. If the matrix
does not exist, write impossible.

6. 
Use matrices A , B, C, and D to find the
SOLUTION:   following.
Distribute the scalar.

8. 4B − 2A
 
Multiply. SOLUTION:  
Distribute the scalar in each matrix.

 
ANSWER:   Multiply.

 
Subtract the corresponding elements.

7. 
 
Simplify.
SOLUTION:  
Distribute the scalar.

eSolutions Manual - Powered by Cognero ANSWER:   Page 2


ANSWER:  
ANSWER:  

3-5 Operations with Matrices

Use matrices A , B, C, and D to find the 9. −8C + 3A


following.
SOLUTION:  
Distribute the scalar in each matrix.

 
8. 4B − 2A
Multiply.
SOLUTION:  
Distribute the scalar in each matrix.
 
Add the corresponding elements.
 
Multiply.
 
Simplify.
 
Subtract the corresponding elements.

ANSWER:  

 
Simplify.
10. −5B − 2D
SOLUTION:  
ANSWER:   The dimensions of the matrix B and D are not equal.
So, it is impossible.

ANSWER:  
Impossible
9. −8C + 3A
11. −4C − 5B
SOLUTION:  
SOLUTION:  
Distribute the scalar in each matrix.
Distribute the scalar in each matrix.

 
 
Multiply.
Multiply.

 
 
Add the corresponding elements.
Subtract corresponding elements.

 
Simplify.  
Simplify.
eSolutions Manual - Powered by Cognero Page 3
The dimensions of the matrix B and D are not equal.
So, it is impossible.
Test 2:
ANSWER:  with Matrices
3-5 Operations
Impossible
b.
11. −4C − 5B
SOLUTION:  
Distribute the scalar in each matrix.
c.

 
Multiply.

ANSWER:  
 
Subtract corresponding elements. a. Test 1:

      Test 2: 
Simplify.

b.
ANSWER:  

c.

12. GRADES Geraldo, Olivia, and Nikki have had two


tests in their math class. The table shows the test 13. SHOES A consumer service company rated several
grades for each student. pairs of shoes by cost, level of comfort, look, and
  longevity using a scale of 1–5, with 1 being low and 5
being high.
 

 
a. Write a matrix for the information.
b. Find the sum of the scores from the two tests
expressed as a matrix.  
c. Express the difference in scores from test 1 to test a. Write a 4 × 4 matrix to organize this information.
2 as a matrix. b. Which shoe would you buy based on this
SOLUTION:   information, and why?
c. Would finding the sum of the rows or columns
provide any useful information? Explain your
a. Test 1: reasoning.
SOLUTION:  

Test 2:
a.
b.
eSolutions Manual - Powered by Cognero Page 4
b. Sample answer: Brand C; it was given the highest
rating possible for cost and comfort, and a high rating
for looks, and it will last a fairly long time.
rating possible for cost and comfort, and a high rating
for looks, and it will last a fairly long time.
c. Sample answer: Yes; finding the sum of the rows
c.
3-5 Operations with Matrices and then calculating the average will provide an easy
way to compare the data.

13. SHOES A consumer service company rated several Perform the indicated operations. If the matrix
pairs of shoes by cost, level of comfort, look, and does not exist, write impossible.
longevity using a scale of 1–5, with 1 being low and 5
being high. 14. 
 
SOLUTION:  
Add corresponding elements.

 
  Simplify.
a. Write a 4 × 4 matrix to organize this information.
b. Which shoe would you buy based on this
information, and why?
c. Would finding the sum of the rows or columns ANSWER:  
provide any useful information? Explain your
reasoning.
SOLUTION:  

15.  `
a.
SOLUTION:  
The dimensions of the matrixes are not equal. So, we
b. Sample answer: Brand C; it was given the highest
cannot add the matrix.
rating possible for cost and comfort, and a high rating
Impossible.
for looks, and it will last a fairly long time.
c. Sample answer: Yes; finding the sum of the rows ANSWER:  
and then calculating the average will provide an easy impossible
way to compare the data.
16. BUSINESS The drink menu from a fast-food
ANSWER:  
restaurant is shown at the right. The store owner has
decided that all of the prices must be increased by
10%.
a.  

b. Sample answer: Brand C; it was given the highest


rating possible for cost and comfort, and a high rating
for looks, and it will last a fairly long time.
c. Sample answer: Yes; finding the sum of the rows
and then calculating the average will provide an easy  
way to compare the data. a. Write matrix C to represent the current prices.
b. What scalar can be used to determine a matrix N
Perform the indicated operations. If the matrix to represent the new prices?
does not exist, write impossible. c. Find N.
d. What is N − C? What does this represent in this
14. 
situation?

SOLUTION:   SOLUTION:  
AddManual
eSolutions corresponding elements.
- Powered by Cognero Page 5

a.
to represent the new prices? d.
c. Find N.
d. What is N − C? What does this represent in this
3-5 Operations
situation? with Matrices Sample answer: this matrix represents the price
increases for each item.
SOLUTION:  
Use matrices A , B, C, and D to find the
following.
a.
17. 
SOLUTION:  
 
b. The cost is increased by 10%. That is the new Distribute the scalar in each matrix.
price is 110% of old price.
110% = 1.1
So, multiply 1.1 with the matrix to determine the new  
price. Multiply.
 
c. Multiply the matrix by 1.1.
   
Add the corresponding elements.

 
 d. Subtract the matrix C from matrix N.
 
Simplify.

 
Sample answer: this matrix represents the price
increases for each item. ANSWER:  

ANSWER:  

a. 18. 
SOLUTION:  
Distribute the scalar in each matrix.
b. 1.1

 
c. Multiply.

 
Subtract the corresponding elements.
d.  

Sample answer: this matrix represents the price  


increases for each item. Simplify.
Use matrices A , B, C, and D to find the
following.
 
17. 
ANSWER:  
SOLUTION:  
Distribute
eSolutions Manual the scalarbyinCognero
- Powered each matrix. Page 6

19. 
equal.
ANSWER:   So, it is impossible.

3-5 Operations with Matrices ANSWER:  


impossible

18.  20. 
SOLUTION:   SOLUTION:  
Distribute the scalar in each matrix. Distribute the scalar in each matrix.

 
 
Multiply.
Multiply.

 
Subtract the corresponding elements.  
  Subtract the corresponding elements.

   
Simplify. Simplify.

 
ANSWER:  
ANSWER:  

21. CCSS MODELING Library A has 10,000 novels,


19.  5000 biographies, and 5000 children’s books. Library
B has 15,000 novels, 10,000 biographies, and 2500
SOLUTION:   children’s books. Library C has 4000 novels, 700
The dimensions of the matrix C and matrix A are not biographies, and 800 children’s books.
equal. a. Express each library’s number of books as a
So, it is impossible. matrix. Label the matrices A, B, and C.
ANSWER:   b. Find the total number of each type of book in all 3
libraries. Express as a matrix.
impossible
c. How many more books of each type does Library
20.  A have than Library C?
d. Find A + B. Does the matrix have meaning in this
SOLUTION:   situation? Explain.
Distribute the scalar in each matrix.
SOLUTION:  

  a. Library A: ; Library B: ;
Multiply.

Library C:
 
Subtract the corresponding elements.
 
b. Add all matrixes.
 
Simplify.
eSolutions Manual - Powered by Cognero Page 7

 
c. Subtract the matrix C from the matrix A.
Library C: d. ; Sample answer: The sum represents
3-5 Operations with Matrices
  the combined size of the two libraries.
b. Add all matrixes.
Perform the indicated operations. If the matrix
does not exist, write impossible.

  22. 
c. Subtract the matrix C from the matrix A. SOLUTION:  
Distribute the scalar in each matrix.

 
 
Multiply.

d.
 
Sample answer: The sum represents the combined Subtract the corresponding elements.
size of the two libraries.

ANSWER:  
 
Simplify.
a. Library A: ; Library B: ;

ANSWER:  
Library C:

b.

23. 
c. SOLUTION:  
Distribute the scalar in each matrix.

d. ; Sample answer: The sum represents


 
Multiply.
the combined size of the two libraries.

Perform the indicated operations. If the matrix


does not exist, write impossible.
 
Add the corresponding elements.
22. 
SOLUTION:  
Distribute the scalar in each matrix.
 
Simplify.
 
Multiply.
eSolutions Manual - Powered by Cognero Page 8

ANSWER:  
 
ANSWER:  
ANSWER:  

3-5 Operations with Matrices

25. 
23. 
SOLUTION:   SOLUTION:  
Distribute the scalar in each matrix.

 
Multiply. ANSWER:  

 
Add the corresponding elements.
26. 

SOLUTION:  
 
Simplify.

ANSWER:  

ANSWER:  
24. 

SOLUTION:  

27. 

SOLUTION:  

ANSWER:  

25. 

SOLUTION:  
eSolutions Manual - Powered by Cognero Page 9
ANSWER:  

3-5 Operations with Matrices

 
For 2007 Snowfall:
27. 

SOLUTION:  

 
b. Subtract the first matrix from the second matrix.

The matrix represents the difference from normal for


each city and month.
 
c. A negative number means the city was below
normal on snowfall for the month; a positive number
means the city’s snowfall was above normal for the
month.
ANSWER:   Sample answer: All the cities had below normal
snowfall in January. Grand Rapids and Buffalo had
snowfall well above normal in February.

ANSWER:  
28. WEATHER The table shows snowfall in inches.
 

a.  

 
a. Express the normal snowfall data and the 2007
data in two 4 × 3 matrices.
b. Subtract the matrix of normal data from the matrix
of 2007 data. What does the difference represent in
the context of the situation?  
c. Explain the meaning of positive and negative
numbers in the difference matrix. What trends do
you see in the data? b. ;

SOLUTION:  
a. For Normal snowfall:  
The matrix represents the difference from normal for
each city and month.
c. A negative number means the city was below
normal on snowfall for the month; a positive number
means the city’s snowfall was above normal for the
month. Sample answer: All the cities had below
normal snowfall in January. Grand Rapids and
  Buffalo had snowfall well above normal in February.
For 2007
eSolutions ManualSnowfall:
- Powered by Cognero Page 10

29. CCSS MODELING The table shows some of the


world, Olympic and American women’s freestyle
each city and month. 800-m, it was an American who set the world
c. A negative number means the city was below record.
normal on snowfall for the month; a positive number c. 50-m and 100-m
means the city’s
3-5 Operations withsnowfall
Matriceswas above normal for the
month. Sample answer: All the cities had below 30. MULTIPLE REPRESENTATIONS In this
normal snowfall in January. Grand Rapids and problem, you will investigate using matrices to
Buffalo had snowfall well above normal in February. represent transformations.
a. ALGEBRAIC The matrix
29. CCSS MODELING The table shows some of the
world, Olympic and American women’s freestyle represents a triangle with vertices at
swimming records.
  (−3, 8), (−4, 6), and (1, 0). Write a matrix to
represent ΔABC
 

 
a. Find the difference between the American and
World records expressed as a column matrix.
b. What is the meaning of each row in the column?
c. In which events were the fastest times set at the  
Olympics? b. GEOMETRIC Multiply the vertex matrix you
wrote by 2. Then graph the figure represented by the
SOLUTION:  
new matrix.
c. ANALYTICAL How do the figures compare?
Make a conjecture about the result of multiplying the
a. matrix by 0.5. Verify your conjecture.
SOLUTION:  
b. In the 50-m, the fastest American time is 0.5 a. The vertices of the triangle ABC is (4, 2), (–1, –1)
second behind the world record. In the 100-m, the and (–3, 1).
fastest American time is 0.47 second behind the Therefore, the matrix represents the triangle ABC is
world record. In the 200-m, the fastest American
time is 0.87 second behind the world record. In the .
800-m, it was an American who set the world
record. b. Multiply the matrix by 2.
c. 50-m and 100-m  

ANSWER:  

a.
 
Plot the points (8, 4) , (–2, –2) and (–6, 2) on
b. In the 50-m, the fastest American time is 0.5 coordinate plane and connect them.
second behind the world record. In the 100-m, the  
fastest American time is 0.47 second behind the
world record. In the 200-m, the fastest American
time is 0.87 second behind the world record. In the
800-m, it was an American who set the world
record.
c. 50-m and 100-m

30. MULTIPLE REPRESENTATIONS In this


problem, you will investigate using matrices to
represent transformations.
eSolutions Manual - Powered by Cognero Page 11
a. ALGEBRAIC The matrix  
represents a triangle with vertices at c. The new triangle is similar to the original, with
c. The new triangle is similar to the original, with
  sides twice as long as the original. The result of
Plot the points
3-5 Operations (8, 4)
with , (–2, –2) and (–6, 2) on
Matrices multiplying the matrix by 0.5 would be a figure similar
coordinate plane and connect them. to the original with sides half as long.
 
31. PROOF Prove that matrix addition is commutative
for 2 × 2 matrices.
SOLUTION:  
To show that the Commutative Property of Matrix
Addition is true for 2 × 2 matrices, let

 and   Show that A + B = B

+ A.
 
c. The new triangle is similar to the original, with
sides twice as long as the original. The result of
multiplying the matrix by 0.5 would be a figure similar
to the original with sides half as long.

ANSWER:  

a.
ANSWER:  
b. To show that the Commutative Property of Matrix
Addition is true for 2 × 2 matrices, let

 and   Show that A + B = B

+ A.

c. The new triangle is similar to the original, with


sides twice as long as the original. The result of
multiplying the matrix by 0.5 would be a figure similar
to the original with sides half as long. 32. PROOF Prove that matrix addition is associative for
2 × 2 matrices.
31. PROOF Prove that matrix addition is commutative
SOLUTION:  
for 2 × 2 matrices.
SOLUTION:  
To show that the Commutative Property of Matrix
Addition is true for 2 × 2 matrices, let

 and   Show that A + B = B

+ A.

ANSWER:  
eSolutions Manual - Powered by Cognero Page 12
3-5 Operations with Matrices

32. PROOF Prove that matrix addition is associative for 33. CHALLENGE Find the elements of C if:
2 × 2 matrices.
SOLUTION:  
SOLUTION:  

ANSWER:    

33. CHALLENGE Find the elements of C if:

ANSWER:  
SOLUTION:  

34. REASONING Determine whether each statement


is sometimes, always, or never true for matrices A
and B. Explain your reasoning.
a. If A + B exists, then A − B exists.
b. If k is a real number, then kA and kB exist.
c. If A − B does not exist, then B − A does not exist.
d. If A and B have the same number of elements,
  then A + B exists.
e . If kA exists and kB exists, then kA + kB exists.
SOLUTION:  
a. Always; if A + B exists, A and B have the same
dimensions. If A and B have the same dimensions,
then A − B exists.
b. Always; if k is a real number, then by the
definition of scalar multiplication,
eSolutions Manual - Powered by Cognero Page 13

c.
dimensions, then A + B does not exist.
ANSWER:   d. Sometimes; matrices must have the same
dimensions for their sum to exist.
3-5 Operations with Matrices e . Sometimes; matrices must have the same
dimensions for their sum to exist.

34. REASONING Determine whether each statement 35. OPEN ENDED Give an example of matrices A and
is sometimes, always, or never true for matrices A
and B. Explain your reasoning. B if
a. If A + B exists, then A − B exists.
b. If k is a real number, then kA and kB exist. SOLUTION:  
c. If A − B does not exist, then B − A does not exist. Sample answer: 
d. If A and B have the same number of elements, Use the "guess and check" problem solving strategy
then A + B exists. to find values for A and B that satisfy the given
e . If kA exists and kB exists, then kA + kB exists. equation. Check each element one at a time.
4B - 3A = -6. When B = 3 and A = 6, this equation is
SOLUTION:   true.
a. Always; if A + B exists, A and B have the same 4B - 3A = 5. When B = 2 and A = 1, this equation is
dimensions. If A and B have the same dimensions, true.
then A − B exists. 4B - 3A = -2. When B = 4 and A = 6, this equation is
b. Always; if k is a real number, then by the true.
definition of scalar multiplication, 4B - 3A = -1. When B = 2 and A = 3, this equation is
true.
Organize this into matrices.

c. Always; if A – B does not exist, then A and B must


have different dimensions. If A and B have different
dimensions, then A + B does not exist. ANSWER:  
d. Sometimes; matrices must have the same
dimensions for their sum to exist. Sample Answer:
e . Sometimes; matrices must have the same
dimensions for their sum to exist.
36. WRITING IN MATH Explain how to find 4D −
ANSWER:   3C for two given matrices, C and D with the same
a. Always; if A + B exists, A and B have the same dimensions.
dimensions. If A and B have the same dimensions,  
then A − B exists. SOLUTION:  
b. Always; if k is a real number, then by the Sample answer: First, multiply every element in D by
definition of scalar multiplication, 4. Then, multiply every element in C by 3. Finally,
subtract the elements in 3C from the corresponding
elements in 4D. The result is a matrix equivalent to
c. Always; if A – B does not exist, then A and B must 4D − 3C.
have different dimensions. If A and B have different ANSWER:  
dimensions, then A + B does not exist. Sample answer: First, multiply every element in D by
d. Sometimes; matrices must have the same 4. Then, multiply every element in C by 3. Finally,
dimensions for their sum to exist. subtract the elements in 3C from the corresponding
e . Sometimes; matrices must have the same elements in 4D. The result is a matrix equivalent to
dimensions for their sum to exist. 4D − 3C.
35. OPEN ENDED Give an example of matrices A and 37. What is the solution of the system of equations?
B if

A (–0.912, –1.338)
SOLUTION:  
B (0.912, –3.162)
Sample answer: 
C (–2, 0.25)
Use the "guess and check" problem solving strategy
to find
eSolutions values
Manual for Abyand
- Powered B that satisfy the given
Cognero
D (–2, –4.25) Page 14
equation. Check each element one at a time. SOLUTION:  
4B - 3A = -6. When B = 3 and A = 6, this equation is
Multiply the second equation by 4 and add with the
Sample answer: First, multiply every element in D by
4. Then, multiply every element in C by 3. Finally, ANSWER:  
subtract the elements in 3C from the corresponding
elements in 4D.
3-5 Operations withThe result is a matrix equivalent to
Matrices
4D − 3C.

37. What is the solution of the system of equations? 39. SAT/ACT Solve for x and y.

A (–0.912, –1.338) F x = −5, y = 7


B (0.912, –3.162) G x = 7, y = 3
C (–2, 0.25) H x = 7, y = 5
D (–2, –4.25) J x = 5, y = 7
SOLUTION:   K x = - 5, y = 3
Multiply the second equation by 4 and add with the SOLUTION:  
first equation. Solve the second equation for x.
   

 
  Substitute –5 for x in the first equation and solve for
Substitute –2 for p in the second equation and solve y.
for q.  
 

   
Option C is the correct answer. The solution is (–5, 7). Therefore, option F is the
correct answer.
ANSWER:  
C ANSWER:  
F
38. SHORT RESPONSE Find A + B if
40. PROBABILITY A local pizzeria offers 5 different
meat toppings and 6 different vegetable toppings.
You decide to get two vegetable toppings and one
SOLUTION:   meat topping. How many different types of pizzas
Add corresponding elements. can you order?
  A 60
B 75
C 120
D 150
 
Simplify. SOLUTION:  
The number of ways to select two vegetable toppings
out of 6 is .
The number of ways to select one meat topping out
ANSWER:   of 5 is .
The number of different types of pizzas can order is
15 × 5 = 75.
Therefore, option B is the correct answer.
 
39. SAT/ACT Solve for x and y.
eSolutions Manual - Powered by Cognero Page 15
ANSWER:  
B
The solution is (–5, 7). Therefore, option F is the
correct answer.

ANSWER:  with Matrices
3-5 Operations
F
 
40. PROBABILITY A local pizzeria offers 5 different Subtract the fifth equation from the fourth equation
meat toppings and 6 different vegetable toppings. and solve for x.
You decide to get two vegetable toppings and one  
meat topping. How many different types of pizzas
can you order?
A 60
 
B 75
Substitute 3 for x in the fifth equation and solve for y.
C 120  
D 150
SOLUTION:  
The number of ways to select two vegetable toppings
out of 6 is .
The number of ways to select one meat topping out
 
of 5 is .
Substitute 3 and –1 for x and y in the first equation
The number of different types of pizzas can order is and solve for z.
15 × 5 = 75.
 
Therefore, option B is the correct answer.
 

ANSWER:  
B  
Solve each system of equations. Therefore, the solution is (3, –1, 4).

ANSWER:  
41.  (3, −1, 4)

SOLUTION:  
42. 

SOLUTION:  
 
Multiply the first equation by 4 and add with the
second equation.
 
 
Multiply the second equation by –3 and add with the
first equation.
 
 
Multiply the first equation by 5 and add with the third
equation.
 
 
Multiply the third equation by 2 and add with the first
equation.
 
 
Subtract the fifth equation from the fourth equation
eSolutions Manual - Powered by Cognero Page 16
and solve for x.
 
 
   
Multiply the with
3-5 Operations third equation
Matricesby 2 and add with the first Multiply the third equation by 4 and add with the first
equation. equation.
   

   
Multiply the fifth equation by –12 and add with the Multiply the third equation by –5 and add with the
fourth equation. second equation.
   

 
  Solve the fourth and the fifth equation.
Substitute 1 for y in the fifth equation and solve for x.  
 

 
 
Substitute –2 and 1 for x and y in the second
Substitute –3 for y in the fifth equation and solve for
equation and solve for z.
x.
 
 

 
 
The solution is (–2, 1, 6).
Substitute the values of x and y in the second
ANSWER:   equation and solve for z.
(−2, 1, 6)  

43. 

SOLUTION:    
Therefore, the solution is (4, –3, –2).

ANSWER:  
(4, −3, −2)
  44. PACKAGING The Cookie Factory sells chocolate
Multiply the third equation by 4 and add with the first chip and peanut butter cookies in combination
equation. packages that contain between six and twelve
  cookies. At least three of each type of cookie should
be in each package. How many of each type of
cookie should be in each package to maximize the
profit?
eSolutions Manual - Powered by Cognero Page 17
 
 
Multiply the third equation by –5 and add with the
  Therefore, 3 chocolate chip cookies and 9 peanut
Therefore, the solution is (4, –3, –2). butter cookies make the maximum profit.

ANSWER:  with Matrices
3-5 Operations ANSWER:  
(4, −3, −2) 3 chocolate chip, 9 peanut butter

44. PACKAGING The Cookie Factory sells chocolate Solve each system of inequalities by graphing.
chip and peanut butter cookies in combination
45. 
packages that contain between six and twelve
cookies. At least three of each type of cookie should
be in each package. How many of each type of SOLUTION:  
cookie should be in each package to maximize the  
profit?
 

SOLUTION:  
Let x and y be the number of chocolate chip and
peanut butter cookies, respectively. \
Optimizing function: ANSWER:  
 

 
Constraints:

 
The vertices of the solution region are (9, 3), (3, 9)
and (3, 3).
46. 
Substitute the points (9, 3), (3, 9) and (3, 3) in the
function f (x, y) = 0.25x + 0.26y.
  SOLUTION:  

 
The maximum 3.09 occurs at (3, 9).
Therefore, 3 chocolate chip cookies and 9 peanut
ANSWER:  
butter cookies make the maximum profit.

ANSWER:  
3 chocolate chip, 9 peanut butter

Solve each system of inequalities by graphing.


45. 

SOLUTION:  
  Manual - Powered by Cognero
eSolutions 47.  Page 18

SOLUTION:  
3-5 Operations with Matrices ANSWER:  
20 + 5x = 100; 16 bags

49. SPORTS There are 15,991 more student athletes in


47. 
New York than in Illinois. Write and solve an
equation to find the number of student athletes in
SOLUTION:   Illinois.
 

ANSWER:    
SOLUTION:  
Let x be the number of student athletes in Illinois.
The equation representing the situation is
.
 
Solve for x.

 
The number of student athletes in Illinois is 334,358.
48. RAKING LEAVES A student can earn $20 plus an
extra $5 for each trash bag he or she completely fill ANSWER:  
with leaves. Write and solve an equation to
350,349 – x = 15,991; 334,358
determine how many bags the student will need to fill
in order to earn $100. Simplify each expression.
SOLUTION:   50. 4(2x − 3y) + 2(5x − 6y)
Let x be the number of bags to fill. SOLUTION:  
The equation representing the situation is
.
 
Solve for x. ANSWER:  
18x − 24y

51. −3(2a − 5b) − 4(4b + a)


ANSWER:   SOLUTION:  
20 + 5x = 100; 16 bags

49. SPORTS There are 15,991 more student athletes in


New York than in Illinois. Write and solve an ANSWER:  
equation to find the number of student athletes in
−10a − b
Illinois.
  52. −7(x − y) + 5(y − x)
SOLUTION:  
eSolutions Manual - Powered by Cognero Page 19
ANSWER:  with Matrices
3-5 Operations
−10a − b

52. −7(x − y) + 5(y − x)


SOLUTION:  

ANSWER:  
−12x + 12y

eSolutions Manual - Powered by Cognero Page 20


ANSWER:  

3-6 Multiplying Matrices

Determine whether each matrix product is


defined. If so, state the dimensions of the 5. 
product.
1.  SOLUTION:  
The inner dimensions of the matrices are equal.
SOLUTION:   So:
The product is defined as the inner dimensions are
equal. Its dimensions are .

ANSWER:  
ANSWER:  
2×3

2. 

SOLUTION:  
6. 
The product is undefined as the inner dimensions are
not equal.
SOLUTION:  
ANSWER:   The inner dimensions of the matrices are equal.
undefined So:

3. 

SOLUTION:  
ANSWER:  
The product is defined as the inner dimensions are
equal. Its dimensions are .

ANSWER:  
7. 
8 × 10

Find each product, if possible. SOLUTION:  


The inner dimensions of the matrices are equal.
4.  So:

SOLUTION:  
The inner dimensions of the matrices are equal.
So: ANSWER:  

ANSWER:  
8. 

SOLUTION:  
The inner dimensions of the matrices are not equal.
So, the matrices cannot be multiplied.
5. 
ANSWER:  
SOLUTION:   Undefined
The inner dimensions of the matrices are equal.
So:
eSolutions Manual - Powered by Cognero 9.  Page 1

SOLUTION:  
The inner dimensions of the matrices are not equal.
So, the matrices cannot be multiplied. ANSWER:  
ANSWER:  Matrices
3-6 Multiplying
Undefined

12. CCSS SENSE-MAKING The table shows the


9.  number of people registered for aerobics for the first
quarter.
SOLUTION:   Quinn’s Gym charges the following registration fees:
The inner dimensions of the matrices are equal. class-by-class, $165; 11-class pass, $110; unlimited
So: pass, $239.
 

ANSWER:  
 
a. Write a matrix for the registration fees and a
matrix for the number of students.
b. Find the total amount of money the gym received
10.  from aerobics and step aerobic registrations.
SOLUTION:  
SOLUTION:  
The inner dimensions of the matrices are equal. a.
So:
b.  Money received from aerobics registrations:

ANSWER:  
Money received from step aerobic registrations:

Total = $13,597 + $9,358 = $22,955


11. 
ANSWER:  

a.
SOLUTION:  
The inner dimensions of the matrices are equal.
So: b. $22,955

Use  to 

determine whether the following equations are


true for the given matrices.
 
ANSWER:   13. XY = YX
SOLUTION:  

12. CCSS SENSE-MAKING The table shows the


eSolutions Manual - Powered by Cognero Page 2
number of people registered for aerobics for the first
quarter.
Quinn’s Gym charges the following registration fees:
ANSWER:  
a.
3-6 Multiplying Matrices No;
b. $22,955

Use  to  14. X(YZ) = (XY)Z

determine whether the following equations are SOLUTION:  


true for the given matrices.
 
13. XY = YX
SOLUTION:  
 

Therefore:

ANSWER:    
Therefore:
No;

ANSWER:  
14. X(YZ) = (XY)Z
Yes;  and 
SOLUTION:  

Determine whether each matrix product is


defined. If so, state the dimensions of the
  product.
 
15. P2 × 3 · Q3 × 4

SOLUTION:  
The product is defined as the inner dimensions are
equal. Its dimensions are .
 
ANSWER:  
2×4

16. A 5 × 5 · B5 × 5
  SOLUTION:  
Therefore:
The product is defined as the inner dimensions are
equal. Its dimensions are .
ANSWER:  
eSolutions Manual - Powered by Cognero ANSWER:   Page 3

5×5
Yes;  and 
The product is defined as the inner dimensions are
equal. Its dimensions are .
equal. Its dimensions are .
ANSWER:  
3-6 Multiplying Matrices ANSWER:  
2×4
5×4

16. A 5 × 5 · B5 × 5 Find each product, if possible.

SOLUTION:   21. 
The product is defined as the inner dimensions are
equal. Its dimensions are . SOLUTION:  

ANSWER:  
5×5

17. M 3 × 1 · N 2 × 3

SOLUTION:  
ANSWER:  
The product is undefined as the inner dimensions are
not equal. [ 26 ]

ANSWER:  
undefined 22. 

18. X 2 × 6 · Y6 × 3 SOLUTION:  
SOLUTION:  
The product is defined as the inner dimensions are
equal. Its dimensions are .

ANSWER:  
2×3
ANSWER:  
19. J2 × 1 · K2 × 1

SOLUTION:  
The product is undefined as the inner dimensions are
not equal. 23. 

ANSWER:  
SOLUTION:  
undefined

20. S 5 × 2 · T2 × 4

SOLUTION:  
The product is defined as the inner dimensions are
equal. Its dimensions are .
ANSWER:  
ANSWER:  
5×4

Find each product, if possible.


24. 
21. 

SOLUTION:  
SOLUTION:  

eSolutions Manual - Powered by Cognero Page 4


ANSWER:   ANSWER:  

3-6 Multiplying Matrices

24. 
27. 

SOLUTION:  
SOLUTION:  

ANSWER:   ANSWER:  

25.  28. 

SOLUTION:   SOLUTION:  
The inner dimensions of the matrices are not equal.
So, the matrices cannot be multiplied.

ANSWER:  
Undefined

ANSWER:  
26. 

SOLUTION:  
29. TRAVEL The Wolf family owns three bed and
breakfasts in a vacation spot. A room with a single
bed is $220 a night, a room with two beds is $250 a
night, and a suite is $360.
 

ANSWER:  
 
a. Write a matrix for the number of each type of
room at each bed and breakfast. Then write a room-
cost matrix.
b. Write a matrix for total daily income, assuming
27. 
that all the rooms are rented.
c. What is the total daily income from all three bed
SOLUTION:   and breakfasts, assuming that all the rooms are
rented?
eSolutions Manual - Powered by Cognero SOLUTION:   Page 5

a.
b. Write a matrix for total daily income, assuming
that all the rooms are rented. b.
c. What is the total daily income from all three bed
3-6 Multiplying
and breakfasts,Matrices
assuming that all the rooms are
rented? c.  $5060

SOLUTION:  
Use , and k

a. = 2 to determine whether the following


equations are true for the given matrices.
 
30. k(PQ) = P(k Q)
SOLUTION:  

b. Multiply the matrices.

 
When k = 2:
The total daily income is given by the matrix

c. Add the elements of the matrix.


Total daily income = $5060

ANSWER:  

a.
 
When k = 2:

b.
 
c.  $5060 So:

Use , and k
ANSWER:  
= 2 to determine whether the following Yes;
equations are true for the given matrices.
 
30. k(PQ) = P(k Q)
31. PQR = RQP
SOLUTION:  
SOLUTION:  

 
eSolutions Manual - Powered by Cognero Page 6

 
When k = 2:
ANSWER:  
ANSWER:  
Yes;
3-6 Multiplying Matrices No;

31. PQR = RQP 32. PR + QR = (P + Q)R


SOLUTION:   SOLUTION:  

   

   

   

   
So:

ANSWER:  
No;
 
So:
32. PR + QR = (P + Q)R
SOLUTION:   ANSWER:  
Yes;

33. R(P + Q) = PR + QR
SOLUTION:  
 

 
eSolutions Manual - Powered by Cognero Page 7
So: So:

ANSWER:   ANSWER:  
3-6 Multiplying
Yes; Matrices No;

33. R(P + Q) = PR + QR 34. CCSS SENSE-MAKING Student Council is selling


flowers for Mother’s Day. They bought 200 roses,
SOLUTION:   150 daffodils, and 100 orchids for the purchase prices
shown. They sold all of the flowers for the sales
prices shown.
 

 
a. Organize the data in two matrices, and use matrix
multiplication to find the total amount that was spent
on the flowers.
  b. Write two matrices, and use matrix multiplication
So: to find the total amount the student council received
for the flower sale.
ANSWER:   c. Use matrix operations to find how much money
No; the student council made on their project.
SOLUTION:  
34. CCSS SENSE-MAKING Student Council is selling
flowers for Mother’s Day. They bought 200 roses, a.
150 daffodils, and 100 orchids for the purchase prices
shown. They sold all of the flowers for the sales
prices shown. The amount was $747.50.
 
b.

The amount the student council received was


$1387.50.
c.
    
a. Organize the data in two matrices, and use matrix
multiplication to find the total amount that was spent The profit was $640.
on the flowers.
b. Write two matrices, and use matrix multiplication ANSWER:  
to find the total amount the student council received a. $747.50
for the flower sale. b. $1387.50
c. Use matrix operations to find how much money c. $640
the student council made on their project.
35. AUTO SALES A car lot has four sales associates.
SOLUTION:  
At the end of the year, each sales associate gets a
bonus of $1000 for every new car they have sold and
a. $500 for every used car they have sold.
 
The amount was $747.50.
eSolutions Manual - Powered by Cognero Page 8
b.
ANSWER:   a.
a. $747.50
b. $1387.50 Matrices
3-6 Multiplying
c. $640 b. $187,500

35. AUTO SALES A car lot has four sales associates. Use matrices
At the end of the year, each sales associate gets a  to find 
bonus of $1000 for every new car they have sold and
$500 for every used car they have sold.
  each of the following. If the matrix does not
exist, write undefined.
36. XY
SOLUTION:  
The inner dimensions are not equal. So, the product is
undefined.

ANSWER:  
undefined
 
a. Use a matrix to determine which sales associate
earned the most money. 37. YX
b. What is the total amount of money the car lot SOLUTION:  
spent on bonuses for the sales associates this year?
SOLUTION:  

a.
ANSWER:  

 
Therefore, Westin earned the most money.
 
b. Find the sum of the bonus amounts earned by 38. ZY
each associate.
SOLUTION:  
51500 + 53000 + 37000 + 46000 = 187500
So, the total amount is $187,500. The inner dimensions are not equal. So, the product is
undefined.
ANSWER:  
ANSWER:  
undefined
a.
39. YZ
SOLUTION:  
b. $187,500

Use matrices
 to find 

each of the following. If the matrix does not


exist, write undefined.
36. XY
SOLUTION:   ANSWER:  
The inner dimensions are not equal. So, the product is
undefined.
eSolutions Manual - Powered by Cognero Page 9
ANSWER:  
undefined
40. (YX)Z
The inner dimensions are not equal. So, the product is The product of the matrices  is not defined. So, 
undefined. the product X(ZZ) is undefined.

ANSWER:  Matrices
3-6 Multiplying ANSWER:  
undefined undefined

39. YZ 43. (XX)Z
SOLUTION:   SOLUTION:  

ANSWER:  
ANSWER:  

44. CAMERAS Prices of digital cameras depend on


features like optical zoom, digital zoom, and
40. (YX)Z megapixels.
SOLUTION:    

 
a. The 10-mp cameras are on sale for 20% off, and
the other models are 10% off. Write a new matrix
ANSWER:  
for these changes.
b. Write a new matrix allowing for a 6.25% sales tax
on the discounted prices.
c. Describe what the differences in these two
matrices represent.
41. (XZ)X SOLUTION:  
SOLUTION:  
The inner dimensions of the matrices are not equal. a.
So, the product is undefined.

ANSWER:    
undefined b. Add 6.25% of the discounted price to each
element.
42. X(ZZ)
SOLUTION:  
The product of the matrices  is not defined. So, 
the product X(ZZ) is undefined.  
ANSWER:   c. Sales tax
undefined ANSWER:  
43. (XX)Z
SOLUTION:   a.
eSolutions Manual - Powered by Cognero Page 10

 
 
c. sales tax
ANSWER:  

3-6 Multiplying Matrices 45. BUSINESS The Kangy Studio has packages


available for senior portraits.
 
44. CAMERAS Prices of digital cameras depend on
features like optical zoom, digital zoom, and
megapixels.
 

   
a. The 10-mp cameras are on sale for 20% off, and a. Use matrices to determine the total cost of each
the other models are 10% off. Write a new matrix package.
for these changes.
b. The studio offers an early bird discount of 15% off
b. Write a new matrix allowing for a 6.25% sales tax
any package. Find the early bird price for each
on the discounted prices. package.
c. Describe what the differences in these two
matrices represent. SOLUTION:  
a. Package A:
SOLUTION:  

a.

 
b. Add 6.25% of the discounted price to each
element.

 
c. Sales tax
Total cost for package A is $421.
ANSWER:    
Package B:
a.

b.

 
c. sales tax

45. BUSINESS The Kangy Studio has packages


available for senior portraits.
  Total cost for package B is $274.
 
Package C:

eSolutions Manual - Powered by Cognero Page 11


.

ANSWER:  
Total cost forMatrices
3-6 Multiplying package B is $274. a. A: $421; B: $274; C: $150; D: $68
  b. A: $357.85; B: $232.90; C: $127.50; D: $57.80
Package C:
46. REASONING  If the product matrix AB has
dimensions 5 × 8, and A has dimensions 5 × 6, what 
are the dimensions of matrix B?
SOLUTION:  
The inner dimensions should be equal. So, the
dimensions of the matrix B are .

ANSWER:  
6 × 8

47. CCSS ARGUMENTS Show that each property of


matrices.
a. Scalar Distributive Property
Total cost for package C is $150. b. Matrix Distributive Property
  c. Associative Property of Multiplication
Package D: d. Associative Property of Scalar Multiplication
SOLUTION:  
a.

Total cost for package D is $68. b.  Use the definitions of matrix multiplication, Distrib
  Property of Addition, 
b. The early bird price for package A is and the definition of matrix addition.
.
The early bird price for package B is
.
The early bird price for package C is
.
The early bird price for package D is
.

ANSWER:  
a. A: $421; B: $274; C: $150; D: $68
b. A: $357.85; B: $232.90; C: $127.50; D: $57.80

46. REASONING  If the product matrix AB has
dimensions 5 × 8, and A has dimensions 5 × 6, what 
are the dimensions of matrix B?
SOLUTION:  
The Manual
eSolutions inner dimensions should be equal. So, the
- Powered by Cognero Page 12
dimensions of the matrix B are .

ANSWER:  
=
Distri
b.  Use the definitions
3-6 Multiplying Matricesof matrix multiplication, Distrib
Property of Addition,  =
and the definition of matrix addition. Defini

=
Defini

= (cA)B Substi
 Subst

c(AB) =
Defini

=c
Defini
=
Distri

=
Com

=
Defini
c.  Use the definition of matrix multiplication, Distribu
Commutative Property of Addition. =
= A(cB) Substi
 
ANSWER:  
a.

d.
Substi

c(AB) =
Defini
b.  
 = c
Defini
=
Distri

=
Defini

=
eSolutions Manual - Powered by Cognero Defini Page 13

= (cA)B
=
Distri
3-6 Multiplying Matrices
=
b.   Defini

=
Defini

= (cA)B Substi
 Subst

c(AB) =
Defini

=c
Defini
=
Distri

=
Com

=
Defini
c.  
=
= A(cB) Substi
 

48. OPEN ENDED Write two matrices A and B such


that AB = BA.
SOLUTION:  
Sample answer:

d.
Substi

c(AB) =
ANSWER:  
Defini
Sample Answer:
 = c
Defini
49. CHALLENGE  Find the missing values in 
=
Distri

= SOLUTION:  
Defini

=
Defini
eSolutions Manual - Powered by Cognero Page 14

= (cA)B Substi
which does not change, by the scoring matrix, which
ANSWER:   changes after each season. The total number of
points for her career can be found by multiplying the
3-6 Multiplying Matrices
Sample Answer: scoring matrix S by the point matrix P. Basketball
and wrestling use different point values in scoring.

49. CHALLENGE  Find the missing values in  51. GRIDDED RESPONSE The average (arithmetic


mean) of r, w, x, and y is 8, and the average of x and
y is 4. What is the average of r and w?
SOLUTION:  
SOLUTION:  

So:

Solve the equations:


a = 2, b = 1, c = 3, and d = 4

ANSWER:   That is, the average of r and w is 12.


a = 2, b = 1, c = 3, d = 4  

ANSWER:  
50. WRITING IN MATH Use the data on Lisa Leslie
found at the beginning of the lesson to explain how 12
matrices can be used in sports statistics. Describe a
52. Carla, Meiko, and Kayla went shopping to get ready
matrix that represents the total number of points she
for college. Their purchases and total amounts spent
has scored during her career and an example of a
are shown in the table below.
sport in which different point values are used in
 
scoring.
 
SOLUTION:  
Sports statistics are often listed in columns and
matrices. In this case, you can find the total number
of points scored by multiplying the point value matrix,  
which does not change, by the scoring matrix, which Assume that all of the shirts were the same price, all
changes after each season. The total number of of the pants were the same price, and all of the shoes
points for her career can be found by multiplying the were the same price. What was the price of each
scoring matrix S by the point matrix P. Basketball item?
and wrestling use different point values in scoring. A  shirt, $12.95; pants, $15.99; shoes, $23.49
B   shirt, $15.99; pants, $12.95; shoes, $23.49
ANSWER:   C  shirt, $15.99; pants, $23.49; shoes, $12.95
Sports statistics are often listed in columns and D  shirt, $23.49; pants, $15.99; shoes, $12.95
matrices. In this case, you can find the total number
of points scored by multiplying the point value matrix, SOLUTION:  
which does not change, by the scoring matrix, which Let a be the price of a shirt, b be the price of pants,
changes after each season. The total number of and c be the price of a pair of shoes.
points for her career can be found by multiplying the  
scoring matrix S by the point matrix P. Basketball
and wrestling use different point values in scoring.

51. GRIDDED RESPONSE The average (arithmetic


mean) of r, w, x, and y is 8, and the average of x and
y is 4.
eSolutions What
Manual is the average
- Powered of r and w?
by Cognero Page 15

SOLUTION:  
That is, the average of r and w is 12. a = 12.95, b = 15.99, and c = 23.49
  The correct choice is A.

ANSWER:  Matrices
3-6 Multiplying ANSWER:  
12 A

52. Carla, Meiko, and Kayla went shopping to get ready 53. GEOMETRY Rectangle LMNQ has diagonals that
for college. Their purchases and total amounts spent intersect at point P. Which of the following
are shown in the table below. represents point P?
 

  F   (2, 2)          
Assume that all of the shirts were the same price, all G  (1, 1)           
of the pants were the same price, and all of the shoes H  (0, 0)
were the same price. What was the price of each J    (−1, −1)
item? SOLUTION:  
A  shirt, $12.95; pants, $15.99; shoes, $23.49
The diagonals of a rectangle bisect each other. So, P
B   shirt, $15.99; pants, $12.95; shoes, $23.49
is the midpoint of LN.
C  shirt, $15.99; pants, $23.49; shoes, $12.95
 
D  shirt, $23.49; pants, $15.99; shoes, $12.95 So:
SOLUTION:  
Let a be the price of a shirt, b be the price of pants,
and c be the price of a pair of shoes. The correct choice is H.
 
ANSWER:  
H

54. SAT/ACT What are the dimensions of the matrix


that results from the multiplication shown?

A  1 × 4           
B   3 × 3
  C  3 × 1
So: D  4 × 1           
E  4 × 3
SOLUTION:  
The dimensions of the matrices are  and  .
 
So, the dimensions of the resultant matrix are .
Solve:
The correct choice is D.
a = 12.95, b = 15.99, and c = 23.49
The correct choice is A. ANSWER:  
D
ANSWER:  
A Perform the indicated operations. If the matrix
does not exist, write impossible.
53. GEOMETRY Rectangle LMNQ has diagonals that
intersect at point P. Which of the following 55. 
represents point P?
eSolutions Manual - Powered by Cognero SOLUTION:   Page 16
So, the dimensions of the resultant matrix are .
The correct choice is D. ANSWER:  
ANSWER:  Matrices
3-6 Multiplying
D

Perform the indicated operations. If the matrix


does not exist, write impossible. 57. 

55. 
SOLUTION:  

SOLUTION:  

ANSWER:  
ANSWER:  

Solve each system of equations.


56. 

SOLUTION:  
58. 
SOLUTION:  

 
Eliminate one variable.
Multiply the third equation by –2 and add with the
first equation.
 
ANSWER:  

 
57.  Multiply the first equation by 7 and the second
equation by 2. Then add.
 
SOLUTION:  

 
Solve for one variable using the the fourth and fifth
equations.
eSolutions Manual - Powered by Cognero   Page 17
 
3-6 Multiplying Matrices
Solve for one variable using the the fourth and fifth  
equations. Multiply the second equation by 3 and the third
  equation by -10. Then add.
 

 
  Solve for one variable using the the fourth and fifth
Substitute 7 for z in the fourth equation and solve for equations.
y.  
 

 
Substitute 1 for z in the fourth equation and solve for
  y.
Substitute 5 for y and 7 for z in the third equation,  
and solve for x.
 

 
  Substitute 3 for y and 1 for z in the third equation,
Therefore, the solution is (–2, 5, 7). and solve for x.
 
ANSWER:  
(–2, 5, 7)

 
59.  Therefore, the solution is (8, 3, 1).
SOLUTION:   ANSWER:  
(8, 3, 1)

 
Eliminate one variable. 60. 
Multiply the first equation by 3 and the third equation SOLUTION:  
by 4. Then add.
 

 
Eliminate one variable.
  Multiply the second equation by 4 and add with the
Multiply the second equation by 3 and the third first equation.
equation
eSolutions by- Powered
Manual -10. Then add.
by Cognero   Page 18
 
ANSWER:  
(–3, 2, –6)

 
3-6 Multiplying Matrices 61. MEDICINE The graph shows how much
Eliminate one variable. Americans spent on doctors’ visits in some recent
Multiply the second equation by 4 and add with the years and a prediction for 2014.
first equation. a. Find a regression equation for the data without the
  predicted value.
b. Use your equation to predict the expenditures for
2014.
c. Compare your prediction to the one given in the
graph.
   
Multiply the third equation by  and and add with the 
first equation.
 

 
Solve for one variable using the the fourth and fifth
equations.  
  SOLUTION:  
a. A sample answer is given. Use a graphing
calculator to make a scatter plot of the data. Enter
1999, 2000, 2001, 2002, 2003 into L1 and 1222.2,
1309.9, 1426.4, 1559.0, and 1678.9 in L2. Then press
LIST, select the CALC menu, and select LinReg
  (ax+b). The calculator returns an a-value of 116.25
Substitute -6 for z in the fourth equation and solve for and a b-value of 231176.97, so a regression equation
y. for the data where x is the year and y is the national
  health expenditure in billions is y = 116.25x -
231,176.97.
 
b. Replace x with 2014 in the equation y = 116.25x -
231,176.97 to calculate the national health
expenditure in billions for the year 2014.
 
Substitute -3 for x and -6 for z in the third equation,
and solve for y. Therefore, using this equation, we predict that in
  2014 the national health expenditure will be $2950.53
billion.
 
c. The value predicted by the equation, $2950.53
billion, is significantly lower than the one given in the
graph, $3585.7 billion.
 
ANSWER:  
Therefore, the solution is (–3, 2, –6).
a. Sample answer: y = 116.25x - 231,176.97
ANSWER:   b. Sample answer: $2950.53 billion
(–3, 2, –6) c. The value predicted by the equation is significantly
lower than the one given in the graph.
61. MEDICINE The graph shows how much
Americans spent on doctors’ visits in some recent 62. How many different ways can the letters of the word
years and a prediction for 2014. MATHEMATICS be arranged?
a. Find
eSolutions a regression
Manual equation
- Powered by Cognero for the data without the
SOLUTION:   Page 19
predicted value.
b. Use your equation to predict the expenditures for
ANSWER:  
a. Sample answer: y = 116.25x - 231,176.97
b. Sample answer: $2950.53 billion
c. The value predicted
3-6 Multiplying Matricesby the equation is significantly
lower than the one given in the graph.

62. How many different ways can the letters of the word 64. 
MATHEMATICS be arranged?
SOLUTION:  
SOLUTION:  
The graph of is a translation of the
graph of  down 5 units and left 3 units. Also, a
vertical stretch of the graph of  by a degree 2.

ANSWER:  
4,989,600

Describe the transformation in each function.


Then graph the function.
63. 

SOLUTION:  
The graph of  is a translation of the 
graph of  right 4 units and up 3 units.
ANSWER:  
Translated 3 units to the left and 5 units down and
stretched vertically..

ANSWER:  
Translated 4 units to the right and 3 units up.
65. 

SOLUTION:  
The graph of  is a translation of the 
graph of  left 2 units and down 6 units.

64. 

SOLUTION:  
The graph of is a translation of the
graph of  down 5 units and left 3 units. Also, a
ANSWER:  
vertical stretch of the graph of  by a degree 2.
Translated 2 units to the left and 6 units down.
eSolutions Manual - Powered by Cognero Page 20
3-6 Multiplying Matrices

65. 

SOLUTION:  
The graph of  is a translation of the 
graph of  left 2 units and down 6 units.

ANSWER:  
Translated 2 units to the left and 6 units down.

eSolutions Manual - Powered by Cognero Page 21


3-7 Solving Systems of Equations Using Cramer's Rule ANSWER:  
−128

Evaluate each determinant.


4. 
1. 
SOLUTION:  
SOLUTION:  

ANSWER:  
ANSWER:   0
26
Evaluate each determinant using diagonals.

2.  5. 

SOLUTION:  
SOLUTION:  
Rewrite the first two columns in the right of the
determinant.
 

ANSWER:  
−12
 
Find the product of the element of the diagonal.
3. 
 

SOLUTION:  

 
ANSWER:   Find the sum of each group.
−128  

4.   
Subtract the value of the second group from the first
group.
SOLUTION:    

 
Therefore, the value of the determinant is –19.

ANSWER:  
ANSWER:   −19
0 Manual - Powered by Cognero
eSolutions Page 1

Evaluate each determinant using diagonals.


6. 
 
Therefore, the value of the determinant is –19.

ANSWER:  
ANSWER:  
3-7 Solving Systems of Equations Using Cramer's Rule −19
0

Evaluate each determinant using diagonals.


6. 
5. 

SOLUTION:  
SOLUTION:   Rewrite the first two columns in the right of the
Rewrite the first two columns in the right of the determinant.
determinant.  
 

 
  Find the product of the element of the diagonal.
Find the product of the element of the diagonal.  
 

 
Find the sum of each group.
 
 
Find the sum of each group.
 
 
Subtract the value of the second group from the first
 
group.
Subtract the value of the second group from the first
 
group.
 
 
Therefore, the value of the determinant is –80.
 
Therefore, the value of the determinant is –19. ANSWER:  
ANSWER:   −80
−19
7. 

6. 
SOLUTION:  
Rewrite the first two columns in the right of the
SOLUTION:   determinant.
Rewrite the first two columns in the right of the  
determinant.
 

 
eSolutions Manual - Powered by Cognero Find the product of the element of the diagonal. Page 2
   
Find the product of the element of the diagonal.
   
Therefore, the value of the determinant is –80. Therefore, the value of the determinant is –284.

ANSWER:  
3-7 Solving Systems of Equations Using Cramer's Rule ANSWER:  
−80 −284

7.  8. 

SOLUTION:   SOLUTION:  
Rewrite the first two columns in the right of the Rewrite the first two columns in the right of the
determinant. determinant.
   

   
Find the product of the element of the diagonal. Find the product of the element of the diagonal.
   

   
Find the sum of each group. Find the sum of each group.
   

   
Subtract the value of the second group from the first Subtract the value of the second group from the first
group. group.
   

   
Therefore, the value of the determinant is –284. Therefore, the value of the determinant is 302.

ANSWER:   ANSWER:  
−284 302

8.  9. 

SOLUTION:   SOLUTION:  
Rewrite the first two columns in the right of the Rewrite the first two columns in the right of the
determinant. determinant.
 

Find the product of the element of the diagonal.


 
eSolutions Manual - Powered by Cognero Page 3
Find the product of the element of the diagonal.
 
 
Therefore, the value of the determinant is 302. Therefore, the value of the determinant is 72.

ANSWER:  
3-7 Solving Systems of Equations Using Cramer's Rule ANSWER:  
302 72

9.  10. 

SOLUTION:   SOLUTION:  
Rewrite the first two columns in the right of the Rewrite the first two columns in the right of the
determinant. determinant.
 

Find the product of the element of the diagonal.


 
Find the product of the element of the diagonal.
 

Find the sum of each group.


 
Subtract the value of the second group from the first Find the sum of each group.
group.  

Therefore, the value of the determinant is 72.  


Subtract the value of the second group from the first
ANSWER:   group.
72  

 
10. 
Therefore, the value of the determinant is 139.

ANSWER:  
SOLUTION:   139
Rewrite the first two columns in the right of the
determinant.
  11. 

SOLUTION:  
Rewrite the first two columns in the right of the
  determinant.
Find the product of the element of the diagonal.  
 

 
eSolutions Manual - Powered by Cognero Page 4
Find the product of the element of the diagonal.
 
   
Therefore, the value of the determinant is 139. Therefore, the value of the determinant is 182.

ANSWER:  
3-7 Solving Systems of Equations Using Cramer's Rule ANSWER:  
139 182

11.  12. 

SOLUTION:   SOLUTION:  
Rewrite the first two columns in the right of the Rewrite the first two columns in the right of the
determinant. determinant.
   

   
Find the product of the element of the diagonal. Find the product of the element of the diagonal.
   

   
Find the sum of each group. Find the sum of each group.
   

   
Subtract the value of the second group from the first Subtract the value of the second group from the first
group. group.
   

   
Therefore, the value of the determinant is 182. Therefore, the value of the determinant is 562.

ANSWER:   ANSWER:  
182 562

Use Cramer’s Rule to solve each system of


equations.
12. 
13. 4x − 5y = 39
3x + 8y = −6
SOLUTION:   SOLUTION:  
Rewrite the first two columns in the right of the Use Cramer’s Rule.
determinant.  
  Let C be the coefficient matrix of the system
 

  .

   
eSolutions Manual - Powered by Cognero Page 5
Find the product of the element of the diagonal. The solution of the system is
   
  Therefore, the solution of the system is (6, –3).
Therefore, the value of the determinant is 562.
ANSWER:  
ANSWER:  
3-7 Solving Systems of Equations Using Cramer's Rule (6, −3)
562

Use Cramer’s Rule to solve each system of 14. 5x + 6y = 20


equations. −3x − 7y = −29
13. 4x − 5y = 39
SOLUTION:  
3x + 8y = −6
Use Cramer’s Rule.
SOLUTION:    
Use Cramer’s Rule. Let C be the coefficient matrix of the system
   
Let C be the coefficient matrix of the system
    .

  .  
The solution of the system is
   
The solution of the system is
 
  if .

  if .  

 
Therefore, the solution of the system is (–2, 5).
  ANSWER:  
Therefore, the solution of the system is (6, –3). (−2, 5)
ANSWER:   15. −8a − 5b = −27
(6, −3) 7a + 6b = 22
SOLUTION:  
14. 5x + 6y = 20 Use Cramer’s Rule.
−3x − 7y = −29
 
SOLUTION:   Let C be the coefficient matrix of the system
Use Cramer’s Rule.  
 
  .
Let C be the coefficient matrix of the system
   
  . The solution of the system is
 
eSolutions Manual - Powered by Cognero Page 6
 
The solution of the system is
    if .
   
Therefore, the solution of the system is (–2, 5). Therefore, the solution of the system is (4, –1).

ANSWER:  
3-7 Solving Systems of Equations Using Cramer's Rule ANSWER:  
(−2, 5) (4, −1)

15. −8a − 5b = −27 16. 10c − 7d = −59


7a + 6b = 22 6c + 5d = −63
SOLUTION:   SOLUTION:  
Use Cramer’s Rule. Use Cramer’s Rule.
   
Let C be the coefficient matrix of the system Let C be the coefficient matrix of the system
   
  .   .
   
The solution of the system is The solution of the system is
   

  if .   if .

   

   

   
Therefore, the solution of the system is (4, –1). Therefore, the solution of the system is (–8, –3).
ANSWER:   ANSWER:  
(4, −1) (−8, −3)

16. 10c − 7d = −59 17. CCSS PERSEVERANCE The ″Bermuda Triangle″


6c + 5d = −63 is an area located off the southeastern Atlantic coast
of the United States, and noted for reports of
SOLUTION:   unexplained losses of ships, small boats, and aircraft.
Use Cramer’s Rule.  
 
Let C be the coefficient matrix of the system
 

  .

 
The solution of the system is
 
eSolutions Manual - Powered by Cognero Page 7

  if .
17. CCSS PERSEVERANCE The ″Bermuda Triangle″
is an area located off the southeastern Atlantic coast
of the United
3-7 Solving States,
Systems and noted for
of Equations reports
Using of
Cramer's Rule
unexplained losses of ships, small boats, and aircraft.  
2
  A = 15.75 unit .
b. 1 unit = 175 miles
2 2
Therefore, 1 unit = 30625 mi
2 2
Therefore, 15.75 unit = 482343.75 mi .
2
Area of the Bermuda triangle is 482,343.75 mi .

ANSWER:  
2
a. 15.75 units
b. 482,343.75 mi2

Use Cramer’s Rule to solve each system of


equations.
  18. 4x − 2y + 7z = 26
a. Find the area of the triangle on the map. 5x + 3y − 5z = −50
b. Suppose each grid represents 175 miles. What is −7x – 8y − 3z = 49
the area of the Bermuda Triangle?
SOLUTION:  
SOLUTION:   Use Cramer’s Rule.
a. The area of a triangle with vertices (a, b), (c, d)  
Let C be the coefficient matrix of the system
 
and (f , g) is | A |, where .
  .
The vertices of the Bermuda triangle are (4, 4), (9, 1)
and (9.5, 7).
 
The solution of the system is
 

     if  .

 
 

 
2
A = 15.75 unit .
b. 1 unit = 175 miles
eSolutions Manual - Powered by Cognero Page 8
2 2
Therefore, 1 unit = 30625 mi
2 2
Therefore, 15.75 unit = 482343.75 mi .
  .

3-7 Solving Systems of Equations Using Cramer's Rule  


The solution of the system is
   

   if  .

 
Therefore, the solution of the system is (–3, –5, 4).

ANSWER:  
(−3, −5, 4)

19. −3x − 5y + 10z = −4


−8x + 2y − 3z = −91
6x + 8y − 7z = −35
SOLUTION:  
Use Cramer’s Rule.
 
Let C be the coefficient matrix of the system
 
 
  . Therefore, the solution of the system
is .
 
The solution of the system is ANSWER:  
 

   if  .
eSolutions Manual - Powered by Cognero 20. 6x − 5y + 2z = −49 Page 9
  −5x − 3y − 8z = −22
−3x + 8y − 5z = 55
ANSWER:  

3-7 Solving Systems of Equations Using Cramer's Rule


 
20. 6x − 5y + 2z = −49
−5x − 3y − 8z = −22
−3x + 8y − 5z = 55
SOLUTION:  
Use Cramer’s Rule.
 
Let C be the coefficient matrix of the system
 

  .

 
The solution of the system is
 

   if  .

   
Therefore, the solution of the system is (–3, 7, 2).

ANSWER:  
(−3, 7, 2)

21. −9x + 5y + 3z = 50
7x + 8y − 2z = −60
−5x + 7y + 5z = 46
SOLUTION:  
Use Cramer’s Rule.
 
Let C be the coefficient matrix of the system
 

  .

 
The solution of the system is
 

   if  .
eSolutions Manual - Powered by Cognero Page 10
 

 
  . 3y − 4z = 25
x + 6y + z = 20

  SOLUTION:  
3-7 Solving Systems of Equations Using Cramer's Rule
The solution of the system is Use Cramer’s Rule.
   
Let C be the coefficient matrix of the system
 
   if  .
  .
 
 
The solution of the system is
 

     if  .

 
Therefore, the solution of the system is (–4, –2, 8).

ANSWER:  
(−4, −2, 8)

22. x + 2y = 12
3y − 4z = 25
x + 6y + z = 20
 
SOLUTION:   Therefore, the solution of the system is (6, 3, –4).
Use Cramer’s Rule.
  ANSWER:  
Let C
eSolutions be the
Manual coefficient
- Powered matrix of the system
by Cognero (6, 3, −4) Page 11
 
23. 9a + 7b = −30
8b + 5c = 11
 
Therefore,
3-7 Solving the solution
Systems of the system
of Equations is (6,
Using 3, –4). Rule
Cramer's
ANSWER:    
(6, 3, −4)

23. 9a + 7b = −30
8b + 5c = 11
−3a + 10c = 73
SOLUTION:  
Use Cramer’s Rule.
 
Let C be the coefficient matrix of the system
 

  .

 
The solution of the system is
 

   if  .

 
Therefore, the solution of the system is (–1, –3, 7).
 
ANSWER:  
(−1, −3, 7)

24. 2n + 3p − 4w = 20
4n − p + 5w = 13
3n + 2p + 4w = 15
SOLUTION:  
Use Cramer’s Rule.
 
Let C be the coefficient matrix of the system
 

  .

 
The solution of the system is
 

   if  .
eSolutions Manual - Powered by Cognero Page 12
 
3x + 4y + 2z = 28
  .
SOLUTION:  
Use Cramer’s Rule.
 
3-7 Solving Systems of Equations Using Cramer's Rule
The solution of the system is
 
  Let C be the coefficient matrix of the system
 

   if  .   .

   
The solution of the system is
 

   if  .

 
Therefore, the solution of the system is (5, 2, –1).

ANSWER:  
(5, 2, −1)

25. x + y + z = 12
6x − 2y − z = 16
3x + 4y + 2z = 28
 
SOLUTION:   Therefore, the solution of the system is (4, 0, 8).
Use Cramer’s Rule.
ANSWER:  
 
Let C be the coefficient matrix of the system (4, 0, 8)
 
eSolutions Manual - Powered by Cognero Evaluate each determinant. Page 13

  . 26. 
 
Therefore, the solution of the system is (4, 0, 8).

ANSWER:  
3-7 Solving Systems of Equations Using Cramer's Rule ANSWER:  
(4, 0, 8) 83

Evaluate each determinant.

26.  29. 

SOLUTION:   SOLUTION:  
Rewrite the first two columns in the right of the
determinant.
 

ANSWER:  
−102
 
Find the product of the element of the diagonal.
 
27. 

SOLUTION:  

 
Find the sum of each group.
ANSWER:    
3
 
28.  Subtract the value of the second group from the first
group.
 
SOLUTION:  

 
Therefore, the value of the determinant is –135.

ANSWER:  
−135
ANSWER:  
83
30. 

29. 
SOLUTION:  
Rewrite the first two columns in the right of the
SOLUTION:   determinant.
Rewrite the first two columns in the right of the  
determinant.
 

 
eSolutions Manual - Powered by Cognero Page 14
Find the product of the element of the diagonal.
   
   
Therefore, the value of the determinant is –135. Therefore, the value of the determinant is 124.

ANSWER:  
3-7 Solving Systems of Equations Using Cramer's Rule ANSWER:  
−135 124

30.  31. 

SOLUTION:   SOLUTION:  
Rewrite the first two columns in the right of the Rewrite the first two columns in the right of the
determinant. determinant.
   

   
Find the product of the element of the diagonal. Find the product of the element of the diagonal.
   

   
Find the sum of each group. Find the sum of each group.
   

   
Subtract the value of the second group from the first Subtract the value of the second group from the first
group. group.
   

   
Therefore, the value of the determinant is 124. Therefore, the value of the determinant is –459.

ANSWER:   ANSWER:  
124 −459

31.  32. 

SOLUTION:   SOLUTION:  
Rewrite the first two columns in the right of the Rewrite the first two columns in the right of the
determinant. determinant.
   

   
eSolutions Manual - Powered by Cognero Find the product of the element of the diagonal.Page 15
Find the product of the element of the diagonal.
   
   
Therefore, the value of the determinant is –459. Therefore, the value of the determinant is 63.

ANSWER:  
3-7 Solving Systems of Equations Using Cramer's Rule ANSWER:  
−459 63

32.  33. 

SOLUTION:   SOLUTION:  
Rewrite the first two columns in the right of the Rewrite the first two columns in the right of the
determinant. determinant.
   

   
Find the product of the element of the diagonal. Find the product of the element of the diagonal.
   

   
Find the sum of each group. Therefore, the value of the determinant is 0.
  ANSWER:  
0
 
Subtract the value of the second group from the first
group.
34. 
 

  SOLUTION:  
Therefore, the value of the determinant is 63.
Rewrite the first two columns in the right of the
ANSWER:   determinant.
63  

33. 
 
Find the product of the element of the diagonal.
SOLUTION:  
 
Rewrite the first two columns in the right of the
determinant.
 

   
eSolutions Manual - Powered by Cognero Page 16
Find the product of the element of the diagonal. Find the sum of each group.
   
   
Therefore, the value of the determinant is 0. Therefore, the value of the determinant is –13.

ANSWER:  
3-7 Solving Systems of Equations Using Cramer's Rule ANSWER:  
0 −13

34.  35. 

SOLUTION:   SOLUTION:  
Rewrite the first two columns in the right of the Rewrite the first two columns in the right of the
determinant. determinant.
   

   
Find the product of the element of the diagonal. Find the product of the element of the diagonal.
   

   
Find the sum of each group. Find the sum of each group.
   

   
Subtract the value of the second group from the first Subtract the value of the second group from the first
group. group.
   

   
Therefore, the value of the determinant is –13. Therefore, the value of the determinant is 728.

ANSWER:   ANSWER:  
−13 728

35.  36. 

SOLUTION:   SOLUTION:  
Rewrite the first two columns in the right of the Rewrite the first two columns in the right of the
determinant. determinant.
   

   
eSolutions Manual - Powered by Cognero Page 17
Find the product of the element of the diagonal. Find the product of the element of the diagonal.
   
   
Therefore, the value of the determinant is 728. Therefore, the value of the determinant is –120.

ANSWER:  
3-7 Solving Systems of Equations Using Cramer's Rule ANSWER:  
728 −120

36.  37. 

SOLUTION:   SOLUTION:  
Rewrite the first two columns in the right of the Rewrite the first two columns in the right of the
determinant. determinant.
   

   
Find the product of the element of the diagonal. Find the product of the element of the diagonal.
   

   
Find the sum of each group. Find the sum of each group.
   

   
Subtract the value of the second group from the first Subtract the value of the second group from the first
group. group.
   

   
Therefore, the value of the determinant is –120. Therefore, the value of the determinant is –952.

ANSWER:   ANSWER:  
−120 −952

38. TRAVEL Mr. Smith’s art class took a bus trip to an


art museum. The bus averaged 65 miles per hour on
37. 
the highway and 25 miles per hour in the city. The art
museum is 375 miles away from the school, and it
took the class 7 hours to get there. Use Cramer’s
SOLUTION:   Rule to find how many hours the bus was on the
Rewrite the first two columns in the right of the highway and how many hours it was driving in the
determinant. city.
 
SOLUTION:  
Let x and y be the number of hours driving in the
highway and the city respectively.
The system of equation represents the situation is:
   
eSolutions Manual - Powered by Cognero Page 18
Find the product of the element of the diagonal.
 
 
  The bus was driving 5 hours on the highway and 2
Therefore, the value of the determinant is –952. hours in the city.

ANSWER:  
3-7 Solving Systems of Equations Using Cramer's Rule ANSWER:  
−952 5 hours on the highway, 2 hours in the city

38. TRAVEL Mr. Smith’s art class took a bus trip to an Use Cramer’s Rule to solve each system of
art museum. The bus averaged 65 miles per hour on equations.
the highway and 25 miles per hour in the city. The art 39. 6x − 5y = 73
museum is 375 miles away from the school, and it −7x + 3y = −71
took the class 7 hours to get there. Use Cramer’s
Rule to find how many hours the bus was on the SOLUTION:  
highway and how many hours it was driving in the Use Cramer’s Rule.
city.  
Let C be the coefficient matrix of the system
SOLUTION:    
Let x and y be the number of hours driving in the
highway and the city respectively.   .
The system of equation represents the situation is:
   
The solution of the system is
 
 
Solve the above equation using Cramer’s Rule.   if .
 
 

 
 

   
The bus was driving 5 hours on the highway and 2 Therefore, the solution of the system is (8, –5).
hours in the city.
ANSWER:  
ANSWER:   (8, −5)
5 hours on the highway, 2 hours in the city

Use Cramer’s Rule to solve each system of 40. 10a − 3b = −34


equations. 3a + 8b = −28
39. 6x − 5y = 73 SOLUTION:  
−7x + 3y = −71
Use Cramer’s Rule.
SOLUTION:    
Use Cramer’s Rule. Let C be the coefficient matrix of the system
   
Let C be the coefficient matrix of the system
    .

  .  
eSolutions Manual - Powered by Cognero The solution of the system is Page 19

   
The solution of the system is
 
 
Therefore, the solution of the system is (8, –5).
Therefore, the solution of the system is (–4, –2).
ANSWER:  
3-7 Solving ANSWER:  
(8, −5) Systems of Equations Using Cramer's Rule
(−4,−2)

40. 10a − 3b = −34 41. −4c − 5d = −39


3a + 8b = −28 5c + 8d = 54
SOLUTION:   SOLUTION:  
Use Cramer’s Rule. Use Cramer’s Rule.
   
Let C be the coefficient matrix of the system Let C be the coefficient matrix of the system
   

  .   .

   
The solution of the system is The solution of the system is
   

  if .   if .

   

   

   
Therefore, the solution of the system is (–4, –2). Therefore, the solution of the system is (6, 3).

ANSWER:   ANSWER:  
(−4,−2) (6, 3)

41. −4c − 5d = −39 42. −6f − 8g = −22


5c + 8d = 54 –11f + 5g = −60

SOLUTION:   SOLUTION:  
Use Cramer’s Rule. Use Cramer’s Rule.
   
Let C be the coefficient matrix of the system Let C be the coefficient matrix of the system
   

  .   .

   
The solution of the system is The solution of the system is
   
eSolutions Manual - Powered by Cognero Page 20

  if .   if .
   
Therefore, the solution of the system is (6, 3). Therefore, the solution of the system is (5, –1).
ANSWER:  
3-7 Solving Systems of Equations Using Cramer's Rule ANSWER:  
(6, 3) (5, −1)

42. −6f − 8g = −22 43. 9r + 4s = −55


–11f + 5g = −60 −5r − 3s = 36
SOLUTION:   SOLUTION:  
Use Cramer’s Rule. Use Cramer’s Rule.
   
Let C be the coefficient matrix of the system Let C be the coefficient matrix of the system
   

  .   .

   
The solution of the system is The solution of the system is
   

  if .   if .

   

   

   
Therefore, the solution of the system is (5, –1). Therefore, the solution of the system is (–3, –7).

ANSWER:   ANSWER:  
(5, −1) (−3, −7)

43. 9r + 4s = −55 44. −11u − 7v = 4


−5r − 3s = 36 9u + 4v = −24
SOLUTION:   SOLUTION:  
Use Cramer’s Rule. Use Cramer’s Rule.
   
Let C be the coefficient matrix of the system Let C be the coefficient matrix of the system
   

  .   .

   
The solution of the system is The solution of the system is
   
eSolutions Manual - Powered by Cognero Page 21

  if .   if .
   
Therefore, the solution of the system is (–3, –7).
  .
ANSWER:  
3-7 Solving Systems of Equations Using Cramer's Rule
(−3, −7)
 
44. −11u − 7v = 4 The solution of the system is
9u + 4v = −24  
SOLUTION:  
Use Cramer’s Rule.    if  .
 
Let C be the coefficient matrix of the system  
 

  .

 
The solution of the system is
 
 

  if .

 
Therefore, the solution of the system is (–8, 12).

ANSWER:  
(−8, 12)

45. 5x − 4y + 6z = 58
−4x + 6y + 3z = −13
6x + 3y + 7z = 53
 
SOLUTION:  
Therefore, the solution of the system is (4, –2, 5).
Use Cramer’s Rule.
  ANSWER:  
Let C be the coefficient matrix of the system (4, −2, 5)
 
46. 8x − 4y + 7z = 34
  . 5x + 6y + 3z = −21
3x + 7y − 8z = −85
  SOLUTION:  
The solution of the system is
eSolutions Manual - Powered by Cognero Use Cramer’s Rule. Page 22
   
Let C be the coefficient matrix of the system
 
46. 8x − 4y + 7z = 34
5x + 6y + 3z = −21
3-7 Solving
3x + 7y −Systems
8z = −85of Equations Using Cramer's Rule
 
SOLUTION:   Therefore, the solution of the system is (–3, –4, 6).
Use Cramer’s Rule.
ANSWER:  
 
Let C be the coefficient matrix of the system (−3, −4, 6)
 
47. DOUGHNUTS Mi-Ling is ordering doughnuts for a
class party. The box contains 2 dozen doughnuts,
  . some of which are plain and some of which are jelly-
filled. The plain doughnuts each cost $0.50, and the
  jelly-filled cost $0.60. If the total cost is $12.60, use
Cramer’s Rule to find how many jelly-filled
The solution of the system is
doughnuts Mi-Ling ordered.
 
 

   if  .

  SOLUTION:  
Let x and y be the number of plain and jelly-filled
doughnuts.
The system of equation that represents the situation
is:
 

 
Solve the above equation using Cramer’s Rule.
 

 
Mi-Ling ordered 6 jelly-filled doughnuts.
 
Therefore, the solution of the system is (–3, –4, 6). ANSWER:  
eSolutions Manual - Powered by Cognero 6 Page 23
ANSWER:  
(−3, −4, 6) 48. CCSS PERSEVERANCE The salary for each of
   
Therefore, the solution of the system is (–3, –4, 6). Mi-Ling ordered 6 jelly-filled doughnuts.
ANSWER:  
3-7 Solving Systems of Equations Using Cramer's Rule ANSWER:  
(−3, −4, 6) 6

47. DOUGHNUTS Mi-Ling is ordering doughnuts for a 48. CCSS PERSEVERANCE The salary for each of
class party. The box contains 2 dozen doughnuts, the stars of a new movie is $5 million, and the
some of which are plain and some of which are jelly- supporting actors each receive $1 million. The total
filled. The plain doughnuts each cost $0.50, and the amount spent for the salaries of the actors and
jelly-filled cost $0.60. If the total cost is $12.60, use actresses is $19 million. If the cast has 7 members,
Cramer’s Rule to find how many jelly-filled use Cramer’s Rule to find the number of stars in the
doughnuts Mi-Ling ordered. movie.
 
SOLUTION:  
Let x and y be the number of starts and the
supporting actors in the movie respectively.
The system of equation that represents the situation
is:
 

 
Solve the above equation using Cramer’s Rule.
SOLUTION:    
Let x and y be the number of plain and jelly-filled
doughnuts.
The system of equation that represents the situation
is:
 

 
 
Solve the above equation using Cramer’s Rule.
 

 
The number of stars in the movie is 3.
 
ANSWER:  
3

49. ARCHAEOLOGY Archaeologists found whale


bones at coordinates (0, 3), (4, 7), and (5, 9). If the
units of the coordinates are meters, find the area of
the triangle formed by these finds.

  SOLUTION:  
Mi-Ling ordered 6 jelly-filled doughnuts. The area of a triangle with vertices (a, b), (c, d) and
(f , g) is |A|, where
ANSWER:    
6

48. CCSS PERSEVERANCE The salary for each of  

eSolutions Manual - Powered by Cognero Page 24


 
Substitute the coordinates (0, 3), (4, 7), and (5, 9) in
    .
The number of stars in the movie is 3.

ANSWER:  
3-7 Solving Systems of Equations Using Cramer's Rule  
3 The solution of the system is
 
49. ARCHAEOLOGY Archaeologists found whale
bones at coordinates (0, 3), (4, 7), and (5, 9). If the
units of the coordinates are meters, find the area of    if  .
the triangle formed by these finds.
 
SOLUTION:  
The area of a triangle with vertices (a, b), (c, d) and
(f , g) is |A|, where
 

 
Substitute the coordinates (0, 3), (4, 7), and (5, 9) in
the area formula.
 

 
2
Therefore, the area of the triangle 2 m .

ANSWER:  
2
2m

Use Cramer’s Rule to solve each system of


equations.
50. 6a − 7b = −55
2a + 4b − 3c = 35
−5a − 3b + 7c = −37
SOLUTION:  
 
Use Cramer’s Rule.
Therefore, the solution of the system is (–1, 7, –3).
 
Let C be the coefficient matrix of the system ANSWER:  
 
(−1, 7, −3)

  .
51. 3a − 5b − 9c = 17
4a − 3c = 31
  −5a − 4b − 2c = −42
The solution of the system is
  SOLUTION:  
Use Cramer’s Rule.
 
   if  . Let C be the coefficient matrix of the system
eSolutions Manual - Powered by Cognero   Page 25

 
  .
4a − 3c = 31
 
−5a − 4b − 2c = −42
Therefore, the solution of the system is (4, 8, –5).
SOLUTION:  
3-7 Solving Systems of Equations Using Cramer's Rule ANSWER:  
Use Cramer’s Rule.
  (4, 8, −5)
Let C be the coefficient matrix of the system
  52. 4x − 5y = −2
7x + 3z = −47
8y − 5z = −63
  .
SOLUTION:  
  Use Cramer’s Rule.
The solution of the system is  
  Let C be the coefficient matrix of the system
 

   if  .   .

 
 
The solution of the system is
 

   if  .

 
Therefore, the solution of the system is (4, 8, –5).

ANSWER:  
(4, 8, −5)
eSolutions Manual - Powered by Cognero Page 26
52. 4x − 5y = −2
7x + 3z = −47
 
  .

3-7 Solving Systems of Equations Using Cramer's Rule  


The solution of the system is
 

   if  .

 
Therefore, the solution of the system is (–8, –6, 3).

ANSWER:  
(−8, −6, 3)

53. 7x + 8y + 9z = −149
−6x + 7y − 5z = 54  
4x + 5y − 2z = −44 Therefore, the solution of the system is
SOLUTION:  
.
Use Cramer’s Rule.
 
Let C be the coefficient matrix of the system ANSWER:  
 

  .
54. GARDENING Rob wants to build a triangular
flower garden. To plan out his garden he uses a
  coordinate grid where each of the squares represents
The solution of the system is one square foot. The coordinates for the vertices of
  his garden are (−1, 7), (2, 6), and (4, −3). Find the
area of the garden.
SOLUTION:  
   if  .
eSolutions Manual - Powered by Cognero
The area of a triangle with vertices (a, b), (c, d)Page
and27
  (f , g) is |A|, where
 
for that week.
ANSWER:   c. Was raising the price of the small drink a good
business move for the vendor? Explain your
3-7 Solving Systems of Equations Using Cramer's Rule reasoning.
SOLUTION:  
54. GARDENING Rob wants to build a triangular a. Let x, y and z be the number of small, medium and
flower garden. To plan out his garden he uses a large drinks.
coordinate grid where each of the squares represents The system of equation that represents the situation
one square foot. The coordinates for the vertices of is:
his garden are (−1, 7), (2, 6), and (4, −3). Find the  
area of the garden.
SOLUTION:  
The area of a triangle with vertices (a, b), (c, d) and
(f , g) is |A|, where  
  Solve the above equation using Cramer’s Rule.
 
 

 
Substitute the coordinates (−1, 7), (2, 6), and (4, −3)
in the area formula.
   

 
2
Therefore, the area of the garden is 12.5 ft .

ANSWER:  
2
12.5 ft

55. FINANCIAL LITERACY A vendor sells small


drinks for $1.15, medium drinks for $1.75, and large
drinks for $2.25. During a week in which he sold
twice as many small drinks as medium drinks, his
total sales were $2,238.75 for 1385 drinks.
a. Use Cramer’s Rule to determine how many of
each drink were sold.
b. The vendor decided to increase the price for small
drinks to $1.25 the next week. The next week, he
sold 140 fewer small drinks, 125 more medium
drinks, and 35 more large drinks. Calculate his sales  
for that week.
The vendor sells 650 small drinks, 325 medium drinks
c. Was raising the price of the small drink a good and 410 large drinks.
business move for the vendor? Explain your b. After increasing the price, the vendor sells (650 –
reasoning.
140) 510 small drinks, (325+125) 450 medium drinks
SOLUTION:   and (410 + 35) 445 large drinks.
eSolutions Manual - Powered by Cognero
a. Let x, y and z be the number of small, medium and Total sales for that week is = 510 × 1.25 + 450 Page
×  28
large drinks. 1.75 + 445 × 2.25 = 2426.25
The system of equation that represents the situation c. It seems like it was a good move for the vendor.
 
a. ; If ag − bf = 0, then ag = bf.
The vendor sells 650 small drinks, 325 medium drinks
and 410 large drinks. b. Sample answer: x + 3y = 8 and 2x + 6y = 12; The
b. After Systems
3-7 Solving increasingofthe price, the Using
Equations sells (650 –Rule
vendorCramer's system is dependent or inconsistent depending on the
140) 510 small drinks, (325+125) 450 medium drinks values of m and n.
and (410 + 35) 445 large drinks.
Total sales for that week is = 510 × 1.25 + 450 ×  57. REASONING What can you determine about the
1.75 + 445 × 2.25 = 2426.25 solution of a system of linear equations if the
c. It seems like it was a good move for the vendor. determinant of the coefficients is 0?
Although he sold fewer small drinks, he sold more
SOLUTION:  
medium and large drinks and on the whole, made
more money this week than in the previous week. Sample answer: There is no unique solution of the
system. There are either infinite or no solutions.
ANSWER:  
ANSWER:  
a. small: 650; medium: 325; large: 410
Sample answer: There is no unique solution of the
b. $2,426.25
system. There are either infinite or no solutions.
c. It seems like it was a good move for the vendor.
Although he sold fewer small drinks, he sold more 58. CCSS CRITIQUE James and Amber are finding
medium and large drinks and on the whole, made
more money this week than in the previous week. the value of

56. REASONING Some systems of equations cannot


be solved by using Cramer’s Rule.
 
a. Find the value of . When is the value 0?

b. Choose values for a, b, f , and g to make the


determinant of the coefficient matrix 0. What type of
system is formed?  

SOLUTION:  

a.

If ag − bf = 0, then ag = bf.
 
b. Sample answer: x + 3y = 8 and 2x + 6y = 12; The  
system is dependent or inconsistent depending on the
values of m and n. SOLUTION:  

ANSWER:   James; to find the determinant of .

a. ; If ag − bf = 0, then ag = bf. James solved this correctly because 3(−5) = −15.

b. Sample answer: x + 3y = 8 and 2x + 6y = 12; The ANSWER:  


system is dependent or inconsistent depending on the James; because 3(−5) = −15
values of m and n.
59. CHALLENGE Find the determinant of a 3 × 3
57. REASONING What can you determine about the matrix defined by
solution of a system of linear equations if the
determinant of the coefficients is 0? .
SOLUTION:    
Sample answer: There is no unique solution of the
SOLUTION:  
system. There are either infinite or no solutions.

ANSWER:  
Sample answer: There is no unique solution of the
system.
eSolutions There
Manual are either
- Powered infinite or no solutions.
by Cognero Page 29

58. CCSS CRITIQUE James and Amber are finding


b. Sample answer:
James solved this correctly because 3(−5) = −15.

ANSWER:  
3-7 Solving Systems of Equations Using Cramer's Rule c. Sample answer:
James; because 3(−5) = −15

59. CHALLENGE Find the determinant of a 3 × 3 61. WRITING IN MATH Describe the possible


matrix defined by graphical representations of a 2 × 2 system of linear
equations if the determinant of the matrix of
. coefficients is 0.
  SOLUTION:  
SOLUTION:   Sample answer: Given a 2 × 2 system of linear
equations, if the determinant of the matrix of
coefficients is 0, then the system does not have a
unique solution. The system may have no solution
and the graphical representation shows two parallel
lines. The system may have infinitely many solutions
in which the graphical representation will be the
same line.

ANSWER:  
Sample answer: Given a 2 × 2 system of linear
equations, if the determinant of the matrix of
ANSWER:   coefficients is 0, then the system does not have a
0 unique solution. The system may have no solution
and the graphical representation shows two parallel
60. OPEN ENDED Write a 2 × 2 matrix with each of lines. The system may have infinitely many solutions
the following characteristics. in which the graphical representation will be the
a. The determinant equals 0. same line.
b. The determinant equals 25.
c. The elements are all negative numbers and the 62. Tyler paid $25.25 to play three games of miniature
determinant equals −32 golf and two rides on go-karts. Brent paid $25.75 for
four games of miniature golf and one ride on the go-
SOLUTION:   karts. How much does one game of miniature golf
cost?
a. Sample answer:   A $4.25
B $4.75
b. Sample answer: C $5.25
D $5.75
c. Sample answer: SOLUTION:  
Let x and y be the cost for miniature golf and go-
karts.
ANSWER:  
The system of equation that represents this situation
is:
a. Sample answer:  

b. Sample answer: Solve the above equation using Cramer’s Rule.


 
c. Sample answer:

61. WRITING IN MATH Describe the possible


graphical representations of a 2 × 2 system of linear
equations if the determinant of the matrix of
coefficients is 0.  
SOLUTION:  
eSolutions Manual - Powered by Cognero Page 30
Sample answer: Given a 2 × 2 system of linear
equations, if the determinant of the matrix of
unique solution. The system may have no solution  
and the graphical representation shows two parallel Therefore, option C is the correct answer.
lines. The system may have infinitely many solutions
3-7 Solving
in whichSystems of Equations
the graphical Using
representation willCramer's
be the Rule ANSWER:  
same line. C

62. Tyler paid $25.25 to play three games of miniature 63. Use the table to determine the expression that best
golf and two rides on go-karts. Brent paid $25.75 for represents the number of faces of any prism having a
four games of miniature golf and one ride on the go- base with n sides.
karts. How much does one game of miniature golf  
cost?
A $4.25
B $4.75
C $5.25
D $5.75
SOLUTION:  
Let x and y be the cost for miniature golf and go-
karts.  
The system of equation that represents this situation F 2(n − 1)
is:
G 2(n + 1)
Hn+2
J 2n
Solve the above equation using Cramer’s Rule.
  SOLUTION:  
The number of faces of any prism is 2 more than the
number of sides.
Therefore, option H is the correct answer.

ANSWER:  
H
  64. SHORT RESPONSE A right circular cone has
radius 4 inches and height 6 inches. What is the
lateral area of the cone? (lateral area of cone = ,
where  = slant height)

 
Therefore, option C is the correct answer. SOLUTION:  

ANSWER:  
C

63. Use the table to determine the expression that best


represents the number of faces of any prism having a
base with n sides.  
  The slant height of the cone is
2
Lateral area =  in .

ANSWER:  

65. SAT/ACT Find the area of ΔABC.


eSolutions Manual - Powered by Cognero Page 31
 
F 2(n − 1)
G 2(n + 1)
2 2
Lateral area =  in . The area of the garden is 14 ft .
Therefore, option D is the correct answer.
ANSWER:   ANSWER:  
3-7 Solving Systems of Equations Using Cramer's Rule
D

65. SAT/ACT Find the area of ΔABC. Determine whether each matrix product is
defined. If so, state the dimensions of the
product.
66. A 4 × 2 · B2 × 6

SOLUTION:  
The inner dimensions are equal, so the product is
defined.
2 Its dimensions are 4 × 6.
A 10 units
B 12 units2 ANSWER:  
2
C 13 units yes; 4 × 6
D 14 units2
2
E 16 units 67. C5 × 4 · D5 × 3
SOLUTION:   SOLUTION:  
The coordinates of the vertices of the triangle are (2, The inner dimensions are not equal, so the product is
3), (1, –3) and (–3, 1). not defined.
The area of a triangle with vertices (a, b), (c, d) and
ANSWER:  
(f , g) is | A |, where . no

68. E2 × 7 · F7 × 1

SOLUTION:  
The inner dimensions are equal, so the product is
defined.
Its dimensions are 2 × 1.

ANSWER:  
yes; 2 × 1

2 69. BUSINESS The table lists the prices at the


The area of the garden is 14 ft . Sandwich Shoppe.
Therefore, option D is the correct answer.

ANSWER:  
D

Determine whether each matrix product is


defined. If so, state the dimensions of the
product.  
66. A 4 × 2 · B2 × 6 a. List the prices in a 4 × 3 matrix.
b. The manager decides to cut the prices of every
SOLUTION:  
item by 20%. List this new set of data in a 4 × 3
The inner dimensions are equal, so the product is matrix.
defined.
c. Subtract the second matrix from the first and
Its dimensions are 4 × 6. determine the savings to the customer for each
ANSWER:   sandwich.
 
yes; 4 × 6
SOLUTION:  
eSolutions Manual - Powered by Cognero Page 32
67. C5 × 4 · D5 × 3 a. Write the prices in a 4 × 3 matrix.

SOLUTION:  
matrix.
c. Subtract the second matrix from the first and
determine the savings to the customer for each c.
3-7 Solving Systems of Equations Using Cramer's Rule
sandwich.
 
SOLUTION:   Graph each function.
a. Write the prices in a 4 × 3 matrix. 70. f (x) = 2| x – 3| − 4
SOLUTION:  

b. Write a new matrix providing the prices after a


20% discount.

c. Subtract the second matrix from the first.


ANSWER:  

ANSWER:  

a.

b. 71. f (x) = −3| 2x | + 4


SOLUTION:  

c.

Graph each function.


70. f (x) = 2| x – 3| − 4
SOLUTION:   ANSWER:  

72. f (x) = | 3x – 1 | + 2
ANSWER:   SOLUTION:  

eSolutions Manual - Powered by Cognero Page 33


3-7 Solving Systems of Equations Using Cramer's Rule

72. f (x) = | 3x – 1 | + 2 Solve each system of equations.


73. 2x − 5y = −26
SOLUTION:  
5x + 3y = −34
SOLUTION:  
Multiply the first equation by 3 and the second
equation by 5 and then add.
 

 
ANSWER:   Substitute –8 for x in the first equation and solve for
y.
 

 
The solution is (–8, 2).

ANSWER:  
Solve each system of equations. (−8, 2)
73. 2x − 5y = −26
5x + 3y = −34 74. 4y + 6x = 10
2x − 7y = 22
SOLUTION:  
Multiply the first equation by 3 and the second SOLUTION:  
equation by 5 and then add. Multiply the second equation by –3 and with the first
  equation.
 

 
Substitute –8 for x in the first equation and solve for
y.  
 
Substitute for y in the second equation and
solve for x.
 

 
The solution is (–8, 2).

ANSWER:  
eSolutions Manual - Powered by Cognero Page 34

(−8, 2)
 
The solution is (–8, 2).
ANSWER:  
ANSWER:  
3-7 Solving Systems of Equations Using Cramer's Rule
(−8, 2)

74. 4y + 6x = 10 75. –3x – 2y = 17
2x − 7y = 22 –4x + 5y = –8

SOLUTION:   SOLUTION:  
Multiply the second equation by –3 and with the first Multiply the first equation by 5 and the second
equation. equation by 2 and then add.
   

 
Substitute –3 for x in the second equation and solve
 
for y.
Substitute for y in the second equation and  
solve for x.
 

 
The solution is (–3, –4).

ANSWER:  
(–3, –4)

 
The solution is .

ANSWER:  

75. –3x – 2y = 17
–4x + 5y = –8
SOLUTION:  
Multiply the first equation by 5 and the second
equation by 2 and then add.
 

 
eSolutions Manual - Powered by Cognero Page 35
Substitute –3 for x in the second equation and solve
for y.
 
 
Since , they are not inverses.

3-8 Solving Systems of Equations Using Inverse Matrices ANSWER:  


no

Determine whether the matrices in each pair


are inverses. 3. 

1.  SOLUTION:  
If F and G are inverses, then .
SOLUTION:    
If A and B are inverses, then .
 

 
Since , they are not inverses.

ANSWER:  
no

2.   
Since , F and G are inverses.
SOLUTION:  
ANSWER:  
If C and D are inverses, then . yes
 

4. 

SOLUTION:  
If H and I are inverses, then .
 

 
Since , they are not inverses.

ANSWER:  
no

3.   
Since , they are not inverses.
SOLUTION:  
ANSWER:  
If F and G are inverses, then . no
 
Find the inverse of each matrix, if it exists.

eSolutions Manual - Powered by Cognero 5.  Page 1

SOLUTION:  
  ANSWER:  
Since , they are not inverses.

ANSWER:  
3-8 Solving Systems of Equations Using Inverse Matrices
no

Find the inverse of each matrix, if it exists.


6. 
5. 
SOLUTION:  
SOLUTION:  

 
  Since the determinant does not equal 0, the inverse
Since the determinant does not equal 0, the inverse exists.
exists.  
 

 
  Substitute  and  .
Substitute  and  .  
 

ANSWER:   ANSWER:  

6. 
7. 

SOLUTION:  
SOLUTION:  

   
Since the determinant does not equal 0, the inverse
Since the determinant does not equal 0, the inverse
exists.
exists.
   

   
Substitute  and  .
eSolutions
Substitute  and  .
  Manual - Powered by Cognero  
Page 2
Since the determinant is equal to 0, the inverse does
not exist.

3-8 Solving Systems of Equations Using Inverse Matrices ANSWER:  


does not exist

Use a matrix equation to solve each system of


7.  equations.
9. −2x + y = 9
SOLUTION:   x +y = 3
SOLUTION:  

The matrix equation is .


   
Since the determinant does not equal 0, the inverse Find the inverse of the coefficient matrix.
exists.  
 

 
 
Substitute  and  .
Substitute  and  .
 
 

 
ANSWER:   Multiply each side of the matrix equation by the
inverse matrix.
 

8. 

SOLUTION:  

 
The solution is (–2, 5).
Since the determinant is equal to 0, the inverse does
not exist. ANSWER:  
(−2, 5)
ANSWER:  
does not exist
10. 4x − 2y = 22
Use a matrix equation to solve each system of 6x + 9y = −3
equations.
SOLUTION:  
9. −2x + y = 9
eSolutions Manual - Powered by Cognero Page 3
x +y = 3 The matrix equation is   .
SOLUTION:  
 
 
 
The solution is (–2, 5).
The solution is (4, –3).
ANSWER:  
3-8 Solving ANSWER:  
(−2, 5) Systems of Equations Using Inverse Matrices
(4, −3)

10. 4x − 2y = 22 11. −2x + y = −4
6x + 9y = −3 3x + y = 1
SOLUTION:   SOLUTION:  

The matrix equation is   . The matrix equation is   .

   
Find the inverse of the coefficient matrix. Find the inverse of the coefficient matrix.
   

   
Substitute  and  . Substitute  and  .
   

 
Multiply each side of the matrix equation by the
inverse matrix.
 
 
Multiply each side of the matrix equation by the
inverse matrix.
 

 
The solution is (1, –2).

ANSWER:  
(1, −2)
 
The solution is (4, –3). 12. MONEY Kevin had 25 quarters and dimes. The
ANSWER:   total value of all the coins was $4. How many
quarters and dimes did Kevin have?
(4, −3)
SOLUTION:  
11. −2x + y = −4 Let x and y be the number of quarters and dimes
3x + y = 1 respectively.
eSolutions Manual - Powered by Cognero
SOLUTION:   Page 4

The matrix equation is   .  
   
The solution is (1, –2). Kevin have 10 quarters and 15 dimes.

ANSWER:  
3-8 Solving Systems of Equations Using Inverse Matrices ANSWER:  
(1, −2) 10 quarters and 15 dimes

12. MONEY Kevin had 25 quarters and dimes. The Determine whether each pair of matrices are
total value of all the coins was $4. How many inverses of each other.
quarters and dimes did Kevin have?
13. 
SOLUTION:  
Let x and y be the number of quarters and dimes SOLUTION:  
respectively.
If K and L are inverses, then .
 
 

 The matrix equation is   .

 
Find the inverse of the coefficient matrix.
 
 
Since , they are not inverses.
 
Substitute  and  . ANSWER:  
  no

14. 

SOLUTION:  
If M and N are inverses, then .
 

 
Multiply each side of the matrix equation by the
inverse matrix.
 

 
Since , they are not inverses.

ANSWER:  
no

 
Kevin have 10 quarters and 15 dimes. 15. 

ANSWER:  
10 quarters and 15 dimes SOLUTION:  
If P and Q are inverses, then .
Determine whether each pair of matrices are
 
inverses of each other.
eSolutions Manual - Powered by Cognero Page 5
13. 
   
Since , they are not inverses. Since , they are not inverses.

ANSWER:  
3-8 Solving Systems of Equations Using Inverse Matrices ANSWER:  
no no

Find the inverse of each matrix, if it exists.


15. 
17. 

SOLUTION:   SOLUTION:  
If P and Q are inverses, then .
 

 
Since the determinant does not equal 0, the inverse
exists.
 

 
  Substitute  and  .
Since , they are not inverses.  
ANSWER:  
no

16. 

SOLUTION:  
If R and S are inverses, then .
  ANSWER:  

18. 

  SOLUTION:  
Since , they are not inverses.

ANSWER:  
no
 
Find the inverse of each matrix, if it exists. Since the determinant does not equal 0, the inverse
exists.
17.   

SOLUTION:  
eSolutions Manual - Powered by Cognero   Page 6
Substitute  and  .
 
3-8 Solving Systems of Equations Using Inverse Matrices

18.  19. 

SOLUTION:   SOLUTION:  

   
Since the determinant does not equal 0, the inverse Since the determinant does not equal 0, the inverse
exists. exists.
   

   
Substitute  and  . Substitute  and  .
   

ANSWER:   ANSWER:  

19.  20. 

SOLUTION:   SOLUTION:  

   
Since the determinant does not equal 0, the inverse Since the determinant does not equal 0, the inverse
exists. exists.
   

   
Substitute  and 
eSolutions Manual - Powered by Cognero
. Substitute  and  . Page 7
   
3-8 Solving Systems of Equations Using Inverse Matrices

20.  21. 

SOLUTION:   SOLUTION:  

   
Since the determinant does not equal 0, the inverse Since the determinant does not equal 0, the inverse
exists. exists.
   

   
Substitute  and  . Substitute  and  .
   

ANSWER:   ANSWER:  

21.  22. 

SOLUTION:   SOLUTION:  

   
Since the determinant does not equal 0, the inverse Since the determinant does not equal 0, the inverse
exists. exists.
   

   
Substitute
eSolutions Manual - Powered by Cognero
 and  . Substitute  and  . Page 8
   
3-8 Solving Systems of Equations Using Inverse Matrices

22.  23. 

SOLUTION:   SOLUTION:  

   
Since the determinant does not equal 0, the inverse Since the determinant does not equal 0, the inverse
exists. exists.
   

   
Substitute  and  . Substitute  and  .
   

ANSWER:   ANSWER:  

23.  24. 

SOLUTION:   SOLUTION:  

   
Since the determinant does not equal 0, the inverse Since the determinant does not equal 0, the inverse
exists. exists.
   

   
Substitute
eSolutions Manual - Powered by Cognero
 and  . Substitute  and  . Page 9
   
3-8 Solving Systems of Equations Using Inverse Matrices

24.  25. 

SOLUTION:   SOLUTION:  

   
Since the determinant does not equal 0, the inverse Since the determinant does not equal 0, the inverse
exists. exists.
   

   
Substitute  and  . Substitute  and  .
   

ANSWER:   ANSWER:  

26. BAKING Peggy is preparing a colored frosting for a


25.  cake. For the right shade of purple, she needs 25
milliliters of a 44% concentration food coloring. The
SOLUTION:   store has a 25% red and a 50% blue concentration of
food coloring. How many milliliters each of blue food
coloring and red food coloring should be mixed to
make the necessary amount of purple food coloring?
SOLUTION:  
 
Let  x and y be the amount of blue food coloring and
Since the determinant does not equal 0, the inverse
red food coloring respectively.
exists.
   

   
Simplify the second equation.
Substitute  and  .
eSolutions Manual - Powered by Cognero   Page 10
 
 
  concentration of purple food coloring.

ANSWER:  
3-8 Solving Systems of Equations Using Inverse Matrices 6 mL of the red food coloring and 19 mL of the blue
  food coloring
Simplify the second equation.
  CCSS PERSEVERANCE  Use a matrix 
equation to solve each system of equations.
  27. −x + y = 4
−x + y = −4
The matrix equation is 
  SOLUTION:  
  .

 
Find the inverse of the coefficient matrix. Since the determinant is equal to 0, the inverse does
  not exist.
Therefore, the system has no solution.

ANSWER:  
  no solution
Substitute  and  .
 
28. −x + y = 3
−2x + y = 6
SOLUTION:  
The matrix equation is
 

   .

   
Multiply each side of the matrix equation by the Find the inverse of the coefficient matrix.
inverse matrix.  
 

 
Substitute  and  .
 

 
She has to mix 6 milliliters of the red food coloring
and 19 milliliters of the blue food coloring to get 44%
concentration of purple food coloring.

ANSWER:  
6 mL of the red food coloring and 19 mL of the blue  
food coloring Multiply each side of the matrix equation by the
inverse matrix.
CCSS PERSEVERANCE  Use a matrix   
equation to solve each system of equations.
27. −x + y = 4
−x + y = −4
SOLUTION:  

eSolutions Manual - Powered by Cognero Page 11

Since the determinant is equal to 0, the inverse does  


not exist.
 
Therefore, the system has no solution.
The solution is (–3, 0).
ANSWER:  
3-8 Solving Systems of Equations Using Inverse Matrices ANSWER:  
no solution
(−3, 0)

28. −x + y = 3
−2x + y = 6 29. 

SOLUTION:   SOLUTION:  
The matrix equation is The matrix equation is
   
   .    .
  Find the inverse of the coefficient matrix.
Find the inverse of the coefficient matrix.  
 

 
 
Substitute  and  .
Substitute  and  .  
 

 
Multiply each side of the matrix equation by the  
inverse matrix. Multiply each side of the matrix equation by the
  inverse matrix.
 

 
The solution is (–3, 0).

ANSWER:    
(−3, 0) The solution is (–1, 5).

ANSWER:  
29. 
(−1, 5)

SOLUTION:   30. 3x + y = 3
The matrix equation is 5x + 3y = 6
 
SOLUTION:  
   . The matrix equation is
 
eSolutions
FindManual - Powered
the inverse by Cognero
of the coefficient matrix. Page 12
     .
30. 3x + y = 3
5x + 3y = 6 ANSWER:  
SOLUTION:  
3-8 Solving Systems of Equations Using Inverse Matrices
The matrix equation is
 
31. y − x = 5
   . 2y − 2x = 8

Find the inverse of the coefficient matrix. SOLUTION:  


  Rewrite the given system as below.
 
−x + y = 5
−2x + 2y  = 8
   
Substitute  and  .
 

 
Since the determinant is equal to 0, the inverse does
not exist.
Therefore, the system has no solution.

ANSWER:  
no solution

  32. 4x + 2y = 6
Multiply each side of the matrix equation by the 6x − 3y = 9
inverse matrix. SOLUTION:  
  The matrix equation is
 

   .

 
Find the inverse of the coefficient matrix.
 

 
Substitute  and  .
 
 
The solution is .

ANSWER:  

31. y − x = 5
2y − 2x = 8  
Multiply each side of the matrix equation by the
SOLUTION:  
inverse matrix.
Rewrite the given system as below.
eSolutions
 
  Manual - Powered by Cognero Page 13

−x + y = 5
−2x + 2y  = 8
not exist.  
Therefore, the system has no solution. The solution is (1.5, 0).

ANSWER:  
3-8 Solving Systems of Equations Using Inverse Matrices ANSWER:  
no solution (1.5, 0)

32. 4x + 2y = 6 33. 1.6y − 0.2x = 1


6x − 3y = 9 0.4y −0.1x = 0.5
SOLUTION:   SOLUTION:  
The matrix equation is Rewrite the given system as below.
   
− 0.2x + 1.6y  = 1
   .
−0.1x + 0.4y = 0.5
   
Find the inverse of the coefficient matrix. The matrix equation is
   

    .

   
Find the inverse of the coefficient matrix.
Substitute  and  .
 
 

 
Substitute  and  .
 

 
Multiply each side of the matrix equation by the
inverse matrix.
   
Multiply each side of the matrix equation by the
inverse matrix.
 

   
The solution is (1.5, 0). The solution is (–5, 0).

ANSWER:   ANSWER:  
(1.5, 0) (−5, 0)

33. 1.6y − 0.2x = 1 34. 4y − x = −2


0.4y −0.1x = 0.5 3y − x = 6

SOLUTION:   SOLUTION:  
Rewrite the given system as below. Rewrite the given system as below.
eSolutions Manual - Powered by Cognero
  − x + 4y  = –2 Page 14

− 0.2x + 1.6y  = 1 −x + 3y = 6
−0.1x + 0.4y = 0.5  
  35. 2y − 4x = 3
The solution is (–5, 0). 4x − 3y = −6

ANSWER:   SOLUTION:  
3-8 Solving Systems of Equations Using Inverse Matrices
(−5, 0) Rewrite the given system as below.
 
34. 4y − x = −2 − 4x + 2y  = 3
3y − x = 6 4x − 3y = −6
 
SOLUTION:   The matrix equation is
Rewrite the given system as below.  
− x + 4y  = –2
−x + 3y = 6     .
 
The matrix equation is  
  Find the inverse of the coefficient matrix.
 
    .

 
Find the inverse of the coefficient matrix.  
  Substitute  and  .
 

 
Substitute  and  .
 

 
  Multiply each side of the matrix equation by the
Multiply each side of the matrix equation by the inverse matrix.
inverse matrix.  
 

   
The solution is (–30, –8).
The solution is .
ANSWER:  
(−30, −8) ANSWER:  
35. 2y − 4x = 3
4x − 3y = −6
SOLUTION:   36. POPULATIONS The diagram shows the annual
Rewrite the given system as below. percent migration between a city and its suburbs.
   
− 4x + 2y  = 3
eSolutions Manual - Powered by Cognero Page 15
4x − 3y = −6
 
The matrix equation is
Multiply the matrices  and 

3-8 POPULATIONS
36. Solving Systems ofThe diagram shows
Equations Usingthe annualMatrices
Inverse .
percent migration between a city and its suburbs.
   

 
a. Write a matrix to represent the transitions in city  
population and suburb population. So, about 16,587 people lived in the city last year.
b. There are currently 16,275 people living in the city
and 17,552 people living in the suburbs. Assuming ANSWER:  
that the trends continue, predict the number of people
who will live in the suburbs next year.
c. Use the inverse of the matrix from part b to find a.
the number of people who lived in the city last year.
SOLUTION:   b. about 17,839
a. c. about 16,587

37. MUSIC The diagram shows the trends in digital


audio player and portable CD player ownership over
the past five years for Central City. Every person in
Central City has either a digital audio player or a
  portable CD player. Central City has a stable
population of 25,000 people, of whom 17,252 own
b. Multiply the matrices . digital audio players and 7748 own portable CD
players.
   

   
Therefore, there will be about 17,839 people live in a. Write a matrix to represent the transitions in player
the suburbs next year. ownership.
  b. Assume that the trends continue. Predict the
number of people who will own digital audio players
c. Find the inverse of the matrix .
next year.
c. Use the inverse of the matrix from part b to find
Let . the number of people who owned digital audio
players last year.
 
SOLUTION:  
a.

Multiply the matrices  and 

.  

  b. Multiply the matrices .

 
eSolutions Manual - Powered by Cognero Page 16
a.
 
b. Multiply
3-8 Solving the matrices
Systems .
of Equations Using Inverse Matrices b. about 20,218
c. about 4357
 
38. CCSS CRITIQUE Cody and Megan are setting up
matrix equations for the system 5x + 7y = 19 and 3y
+ 4x = 10. Is either of them correct? Explain your
reasoning.
 
So, about 20,218 people will own digital audio players
next year.
 
c. Find the inverse of the matrix .
 
 

Let .

 
   
SOLUTION:  
Multiply the matrices  and 
In the coefficient matrix, the first column are the
coefficients of the x-terms while the second column
. are the coefficients of the y-terms. Megan is correct;
Cody put 3 for x in the second equation instead of 4.
 
ANSWER:  
Megan; Cody put 3 for x in the second equation
instead of 4.

39. CHALLENGE Describe what a matrix equation


  with infinite solutions looks like.
So, about 4357 people owned digital audio players
last year. SOLUTION:  
The system would have to consist of two equations
ANSWER:   that are the same or one equation that is a multiple of
the other.

a. ANSWER:  
The system would have to consist of two equations
that are the same or one equation that is a multiple of
the other.
b. about 20,218
c. about 4357 40. REASONING Determine whether the following
statement is always, sometimes, or never true.
38. CCSS CRITIQUE Cody and Megan are setting up Explain your reasoning.
matrix equations for the system 5x + 7y = 19 and 3y A square matrix has a multiplicative inverse.
+ 4x = 10. Is either of them correct? Explain your
reasoning. SOLUTION:  
Sometimes; Sample answer: A square matrix has a
multiplicative inverse if its determinant does not equal
0.

ANSWER:  
eSolutions Manual - Powered by Cognero Page 17
Sometimes; Sample answer: A square matrix has a
  multiplicative inverse if its determinant does not equal
0.
as combination or elimination. More complex
ANSWER:   systems can be easier to solve by using matrices.
The system would have to consist of two equations
that are the
3-8 Solving same or
Systems ofone equation Using
Equations that is aInverse
multipleMatrices
of 43. The Yogurt Shoppe sells cones in three sizes: small,
the other. $0.89; medium, $1.19; and large, $1.39. One day
Santos sold 52 cones. He sold seven more medium
40. REASONING Determine whether the following cones than small cones. If he sold $58.98 in cones,
statement is always, sometimes, or never true. how many medium cones did he sell?
Explain your reasoning. A 11    
A square matrix has a multiplicative inverse. B 17    
SOLUTION:   C 24    
D 36
Sometimes; Sample answer: A square matrix has a
multiplicative inverse if its determinant does not equal SOLUTION:  
0. Let x, y, and z be the number of small size, medium
ANSWER:   size, and large size cones respectively.
 
Sometimes; Sample answer: A square matrix has a
multiplicative inverse if its determinant does not equal
0.

41. OPEN ENDED Write a matrix equation that does  


not have a solution.
Substitute    in the first equation.
SOLUTION:    

Sample answer:   any matrix that


 
has a determinant equal to 0, such as . Substitute    in the third equation.
 
ANSWER:  

Sample answer:   any matrix that


 
has a determinant equal to 0, such as . Multiply the fourth equation by 1.39 and subtract it
from 5.
 
42. WRITING IN MATH When would you prefer to
solve a system of equations using algebraic
methods, and when would you prefer to use
matrices? Explain.
 
SOLUTION:   Divide each side by –0.70.
Sample answer: Some systems in two variables can
be easier to solve by using algebraic methods such  
as combination or elimination. More complex Substitute  in (2).
systems can be easier to solve by using matrices.
ANSWER:  
 
Sample answer: Some systems in two variables can The number of medium cones is 24.
be easier to solve by using algebraic methods such So, the correct option is C.
as combination or elimination. More complex
systems can be easier to solve by using matrices. ANSWER:  
C
43. The Yogurt Shoppe sells cones in three sizes: small,
$0.89; medium, $1.19; and large, $1.39. One day 44. The chart shows an expression evaluated for
Santos sold 52 cones. He sold seven more medium different values of x. A student concludes that for all
cones
eSolutions than-small
Manual cones.
Powered If he sold $58.98 in cones,
by Cognero values of x, produces a prime number. Which value
Page 18
how many medium cones did he sell? of x serves as a counterexample to prove this
A 11     conclusion false?
The number of medium cones is 24.
So, the correct option is C.

ANSWER:    
3-8 Solving Systems of Equations Using Inverse Matrices
C Substitute  and  .
 
44. The chart shows an expression evaluated for
different values of x. A student concludes that for all
values of x, produces a prime number. Which value
of x serves as a counterexample to prove this
conclusion false?
 

 
Multiply each side of the matrix equation by the
F −4     inverse matrix.
G −3      
H 2     
J4
SOLUTION:  
21 is not a prime number. So, the correct choice is J.

ANSWER:  
J

45. SHORT RESPONSE What is the solution of the


system of equations 6a + 8b = 5 and
10a − 12b = 2?
 
SOLUTION:  
Rewrite the given system as below. The solution is .
 
6a + 8b = 5 ANSWER:  
5a − 6b = 1
 

The matrix equation is    .
46. SAT/ACT Each year at Capital High School the
  students vote to choose the theme of the
Find the inverse of the coefficient matrix. homecoming dance. The theme “A Night Under the
  Stars” received 225 votes, and “The Time of My
Life” received 480 votes. If 40% of girls voted for
“A Night Under the Stars” and 75% of boys voted
for “The Time of My Life,” how many girls and boys
  voted?
Substitute  and  .  
  A 176 boys and 351 girls
B 395 boys and 310 girls
C 380 boys and 325 girls
D 705 boys and 325 girls
E 854 boys and 176 girls
SOLUTION:  
eSolutions Manual - Powered by Cognero
Let x be the number of girls who voted and let yPage
be 19
the number of boys who voted.
Since 40% of girls voted for “A Night Under the
Stars”, 60% of girls voted for “The Time of My
So, there were 325 girls and 380 boys who voted.
ANSWER:   The correct choice is C.

3-8 Solving Systems of Equations Using Inverse Matrices ANSWER:  


C

46. SAT/ACT Each year at Capital High School the Evaluate each determinant.
students vote to choose the theme of the
homecoming dance. The theme “A Night Under the 47. 
Stars” received 225 votes, and “The Time of My
Life” received 480 votes. If 40% of girls voted for SOLUTION:  
“A Night Under the Stars” and 75% of boys voted
for “The Time of My Life,” how many girls and boys
voted?
 
A 176 boys and 351 girls
B 395 boys and 310 girls
C 380 boys and 325 girls ANSWER:  
D 705 boys and 325 girls −54
E 854 boys and 176 girls
SOLUTION:   48. 
Let x be the number of girls who voted and let y be
the number of boys who voted.
SOLUTION:  
Since 40% of girls voted for “A Night Under the
Stars”, 60% of girls voted for “The Time of My
Life”. Similarly, since 75% of boys voted for “The
Time of My Life”, remaining 25% voted for “A
Night Under the Stars”. Write a matrix equation to
represent the situation. ANSWER:  
  −62

  49. 
Solve the matrix equation.
 
SOLUTION:  

Rewrite the first two columns to the right of the


determinant.

  Find the products of the elements of the diagonals.


So, there were 325 girls and 380 boys who voted.
The correct choice is C.
            
ANSWER:  
C

Evaluate each determinant.                     

47.  Find the sum of each group.


eSolutions Manual - Powered by Cognero
                         Page 20
SOLUTION:   Subtract the sum of the second group from the sum o
the first group.
ANSWER:  

ANSWER:  
3-8 Solving Systems of Equations Using Inverse Matrices
−62

51. 
49. 

SOLUTION:  
SOLUTION:  

Rewrite the first two columns to the right of the


determinant. ANSWER:  

Find the products of the elements of the diagonals. 52. 

             SOLUTION:  
It is impossible to find the product of these matrices.

ANSWER:  
                     impossible

Find the sum of each group. 53. MILK The Yoder Family Dairy produces at most
                         200 gallons of skim and whole milk each day for
Subtract the sum of the second group from the sum o delivery to large bakeries and restaurants. Regular
the first group. customers require at least 15 gallons of skim and 21
gallons of whole milk each day. If the profit on a
gallon of skim milk is $0.82 and the profit on a gallon
The value of the determinant is 551. of whole milk is $0.75, how many gallons of each
type of milk should the dairy produce each day to
ANSWER:   maximize profits?
551 SOLUTION:  
Find each product, if possible. Let x be the number of gallons of skim milk. Let y be
the number of gallons of whole milk.
50. 

SOLUTION:  
The optimize function is .
Graph the inequalities in the same coordinate plane.

ANSWER:  

eSolutions Manual - Powered by Cognero Page 21


51. 
The vertices of the feasible region are (15, 185), (15,
21), and (179, 21).
SOLUTION:   To maximize the profit, the dairy has to produce 179
It is impossible to find the product of these matrices. gallons of skim milk and 21 gallons of whole milk.

ANSWER:  
3-8 Solving Systems of Equations Using Inverse Matrices ANSWER:  
impossible 179 gal of skim and 21 gal of whole milk

53. MILK The Yoder Family Dairy produces at most Identify the type of function represented by
200 gallons of skim and whole milk each day for each graph.
delivery to large bakeries and restaurants. Regular
customers require at least 15 gallons of skim and 21
gallons of whole milk each day. If the profit on a
gallon of skim milk is $0.82 and the profit on a gallon
of whole milk is $0.75, how many gallons of each
type of milk should the dairy produce each day to
maximize profits?
54. 
SOLUTION:  
Let x be the number of gallons of skim milk. Let y be SOLUTION:  
the number of gallons of whole milk. The graph represents a quadratic function.

ANSWER:  
quadratic

The optimize function is .


Graph the inequalities in the same coordinate plane.

55. 
SOLUTION:  
The graph is in a “V” shape. So, it represents an
absolute value function.

The vertices of the feasible region are (15, 185), (15, ANSWER:  
21), and (179, 21). absolute value

To maximize the profit, the dairy has to produce 179


gallons of skim milk and 21 gallons of whole milk.
56. 
ANSWER:  
179 gal of skim and 21 gal of whole milk SOLUTION:  
The graph is a horizontal line. So, it represents a
Identify the type of function represented by constant function.
each graph.
ANSWER:  
constant

54. 
eSolutions Manual - Powered by Cognero Page 22

SOLUTION:  
The graph represents a quadratic function.
Therefore, the solution is .

ANSWER:  
Practice Test - Chapter 3 (–8, –4)

Solve each system of equations by using either


substitution or elimination. 2. 

1.  SOLUTION:  
Multiply the first equation by –2 and add with the
SOLUTION:   second equation.
Substitute x + 4 for y in the second equation and  
solve for x.
 

 
Substitute –1 for y in the second equation and solve
  for x.
Substitute –8 for x in the first equation and solve for  
y.
 

 
Therefore, the solution is . Therefore, the solution is .

ANSWER:  
(–8, –4) ANSWER:  

2. 
3. 
SOLUTION:  
Multiply the first equation by –2 and add with the
SOLUTION:  
second equation.
  Multiply the first and the second equation by 4 and 5
then add.
 

 
Substitute –1 for y in the second equation and solve
for x.  
  Substitute –8 for y in the second equation and solve
for x.
 

Therefore, the solution is .


 
ANSWER:   Therefore, the solution is .
eSolutions Manual - Powered by Cognero Page 1

ANSWER:  
(4, –8)
 
Therefore, the solution is .
ANSWER:  
Practice Test - Chapter 3 ANSWER:  
(4, –8)

3.  4. 

SOLUTION:   SOLUTION:  
Multiply the first and the second equation by 4 and 5 Multiply the first and the second equation by 5 and 3
then add. respectively then add.
   

   
Substitute –8 for y in the second equation and solve Substitute –2 for x in the second equation and solve
for x. for y.
   

   
Therefore, the solution is . Therefore, the solution is .

ANSWER:   ANSWER:  
(4, –8) (–2, –1)

5. MULTIPLE CHOICE Which graph shows the


4. 
solution of the system of inequalities?

SOLUTION:  
Multiply the first and the second equation by 5 and 3
respectively then add.
A
 

 
Substitute –2 for x in the second equation and solve  
for y. B
 

 
Therefore, the solution is .
C
ANSWER:  
eSolutions Manual - Powered by Cognero Page 2

(–2, –1)
Option B is the correct answer.

Practice Test - Chapter 3 ANSWER:  


B
C
Solve each system of inequalities by graphing.
6. 

SOLUTION:  

ANSWER:  

SOLUTION:  
Graph the inequalities.

Option B is the correct answer. 7. 

ANSWER:   SOLUTION:  
B

Solve each system of inequalities by graphing.


6. 

SOLUTION:  

ANSWER:  

ANSWER:  

8. 
eSolutions Manual - Powered by Cognero Page 3

SOLUTION:  
Practice Test - Chapter 3
10. SALONS  Sierra King is a nail technician. She allots
20 minutes for a manicure and 45 minutes for a
pedicure in her 7-hour word day. No more than 5
8.  pedicures can be scheduled each day. The prices are
$18 for a manicure and $45 for a pedicure. How
SOLUTION:   many manicures and pedicures should Ms. King
schedule to maximize her daily income? What is her
maximum daily income?
SOLUTION:  
Let m represent the number of manicures and p
represent the number of pedicures.
Write the system of constraints for the scenario.
Since she cannot do a negative procedure, both
variables must be greater than or equal to 0. 
0 ≤ p  ≤ 5
ANSWER:   m ≥ 0
20m + 45p ≤ 420
 
Graph the system of inequalities and find the vertices
of the feasible region.

9. 

SOLUTION:  

The vertices of the feasible region are (0 ,0), (0, 5),


(9.75, 5), and (21, 0).
 
Since the price for each manicure is $18 and the
price for each pedicure is $45, the function that
represents her daily income is I(m, p ) = 185m + 45p . 
Sierra can only do a whole number of manicures and
ANSWER:   pedicures so substitute (0, 0), (0, 5), (9, 5), and (18, 0)
in the income function to determine the maximum.
 

 
10. SALONS  Sierra King is a nail technician. She allots The maximum income will be produced when she
20 minutes for a manicure and 45 minutes for a schedules 9 manicures and 5 pedicures.
pedicure in her 7-hour word day. No more than 5 This will produce a maximum income of $387.
eSolutions Manual - Powered by Cognero Page 4
pedicures can be scheduled each day. The prices are
$18 for a manicure and $45 for a pedicure. How ANSWER:  
many manicures and pedicures should Ms. King 9 manicures and 5 pedicures; $387
 
 
Practice Test - Chapter 3 Substitute 147 for z in the fourth equation and solve
The maximum income will be produced when she for x.
schedules 9 manicures and 5 pedicures.  
This will produce a maximum income of $387.

ANSWER:  
9 manicures and 5 pedicures; $387
 
11. COLLEGE FOOTBALL In a recent year, Darren Substitute 147 for z in the third equation and solve for
McFadden of Arkansas placed second overall in the y.
Heisman Trophy voting. Players are given 3 points  
for every first-place vote, 2 points for every second-
place vote, and 1 point for every third-place vote.
McFadden received 490 total votes for first, second,
and third place, for a total of 878 points. If he had 4
more than twice as many second-place votes as  
third-place votes, how many votes did he receive for Therefore, he received 45 first-place, 298 second-
each place? place and 147 third-place votes.
SOLUTION:   ANSWER:  
Let x, y and z be the number of first, second and 45 first, 298 second, 147 third
third place vote.
The system of equations represent this situation is: Perform the indicated operations. If the matrix
  does not exist, write impossible.

12. 

  SOLUTION:  
Substitute 2y + 4 for z in the first and second Distribute the scalar.
equation and simplify.  
 

   
Multiply the fourth equation by –3 and add with the Add corresponding elements.
fifth equation.  
 

 
Substitute 147 for z in the fourth equation and solve
for x.
   
ANSWER:  
eSolutions Manual - Powered by Cognero Page 5

 
Therefore, he received 45 first-place, 298 second-
place and 147 third-place votes. ANSWER:  
ANSWER:  
Practice Test - Chapter 3
45 first, 298 second, 147 third

Perform the indicated operations. If the matrix


does not exist, write impossible.
14. 

12. 
SOLUTION:  

SOLUTION:  
Distribute the scalar.
 

ANSWER:  

 
Add corresponding elements.
 
15. 

SOLUTION:  
The inner dimensions are not equal. So, the matrix
subtraction is impossible.

ANSWER:  
impossible
 
16. MULTIPLE CHOICE What is the value of
ANSWER:  
?

 
F –44
G 44
13. 
H

SOLUTION:  
J
 
SOLUTION:  

ANSWER:  

eSolutions Manual - Powered by Cognero Page 6

14.   
Therefore, option F is the correct answer.
The inner dimensions are not equal. So, the matrix  
subtraction is impossible. Therefore, option F is the correct answer.

ANSWER:  
Practice Test - Chapter 3 ANSWER:  
impossible F

16. MULTIPLE CHOICE What is the value of Find the inverse of each matrix, if it exists.
 
? 17. 

  SOLUTION:  
F –44
G 44 Let .

H  
det (A) = 5
J  
 
SOLUTION:  

ANSWER:  

 
Therefore, option F is the correct answer.

ANSWER:  
F 18. 

Find the inverse of each matrix, if it exists. SOLUTION:  


 
Let .
17. 
 
SOLUTION:   det (A) = –3
 
Let .

 
det (A) = 5
 

ANSWER:  

ANSWER:  

19. 
eSolutions Manual - Powered by Cognero Page 7

SOLUTION:  
18. 
The value of the matrix is zero. So the inverse does
not exist.

Practice Test - Chapter 3 ANSWER:  


No Inverse Exists

Use Cramer’s Rule to solve the following


19.  system of equations.
21. 2x − y = −9
SOLUTION:   x + 2y = 8

Let . SOLUTION:  
Cramer’s Rule.
det (A) = 0  
The value of the matrix is zero. So the inverse does Let C be the coefficient matrix of the system
not exist.  
ANSWER:     .
No Inverse Exists
 
The solution of the system is
20.   

SOLUTION:  
  if .
Let .
 
det (A) = 0
The value of the matrix is zero. So the inverse does
not exist.

ANSWER:  
No Inverse Exists  
Use Cramer’s Rule to solve the following
system of equations.
21. 2x − y = −9
x + 2y = 8
SOLUTION:  
Cramer’s Rule.
   
Let C be the coefficient matrix of the system Therefore, the solution of the system is (–2, 5).
 
ANSWER:  
  . (−2, 5)

 
The solution of the system is 22. x − y + 2z = 0
  3x + z = 11
−x + 2y = 0
SOLUTION:  
  if .
Cramer’s Rule.
   
Let C be the coefficient matrix of the system
 

  .
eSolutions Manual - Powered by Cognero Page 8
   
The solution of the system is
SOLUTION:  
Cramer’s Rule.
 
Practice Test - Chapter 3
Let C be the coefficient matrix of the system
 

  .

 
The solution of the system is
 

   if  .

 
Therefore, the solution of the system is (4, 2, –1).

ANSWER:  
(4, 2, −1)

eSolutions Manual - Powered by Cognero Page 9


SOLUTION:  
dimensions

Study Guide and Review - Chapter 3 ANSWER:  


dimensions

Choose the term from above to complete each 6. A system of equations is _________if it has at least
sentence. one solution.
1. A feasible region that is open and can go on forever
is called__________. SOLUTION:  
consistent
SOLUTION:  
unbounded ANSWER:  
consistent
ANSWER:  
unbounded 7. The ______________ matrix is a square matrix
that, when multiplied by another matrix, equals that
same matrix.
2. To _________means to seek the best price or profit
using linear programming. SOLUTION:  
SOLUTION:   identity
Optimize ANSWER:  
ANSWER:   identity
optimize 8. The _________is the point at which the income
equals the cost.
3. A matrix that contains the constants in a system of
equations is called a(n)  ______________. SOLUTION:  
SOLUTION:   break-even point
constant matrix ANSWER:  
break-even point
ANSWER:  
constant matrix 9. A system of equations is __________ if it has no
solutions.
4. A matrix can be multiplied by a constant called a(n)
_________________________. SOLUTION:  
inconsistent
SOLUTION:  
scalar ANSWER:  
inconsistent
ANSWER:  
scalar 10. If the product of two matrices is the identity matrix,
they are ______________.
5. The _________ of a matrix with 4 rows and 3
columns are . SOLUTION:  
inverses
SOLUTION:  
dimensions ANSWER:  
inverses
ANSWER:  
dimensions Solve each system of equations by graphing.

6. A system of equations is _________if it has at least 11. 


one solution.
SOLUTION:   SOLUTION:  
consistent Graph both equations on the coordinate plane.
 
ANSWER:  
consistent
eSolutions Manual - Powered by Cognero Page 1
7. The ______________ matrix is a square matrix
that, when multiplied by another matrix, equals that
SOLUTION:   Since the graph of both the equations coincides, the
inverses system has infinitely many solutions.

ANSWER:  
Study Guide and Review - Chapter 3 ANSWER:  
inverses infinitely many solutions

Solve each system of equations by graphing.


11.  13. 

SOLUTION:   SOLUTION:  
Graph both equations on the coordinate plane. Graph both equations on the coordinate plane.
   

   
The solution of the system is (0, 2). The solution of the system is (–3, 4).

ANSWER:   ANSWER:  
(0, 2) (–3, 4)

14. 

12. 
SOLUTION:  
Graph both equations on the coordinate plane.
 
SOLUTION:  
Graph both equations on the coordinate plane.
 

 
Since the graphs are parallel, the lines never
intersect. So, the system has no solution.
 
Since the graph of both the equations coincides, the ANSWER:  
system has infinitely many solutions. no solution
ANSWER:   15. LAWN CARE André and Paul each mow lawns. 
infinitely many solutions André charges a $30 service fee and $10 per hour. 
Paul charges a $10 service fee and $15 per hour.
After how many hours will André and Paul charge 
13.  the same amount?
SOLUTION:  
SOLUTION:   Let x = number of hours mowed.
Graph
eSolutions both- Powered
Manual equations on the coordinate plane.
by Cognero   Page 2
 
 
Since the graphs are parallel, the lines never The solution is (2, 4).
intersect. So, the system has no solution.
ANSWER:  
ANSWER:  
Study Guide and Review - Chapter 3 (2, 4)
no solution

15. LAWN CARE André and Paul each mow lawns. 


17. 
André charges a $30 service fee and $10 per hour. 
Paul charges a $10 service fee and $15 per hour.
After how many hours will André and Paul charge  SOLUTION:  
the same amount? Substitute –2y = –5x + 4 in the equation –2y + x =
12.
SOLUTION:  
 
Let x = number of hours mowed.
 

 
  Substitute x = –2 in the equation .
Therefore, André and Paul will charge the same   
amount for 4 hours.

ANSWER:  
4 hours

Solve each system of equations by using either


substitution or elimination.
 
16.  The solution is (–2, –7).

ANSWER:  
SOLUTION:  
(–2, –7)
Substitute y = 6 – x in the equation .
 
18. 

SOLUTION:  
Substitute y = x – 7 in the equation .
 

 
Substitute x = 2 in y = 6 – x.
   
Substitute x = 5.25 in the equation .
 
 
The solution is (2, 4).

ANSWER:  
(2, 4)
 
The solution is (5.25, –1.75).
17. 
eSolutions Manual - Powered by Cognero Page 3
ANSWER:  
SOLUTION:   (5.25, –1.75)
Substitute –2y = –5x + 4 in the equation –2y + x =
   
The solution is (–2, –7). The solution is (5.25, –1.75).

ANSWER:  
Study Guide and Review - Chapter 3 ANSWER:  
(–2, –7) (5.25, –1.75)

18.  19. 

SOLUTION:   SOLUTION:  
Substitute y = x – 7 in the equation .
 

 
Substitute  in the equation  .
 
Substitute x = 5.25 in the equation .  
 

 
The solution is (5.25, –1.75).

ANSWER:  
(5.25, –1.75)

19.   
Substitute x = 3 in the equation .
SOLUTION:    

 
Substitute  in the equation  .  
The solution is (3, 5).
 
ANSWER:  
(3, 5)

20. SCHOOL SUPPLIES At an office supply store,


Emilio bought 3 notebooks and 5 pens for $13.75. If a
notebook costs $1.25 more than a pen, how much
does a notebook cost? How much does a pen cost?
SOLUTION:  
Let x = cost of a note book. Let y = cost of a pen.
The system of equations representing the situation is:
 

 
eSolutions Manual - Powered by Cognero Page 4
Substitute x = 3 in the equation .  
  Substitute x = y + 1.25 in the equation
  Therefore, the cost of a note book is $2.50 and the
The solution is (3, 5). cost of a pen is $1.25.

ANSWER:  
Study Guide and Review - Chapter 3 ANSWER:  
(3, 5) notebook: $2.50; pen: $1.25

20. SCHOOL SUPPLIES At an office supply store, Solve each system of inequalities by graphing.
Emilio bought 3 notebooks and 5 pens for $13.75. If a
21. 
notebook costs $1.25 more than a pen, how much
does a notebook cost? How much does a pen cost?
SOLUTION:  
SOLUTION:  
Graph the system of inequalities in the same
Let x = cost of a note book. Let y = cost of a pen. coordinate plane.
The system of equations representing the situation is:  
 

 
Substitute x = y + 1.25 in the equation
.
 

ANSWER:  

 
Substitute y = 1.25 in the equation .
 

 
Therefore, the cost of a note book is $2.50 and the
cost of a pen is $1.25. 22. 
ANSWER:  
SOLUTION:  
notebook: $2.50; pen: $1.25
Graph the system of inequalities in the same
Solve each system of inequalities by graphing. coordinate plane.
 
21. 

SOLUTION:  
Graph the system of inequalities in the same
coordinate plane.
 

ANSWER:  

eSolutions Manual - Powered by Cognero Page 5


Study Guide and Review - Chapter 3

22.  23. 

SOLUTION:   SOLUTION:  
Graph the system of inequalities in the same Graph the system of inequalities in the same
coordinate plane. coordinate plane.
   

ANSWER:   ANSWER:  

23.  24. 

SOLUTION:   SOLUTION:  
Graph the system of inequalities in the same Graph the system of inequalities in the same
coordinate plane. coordinate plane.
   

ANSWER:   ANSWER:  

eSolutions Manual - Powered by Cognero Page 6


Study Guide and Review - Chapter 3

25. JEWELRY Payton makes jewelry to sell at her


24. 
mother’s clothing store. She spends no more than 3
hours making jewelry on Saturdays. It takes her 15
SOLUTION:   minutes to set up her supplies and 25 minutes to
Graph the system of inequalities in the same make each bracelet. Draw a graph that represents
coordinate plane. this.
 
SOLUTION:  
Let x = number of bracelets, and y = number of
minutes.
The system of inequalities representing the situation
is:
 

 
Graph the inequalities in the same coordinate plane.
ANSWER:    

25. JEWELRY Payton makes jewelry to sell at her


mother’s clothing store. She spends no more than 3
hours making jewelry on Saturdays. It takes her 15 ANSWER:  
minutes to set up her supplies and 25 minutes to
make each bracelet. Draw a graph that represents
this.
SOLUTION:  
Let x = number of bracelets, and y = number of
minutes.
The system of inequalities representing the situation
is:
 

 
Graph the inequalities in the same coordinate plane.
  26. FLOWERS A florist can make a grand arrangement
in 18 minutes or a simple arrangement in 10 minutes.
The florist makes at least twice as many of the
simple arrangements as the grand arrangements. The
florist can work only 40 hours per week. The profit
on the simple arrangements is $10 and the profit on
eSolutions Manual - Powered by Cognero Page 7
the grand arrangements is $25. Find the number and
type of arrangements that the florist should produce
to maximize profit.
126 simple arrangements and 63 grand
arrangements.

Study Guide and Review - Chapter 3 ANSWER:  


126 simple and 63 grand

26. FLOWERS A florist can make a grand arrangement 27. MANUFACTURING A shoe manufacturer makes
in 18 minutes or a simple arrangement in 10 minutes. outdoor and indoor soccer shoes. There is a two-step
The florist makes at least twice as many of the process for both kinds of shoes. Each pair of outdoor
simple arrangements as the grand arrangements. The shoes requires 2 hours in step one and 1 hour in step
florist can work only 40 hours per week. The profit two, and produces a profit of $20. Each pair of
on the simple arrangements is $10 and the profit on indoor shoes requires 1 hour in step one and 3 hours
the grand arrangements is $25. Find the number and in step 2, and produces a profit of $15. The company
type of arrangements that the florist should produce has 40 hours of labor available per day for step one
to maximize profit. and 60 hours available for step two. What is the
manufacturer’s maximum profit? What is the
SOLUTION:   combination of shoes for this profit?
Let x be the number of simple arrangement and y be
the number of grand arrangements. SOLUTION:  
  Let x be the number of pair of outdoor shoes and y
be the number of pair of indoor shoes.
 
 
The optimize function is .
 
 
Graph the inequalities in the same coordinate plane. The optimize function is .
   
Graph the inequalities in the same coordinate plane.
 

 
The vertices of the feasible region are (0, 0), (240, 0)  
and (126, 63). The vertices of the feasible region are (0, 20), (20, 0)
  and
(12, 16).
 

 
So, to maximize the profit, the florist should produce
126 simple arrangements and 63 grand  
arrangements. The manufacturer’s maximum profit is $480. For this
ANSWER:   profit, the manufacturer has to produce 12 outdoor
and 16 indoor pair of shoes.
126 simple and 63 grand
ANSWER:  
27. MANUFACTURING A shoe manufacturer makes
$480; 12 outdoor, 16 indoor
outdoor and indoor soccer shoes. There is a two-step
process for both kinds of shoes. Each pair of outdoor
eSolutions Manual - Powered by Cognero Solve each system of equations. Page 8
shoes requires 2 hours in step one and 1 hour in step
two, and produces a profit of $20. Each pair of
indoor shoes requires 1 hour in step one and 3 hours 28. 
 
profit, the manufacturer has to produce 12 outdoor The solution of the system is (–23, –8, –6).
and 16 indoor pair of shoes.
ANSWER:  
ANSWER:  
Study Guide and Review - Chapter 3 (–23, –8, –6)
$480; 12 outdoor, 16 indoor

Solve each system of equations.


29. 
28. 

SOLUTION:  
SOLUTION:  

 
  Substitute (2x – 14) for z in the equations
Substitute (3c + 10) for b in the equation .
.  
 

 
 
Substitute  in the equation 
Substitute c = –6 in the equation .
  .
 

 
Substitute b = –8 and c = –6 in the equation
.
 
 
Substitute x = 5 in the equation .
 

 
The solution of the system is (–23, –8, –6).  
Substitute x = 5 in the equation .
ANSWER:    
(–23, –8, –6)

29. 
 
The solution of the system is (5, –5, –4).
SOLUTION:  
ANSWER:  
(5, –5, –4)
 
Substitute
eSolutions Manual (2x – 14)byfor
- Powered z in the
Cognero equations 30. AMUSEMENT PARKS Dustin, Luis, and Marci Page 9
. went to an amusement park. They purchased snacks
from the same vendor. Their snacks and how much
  they paid are listed in the table. How much did each
 
The solution of the system is (5, –5, –4).

ANSWER:  
Study Guide and Review - Chapter 3  
(5, –5, –4) Substitute x = 3.25 and z = 2.50 in the equation (1).
 
30. AMUSEMENT PARKS Dustin, Luis, and Marci
went to an amusement park. They purchased snacks
from the same vendor. Their snacks and how much
they paid are listed in the table. How much did each
snack cost?
 

 
Therefore, the cost of a hot dog is $3.25. The cost of
SOLUTION:   a popcorn is $2.25 and the cost of a soda is $2.50.
Let x = cost of a hot dog. ANSWER:  
Let y = cost of a popcorn.
Let z = cost of a soda. hot dog: $3.25; popcorn: $2.25; soda: $2.50
  Perform the indicated operations. If the matrix
The system of equations representing the situations does not exist, write impossible.
is:  
 
31. 
 
SOLUTION:  
 
Use equation (3) in (1).
Substitute x + 2y = 10.25 − z in equation (1).
 

ANSWER:  

 
Substitute z = 2.50 in equation (2).
 
32. 

SOLUTION:  

 
Substitute x = 3.25 and z = 2.50 in the equation (1).
  ANSWER:  

33. RETAIL Current Fashions buys shirts, jeans and


shoes from a manufacturer, marks them up, and then
eSolutions Manual - Powered by Cognero sells them. The table shows the purchase price Page
and 10
the selling price.
 
ANSWER:  
c.
Study Guide and Review - Chapter 3

33. RETAIL Current Fashions buys shirts, jeans and Find each product, if possible.
shoes from a manufacturer, marks them up, and then
34. 
sells them. The table shows the purchase price and
the selling price.
  SOLUTION:  

ANSWER:  
[ 62 ]
 
a. Write a matrix for the purchase price.
b. Write a matrix for the selling price.
c. Use matrix operations to find the profit on 1 shirt, 35. 
1 pair of jeans, and 1 pair of shoes.
SOLUTION:   SOLUTION:  
a. Purchase price:

b. Selling price: ANSWER:  

c. Subtract the matrices.


36. 

SOLUTION:  
The inner dimensions of the matrices are not equal.  
ANSWER:   So, the matrices cannot be multiplied.

ANSWER:  
a.
undefined

37. GROCERIES Martin bought 1 gallon of milk, 2


apples, 4 frozen dinners, and 1 box of cereal. The
b.
following matrix shows the prices for each item
respectively.
[ $2.59    $0.49    $5.25    $3.99 ]
Use matrix multiplication to find the total amount of
c.
money Martin spent at the grocery store.
SOLUTION:  
Find each product, if possible.
34. 

SOLUTION:  
eSolutions Manual - Powered by Cognero So, Martin spent $28.56. Page 11

ANSWER:  
$28.56
The inner dimensions of the matrices are not equal.  
So, the matrices cannot be multiplied.

ANSWER:  
Study Guide and Review - Chapter 3 ANSWER:  
undefined −44

37. GROCERIES Martin bought 1 gallon of milk, 2 Use Cramer’s Rule to solve each system of
apples, 4 frozen dinners, and 1 box of cereal. The equations.
following matrix shows the prices for each item 40. 3x − y = 0
respectively. 5x + 2y = 22
[ $2.59    $0.49    $5.25    $3.99 ]
Use matrix multiplication to find the total amount of SOLUTION:  
money Martin spent at the grocery store.
Let   .
SOLUTION:  
 

So, Martin spent $28.56.  

ANSWER:  
$28.56

Evaluate each determinant.

38. 
 
SOLUTION:  

ANSWER:  
−34  
Therefore, the solution is (2, 6).

ANSWER:  
39.  (2, 6)

SOLUTION:   41. 5x + 2y = 4
3x + 4y + 2z = 6
7x + 3y + 4z = 29
SOLUTION:  

Let   .
ANSWER:  
−44  

Use Cramer’s Rule to solve each system of


equations.
40. 3x − y = 0
5x + 2y = 22  

SOLUTION:  
eSolutions Manual - Powered by Cognero Page 12

Let   .
 
 
Therefore, the solution is (2, 6).
The solution is (2, −3, 6).
ANSWER:  
Study ANSWER:  
(2,Guide
6) and Review - Chapter 3
(2, −3, 6)

41. 5x + 2y = 4 42. JEWELRY Alana paid $98.25 for 3 necklaces and 2


3x + 4y + 2z = 6 pairs of earrings. Petra paid $133.50 for 2 necklaces
7x + 3y + 4z = 29 and 4 pairs of earrings. Use Cramer’s Rule to find
out how much 1 necklace costs and how much 1 pair
SOLUTION:   of earrings costs.
SOLUTION:  
Let   . Let x be the number of necklaces and y be the
number of pairs of earrings.
   

 
  Let .

 
 

 
So, the cost of 1 necklace is $15.75 and a pair of
earrings is $25.50.
  ANSWER:  
The solution is (2, −3, 6). necklace: $15.75; pair of earrings: $25.50
ANSWER:   Find the inverse of each matrix, if it exists.
(2, −3, 6)
43. 
42. JEWELRY Alana paid $98.25 for 3 necklaces and 2
pairs of earrings. Petra paid $133.50 for 2 necklaces SOLUTION:  
and 4 pairs of earrings. Use Cramer’s Rule to find
out how much 1 necklace costs and how much 1 pair
of earrings
eSolutions Manual - costs.
Powered by Cognero Page 13

SOLUTION:  
Let x be the number of necklaces and y be the
So, the cost of 1 necklace is $15.75 and a pair of
earrings is $25.50. ANSWER:  
ANSWER:  
Study Guide and Review - Chapter 3
necklace: $15.75; pair of earrings: $25.50

Find the inverse of each matrix, if it exists.


45. 
43. 
SOLUTION:  
SOLUTION:  

Since the determinant is 0, the inverse does not exist.

ANSWER:  
 
No inverse exists.
Since the determinant is non-zero, the inverse exists.
  Use a matrix equation to solve each system of
equations.

46. 
 
ANSWER:   SOLUTION:  

Let .

44. 
 
SOLUTION:  

 
Since the determinant is non-zero, the inverse exists.
 
 
The solution of the system is (8, −12).

ANSWER:  
(8, −12)

ANSWER:  
47. 

SOLUTION:  

45.  Let .

SOLUTION:    

eSolutions Manual - Powered by Cognero


  Page 14
Since the determinant is 0, the inverse does not exist.
 
 
The solution of the system is (8, −12).
The solution of the system is (2, 1).
ANSWER:  
Study ANSWER:  
(8,Guide
−12) and Review - Chapter 3
(2, 1)

48. HEALTH FOOD Heath sells nuts and raisins by


47.  the pound. Sonia bought 2 pounds of nuts and 2
pounds of raisins for $23.50. Drew bought 3 pounds
SOLUTION:   of nuts and 1 pound of raisins for $22.25. What is the
cost of 1 pound of nuts and 1 pound of raisins?
Let .
SOLUTION:  
  Let x = cost of 1 pound of nuts and y = cost of 1
pound of raisins.
 

 
 

The matrix equation is .

The inverse of  is  .


   
The solution of the system is (2, 1).

ANSWER:  
(2, 1)

48. HEALTH FOOD Heath sells nuts and raisins by


the pound. Sonia bought 2 pounds of nuts and 2
pounds of raisins for $23.50. Drew bought 3 pounds  
of nuts and 1 pound of raisins for $22.25. What is the The cost of 1 pound of nuts is $5.25 and 1 pound of
cost of 1 pound of nuts and 1 pound of raisins? raisins is $6.50.

SOLUTION:   ANSWER:  
Let x = cost of 1 pound of nuts and y = cost of 1 nuts: $5.25 per pound; raisins: $6.50 per pound
pound of raisins.
 

The matrix equation is .

The inverse of  is  .

  Manual - Powered by Cognero


eSolutions Page 15

The cost of 1 pound of nuts is $5.25 and 1 pound of


raisins is $6.50.

You might also like